You are on page 1of 268

Past ore part 1 Q & A from DJC groups

1. 'What is the least level of platelets that you can do extraction?'

Answer. If the platelet level is less than 50x10^9 per/l then you cannot proceed
at all for the extraction. generally anything between 50 and 100x10^9 is
considered low in which case though you can proceed with the extraction but
you will have to be careful while dealing with it like you should suture the
socket, use proper packs and stuff. and if the platelet count is above 100
then its okay to do any kind of extraction normally.

1 Milliliter (ml)
1000 Microliter (µl)
=
1 Centiliter (cl) = 10000 Microliter (µl)
1 Deciliter (dl) = 100000 Microliter (µl)
1 Liter (l) = 1000000 Microliter (µl)

a. Regional LA blocks are given, if the platelets levels are above 30x109 /L.
b. Haemostasis of dentoalveolar surgery is adequate if platelet levels are
above 50x109/L.
c. Major surgery requires platelets level above 75x10^9/L.

2. Waste disposal?
Answer. Wedges, silicone syringe tips have to go on the sharps as they can perforate the orange
bag, Also the matrix band goes into sharps bin.

3. When performing operative treatment in patient's oral cavity, you put you finger
support on:

1- Tooth, 2- Gingiva, 3- Cheek, 4- Chin, 5- Lip


Answer. Your finger rest is ideally on tooth.

4. EXPLAIN IN DETAIL WHY SHOULD WE GIVE 5 RESCUE BREATHS FIRST IN CHILD


FOR CPR?
Answer. Because children have ASPHIXIAL ARREST so the most important thing is OXYGEN.

5. What is the dose of Amoxicillin oral suspension


Answer 125mg/5ml-kid dose

6. What is the year of when RIDDOR regulation came out?


Answer 1995 (Reporting of Injuries, Diseases and Dangerous Occurrences Regulations 1995)

7. What is the normal level of Glucose on a BM stick


Answer      4-7m mol

8.What is the altered cast technique?


Answer. Special technique for mucosa and tooth-born partial denture (Kennedy’s class1&2)?
Mucocompressive/mucostatic impression.

9. A non synovial joint with dense connective tissue between two bony surfaces

1 synchondrosis

2 syndesmosis

3 symphysis

4 suture
Answer. Suture

Explanation:
Both Suture and Syndesmoses are connected by dense connective tissue and is the answer for the
question.
Most appropriate answer is Suture for this question.

Fibrous joints are connected by dense connective tissue, consisting mainly of collagen.

These joints are also called "fixed" or "immoveable" joints, because they do not move. These joints
have no joint cavity and are connected via fibrous connective tissue. The skull bones are connected
by fibrous joints.
Types
* Sutures are found between bones of the skull. In fetal skulls the sutures are wide to allow slight
movement during birth. They later become rigid (synarthrodial).
* Syndesmoses are found between long bones of the body, such as the radius and ulna in forearm
and the fibula and tibia in leg. Unlike other fibrous joints, syndesmoses are moveable
amphiarthrodial), albeit not to such degree as synovial joints.
* Gomphosis is a joint between the root of a tooth and the sockets in the maxilla or mandible.

Synchondroses and symphyses: Are cartilaginous joints connected entirely by cartilage


(fibrocartilage or hyaline)

A Synovial joint, also known as a diarthrosis, is the most common and most movable type of joint
in the body of a mammal. As with most other joints, synovial joints achieve movement at the point
of contact of the articulating bones.

Examples for Synovial joint.


The carpals of the wrist, acromioclavicular joint

Elbow (between the humerus and the ulna)

additional info refer


http://www.teachpe.com/anatomy/joints.php

10. Ameloblastoma is the most aggressive

11. What is required to increase efficiency and reduce fatigue of universal


curettes?
12. Ideal angle required for sub gingival curettage from universal curettes?

13. Suppuration is mainly the result of the combined action of four factors which
of the following is not one of them?
1 necrosis
2 presence of lymphocytes
3 collection of neutrophils
4 accumulation of tissue fluid
5 autolysis by proteolytic enzymes

Answer: presence of lymphocytes


the main cells involved are neutrophils and macrophages, there’re proteolytic enzymes and there is
liquefactive necrosis and pus definitely has tissue fluid

14. What would over trituration of silver and mercury cause and what would under
trituration cause?

Answer:
Overtrituration causes: shrinkage on setting
undertrituration causes: porosities and voids

15. As a newly qualified dentist you are offered a contract in which remuneration is on capitation
basis.

What is the most important and well-recognized feature of this method of remuneration?

A. Tends to encourage under-prescribing

B. Tends to encourage high technical quality of work

C. Tends to encourage high output of procedures

D. Tends to encourage over-prescribing

E. Tends to encourage low technical quality of work


Answer: B
16. A patient on examination was found to have swollen gingiva around a crown that had been
present for several years. The papillae were particularly enlarged.
What is the most important feature of a crown that may be responsible for this?
A. Material of the Crown
B. The occlusion
C. Proximal Contour
D. Labial Contour
E. Surface finish

Answer: C

17. It is best to retain dental radiographs for how many years?


Answer:
Adults records shud be kept for: 11 years
for children: till they r 25 yrs old or 11 years, whichever is longer   ......

18. Which is the most important factor to reduce dental irradiation? (speed of film
or collimation?
Answer: Both: rectangular collimation, E or F speed films.

19. after crown placement, how many months will you review it radio graphically?
Answer: Annually.

20. What causes Amalgam expansion? What makes Amalgam black after sometime?
Amalgam Expansion:
1- Moisture contamination during mixing and condensation operation.
2- Moisture in the saliva is the potential contaminant for the amalgam.
3- Zinc containing amalgam, the presence of saliva on the amalgam during condensation probably a
possible source of expansion.

Amalgum Black:
1- corrosion.
2- Reaction of the phases.
3- Marginal leakage.

21. A 50 year old male patient has a Class III jaw relationship with an anterior open bite. It is
planned to restore his lower right second molar, which has suffered tooth wear and fracture, with an
indirect restoration. This tooth has approximately 2mm of coronal height.

What would be the most suitable approach to restore this tooth?


A. Provide an adhesively retained gold onlay
B. Provide a conventional full crown
C. Increase the vertical dimension and provide a full crown
D. Surgically crown lengthen and provide gold crown
E. Provide an adhesively retained ceramic onlay
Answer: A( refer indications for Onlay and inlay in Master dentistry 2)

22. A patient presents with a history of a post-crown having fallen out. The post-crown was
originally placed fifteen years ago and had been successful up until four months ago since when it
has come out and been recemented four times. At recementation there was no evidence of any
caries.
The patient had been a regular attender and not needed any restorative treatment for the last eight
years.
Which of the following is the most likely cause for the failure of this crown?
A. The post was to narrow
B. The post was to short.
C. The root canal treatment was failing.
D. A vertical root fracture was present.
E. There were excessive occlusal loads on the tooth.

Answer: D

23. In dental epidemiology, indices are used to measure the oral health of a population. The DMF
index is commonly used to measure the prevalence and severity of dental caries

24. You want to evaluate the effectiveness of tretracycline as an adjunct to scaling & root planning
for the treatment of chronic periodontitis.
What is the primary study design most appreciate for addressing this topic?
A. Cohort
B. Non randomised controlled trial
C. Randomised controlled trial
D. Control case study
E. Case series

Answer: C

24. Which LA is neurotoxic at 4%


Answer: Articane

25. The management of patients taking corticosteroid for long time with dental infection , abscess or
high temperature , do we need double the dose of antibiotic or double the dose of cortisone ?

Answer: Adrenal insuffiency may follow long term administration of oral corticosteroids and can
persist for years after stopping therapy. A pt with adr.insuf. can become hypotensive when under
physiological stress. Acute adr.insuf. can be prevented by administration of an increased dose of
corticosteroids prior to treatment.

26. Do we need to change the antibiotic dose for patient with rheumatoid arthritis taking 15 mg
prednisolone for 6 months or 3 months had dental abscess?
Answer: No: Antibiotic prophylaxis: amoxicillin, cefalexin or cepharidine 2g orally 30-60 min
before procedure. If unable to take oral meds give ampicillin 2g IM/IV 3O-60 min before procedure
if penicillin allergic give clindamycin 600 mg orally or IV 30-60 min before procedure.

27. What is the main cause of patient collapse with congestive heart failure in the dental clinic ?
Answer: Any surgical procedures may cause undue stress resulting in cardiac dysfunction
(workload increase of the heart) which surpasses the functional ability of the heart followed by
potential acute pulmonary edema. Pts with this condition present with extreme dyspnea,
hyperventilation, cough, hemoptysis, great difficulty in breathing, murmurs due to cardiac asthma
and cyanosis. The pt prefers the sitting position, is anxious and might feel he/she is choking and if
death is imminent.
The preventives measures are:
1. Written consent from the pt's cardiologist and consultation is desirable
2. Oral premedication: 5-10 mg diazepam 1 hour before the surg procedure
3. Small amounts of vasoconstrictors in LA with particular importance of aspiration
4. Short appointments and as painless as possible

28. Why the patient with heart failure bleed easily and how to mange such bleeding?
Answer: Q2/3 pts with heart failure are on blood thinners that help prevent clots from forming in
the blood. Blood thinners may cause to bleed easily or bruise easily. 3. Refer to cardiologist, check
INR

29. Which immunoglobin exists as both a monomer and diamer?


Answer: IgA

30. Appointment for asthmatic patient should be given in?

A morning

B Afternoon

C Evening

D Anytime

Answer: because the highest risk for asthma attack is in the morning and at night...so the better
option would be in the afternoon

30. According to the water regulations, all dental equipment must be protected by back-syphonage
and between the water ingress and drainage points there should be a

a. “Type A” gap of 20mm


b. “Type B” gap of 30mm
c. “Type C” gap of 40mm
d. “Type D” gap of 50mm.
Answer: B

31. Fire certificates are only required for buildings with 20 people working in them or if more than
ten people work on floors other than ground floors (same applies to dental practices). Fire risk
assessment should be carried every
a. 3 months
b. 6 months
c. 9 months
d. one year
Answer:D

32. Discharge in vesicullo bullous lesion - serous or suppurative?


Answer : Serous

33. Rushton bodies are commonly seen in which of the following:


1.ultraviolet rays
2.ultrasonic vibrations
3.lichen planus
4.lupus erythematosus
5. Radicular cyst

Rushton bodies: Translucent or pink staining lamellar bodies are formed by cyst lining epithelium
and indicate the odontogenic origin of a cyst.
Answer: Radicular cyst

34. If a patient has severe gag reflex which kind of sedation will you use?
Answer: Inhalation, IM, IV

35. Sedation for pregnant women?


Answer: Best to avoid sedation

Contraindications and special caution


Benzodiazepines require special precaution if used in the elderly, during pregnancy, in children, in
alcohol or drug-dependent individuals or individuals with comorbid psychiatric disorders.
Additional caution is required in critically ill patients, as accumulation of midazolam and its active
metabolites may occur.[43] Kidney or liver impairments may slow down the elimination of
midazolam leading to prolonged and enhanced effects.[5][44] Contraindications include
hypersensitivity, acute narrow angle glaucoma, shock, hypotension or head injury. Most are relative
contraindications.

Pregnancy

Midazolam, when taken during the third trimester of pregnancy, may cause severe risk to the
neonate, including benzodiazepine withdrawal syndrome with possible symptoms including
hypotonia, apnoeic spells, cyanosis, and impaired metabolic responses to cold stress. Symptoms of
hypotonia and the neonatal benzodiazepine withdrawal syndrome have been reported to persist from
hours to months after birth. [45] Other neonatal withdrawal symptoms include hyperexcitability,
tremor and gastrointestinal upset (diarrhea or vomiting). Breast feeding by mothers using
midazolam is not recommended.[46]

Neonates

Midazolam is sometimes used in neonatal intensive care unit care. When used, additional caution is
required in neonates; midazolam should not be used for longer than 72 hours due to risks of
tachyphylaxis, and the possibility of development of a benzodiazepine withdrawal syndrome, as
well as neurological complications. Bolus injections should be avoided due to the increased risk of
cardiovascular depression, as well as neurological complications.[47]

Elderly

Additional caution is required in the elderly, as they are more sensitive to the pharmacological
effects of benzodiazepines and also metabolise them more slowly, and are more prone to adverse
effects, including drowsiness, amnesia (especially anterograde amnesia), ataxia, hangover effects,
confusion and falls.[13]

36. Which immunoglobulin associated to MALT (mucosa associated lymphatic tissue?)

Answer: Predominantly IgA (IgA & IgM)

Mucosa-associated lymphoid tissue (MALT) is scattered along mucosal linings, measuring roughly
400 m2. It is the most extensive component of human lymphoid tissue. These surfaces protect the
body from an enormous quantity and variety of antigens. The tonsils, Peyer patches within the small
intestine, and the vermiform appendix are examples of mucosa-associated lymphoid tissue (MALT).

Mucosa-associated lymphoid tissue (MALT) includes gut-associated lymphoid tissue (GALT),


bronchial/tracheal-associated lymphoid tissue (BALT), nose-associated lymphoid tissue (NALT),
and vulvovaginal-associated lymphoid tissue (VALT). Additional mucosa-associated lymphoid
tissue (MALT) exists within the accessory organs of the digestive tract, predominantly the parotid
gland.

The germinal center is key to the generation of a normal immune response. The location of mucosa-
associated lymphoid tissue (MALT) is key to its function. Stimulation of B lymphocytes leads to
the production of immunoglobulin A (IgA) and IgM within the Peyer patches, preventing
adherence of bacteria and viruses to the epithelium, thus blocking entry to the subepithelial
layers of the intestine.

The direct secretion of secretory IgA onto mucosal epithelia represents the major effector
mechanism of mucosa-associated lymphoid tissue (MALT). Major accumulations of lymphoid
tissue are found in the lamina propria of the intestine. M cells in the intestinal epithelium overlying
Peyer patches allow transport of antigens to the lymphoid tissue beneath it.
The complex interplay among antigens, cells, and cytokines results in a very efficient immune
response. The efficiency of mucosa-associated lymphoid tissue (MALT) also depends on the
adequate function of IgA. Individuals with selective IgA deficiency are prone to infections along
mucosal surfaces in the respiratory, gastrointestinal, and genitourinary tracts. Adequate function of
IgA depends on the production and acquisition of a joining (J) chain. This glycoprotein is produced
by plasma cells and is important in the formation of IgA dimers and IgM pentamers. It has been
shown that in children who have recurrent tonsillitis, B lymphocytes in tonsillar crypts do not
produce the J chain. The J chain is key in permitting secretory IgA and IgM to function as the first
line of defense in mucosal epithelium.

37. What will be the answer?


Stephan's curve represents
a) change in pH of saliva with time
b) change in pH of plaque with time
c) change in pH of saliva with sugar intake.

Answer: B

Explanation: Beneficial effects of saliva in reducing plaque pH


and therefore caries
 Dr Helen Whelton, Director Oral Health Services Research Centre, University Dental School and
Hospital, Wilton. Cork, Ireland. Email: H.Whelton@ucc.ie

Dental caries is an infectious, communicable disease resulting in demineralisation and destruction


of tooth structure by acid-forming bacteria found in dental plaque, an intraoral biofilm, in the
presence of sugar1. Far from being static, dental enamel is in a constant state of change in that it
undergoes cycles of demineralisation and remineralisation. Acid produced by oral bacteria in the
glycolysis of carbohydrates is the driving force towards enamel demineralisation and caries
progression. Saliva on the other hand is the driver of repair and remineralisation. In the absence of
the protective properties of saliva, acid dissolution of the teeth would progress unchecked in
susceptible individuals with cariogenic bacteria and unfavourable diets. The fundamental aims of
strategies for caries prevention are to reduce demineralisation and/or to increase remineralisation.

The caries process is illustrated in Figure 1. Interaction between three factors over time is needed
for caries to develop; a susceptible host i.e. a tooth surface, microflora with cariogenic potential i.e.
plaque, and fermentable carbohydrate i.e. sugar.
 

Figure 1 Factors necessary for the development of dental caries

The presence of saliva is vital to the maintenance of healthy oral tissues. Its importance in
preventing caries was first illustrated by feeding a cariogenic diet to desalivated rats which
developed over four times as much caries as rats with
intact salivary glands fed the same diet. In humans, although there is a lot of variation in saliva
secretion rates, the average is 0.3ml/minute for unstimulated and 1-2ml per minute for stimulated
saliva. The rampant caries seen in some xerostomia individuals (<0.1ml saliva produced per minute,
unstimulated salivary flow rate) is indicative of the devastation of the dentition that occurs in the
absence of saliva. In healthy subjects the teeth are constantly bathed in 0.5ml of unstimulated saliva
which coats them with a film approximately 10µm thick and which moves as the muscles of the
mouth are used.

The impact of saliva in neutralising plaque pH was illustrated by Stephan 2 who measured the
changes in plaque acidity following sugar intake. He used indwelling antimony microelectrodes to
monitor changes in plaque pH in situ following a sucrose rinse. The characteristics of the Stephan
Curve are illustrated in Figure 2.

Figure 2 Changes in plaque pH over time following a sucrose rinse


 

Characteristically, the Stephan Curve shows a rapid drop in plaque pH which is followed by a slow
rise until resting pH is attained. The critical pH below which demineralisation of enamel generally
takes place is 5.5.Thus the shaded part of
the curve indicates the period of demineralisation. The time taken for these changes to occur varies
between individuals and also varies according to the nature of the challenge. The drop in pH usually
takes no more than five minutes whereas the recovery for the resting pH can take between fifteen
and forty minutes depending on the acid neutralising properties of an individual’s saliva. The initial
sharp drop depends upon the speed with which plaque microbes are able to metabolise sugar.

Large molecules like starch for example, diffuse into the plaque more slowly and take longer to be
broken down resulting in a less steep drop in pH. The rise in pH is dependent upon diffusion of acid
by-products out of the plaque and, secondly salivary bicarbonate diffusing into the plaque buffering
the acid by-products. One of the most important factors governing the overall shape of a Stephan
Curve, but particularly the pH recovery, is saliva flow rate. Saliva exerts two effects. First, it
dilutes and carries away metabolites diffusing out of the plaque. Second it supplies bicarbonate
ions which diffuse into plaque and neutralize the by-products of fermentation (organic acids) in situ.
The bicarbonate-mediated acid neutralisation effect is enhanced by the increase in salivary
bicarbonate associated with increased saliva flow which coincides with eating or chewing e.g.
chewing gum.

Source: http://betteroralhealth.info/orbit_complete/professional-area/resources/cpd/saliva-
benefits/beneficial-effects-of-saliva-in-reducing-plaque-ph-and-therefore-caries/index.htm

38. Cause of early child hood caries? (ECC)

Answer: Fruit based drink

39. Reducing cariogenicity

1. sucrose, glucose, fructose, maltose (honey)

2. Galactose, lactose

3. Complex carbohydrate (e.g. starch in rice, bread, potatoes)

Foods that stimulate salivary flow and can speed the return of plaque PH to normal

E.g. cheese, sugar free gum, slated peanuts.

40. Which antibiotic we can give to patient who is on warfarin? (not allergic to penicillin)
Answer: Penicillin

41. Which antibiotic we can give to patient who is on warfarin? and if he is also allergic to
penicillin ?

Answer: Clindamycin
Avoid Metronidazol and Erythromycin as they potentiate the action of warfarin (metro) or induce
unpredictable effects (erythromycin).

Note: Paracetamol is safer than Aspirin and other NSAIDS which should be also avoided in such
patients.

42. Drug-induced lichenoid reaction

Gold salts, beta blockers, antimalarials, ACE inhibitors, NSAID’s thiazide diuretics, furosemide,
spironolactone, and penicillamine. As well as systemic side effects of hypoglycaemic drugs such as
Metformin.

43. Which drug causes angioedema?

i.)Which drug causes lichenoid reaction,


ii.)Which drug causes dry cough?
Answer:

Phenytoin - Gingival hyperplasia


Ramipril (ACE inhibitors) – dry cough and angioedema – is type 1 hypersensitivity
Penicillin – anaphylaxis (type I hypersensitivity)
Furosemide – lichenoid reaction.

Top 10 drugs/drug classes associated with angioedema. Drug


ACE-inhibitors
bupropion
vaccines
selective serotonin reuptake inhibitors (SSRIs)
COX-II inhibitors
angiotensin II antagonists
other antidepressants
non-steroidal anti-inflammatory drugs (NSAIDs)
statins
proton pump inhibitors

Non-steroidal anti-inflammatory drugs (NSAIDs), another major cause of the condition, provoke
5
angioedema in 0.1–0.3% of patients. Facial angioedema is the most frequent presentation. Both COX-I
and COX-II inhibitors can cause angioedema.
Other drugs that can cause angioedema include aspirin, amoxicillin, and proton-pump inhibitors
(PPIs).

44. Which clotting factor is related to disseminated intravascular coagulopathy?

Answer: Factor VII, XII to XI to IX

TF binds with coagulation factors that then triggers the extrinsic pathway (via Factor VII) which
subsequently triggers the intrinsic pathway (XII to XI to IX) of coagulation.

In DIC, the processes of coagulation and fibrinolysis are dysregulated, and the result is
widespread clotting with resultant bleeding.

Under homeostatic conditions, the body is maintained in a finely tuned balance of coagulation and
fibrinolysis. The activation of the coagulation cascade yields thrombin that converts fibrinogen to
fibrin; the stable fibrin clot being the final product of hemostasis. The fibrinolytic system then
functions to break down fibrinogen and fibrin. Activation of the fibrinolytic system generates
plasmin (in the presence of thrombin), which is responsible for the lysis of fibrin clots. The
breakdown of fibrinogen and fibrin results in polypeptides called fibrin degradation products
(FDPs) or fibrin split products (FSPs). In a state of homeostasis, the presence of plasmin is critical,
as it is the central proteolytic enzyme of coagulation and is also necessary for the breakdown of
clots, or fibrinolysis.

In DIC, the processes of coagulation and fibrinolysis are dysregulated, and the result is
widespread clotting with resultant bleeding. Regardless of the triggering event of DIC, once
initiated, the pathophysiology of DIC is similar in all conditions. One critical mediator of DIC is the
release of a transmembrane glycoprotein called tissue factor (TF). TF is present on the surface of
many cell types (including endothelial cells, macrophages, and monocytes) and is not normally in
contact with the general circulation, but is exposed to the circulation after vascular damage. For
example, TF is released in response to exposure to cytokines (particularly interleukin 1), tumor
necrosis factor, and endotoxin.[5] This plays a major role in the development of DIC in septic
conditions. TF is also abundant in tissues of the lungs, brain, and placenta. This helps to explain
why DIC readily develops in patients with extensive trauma. Upon activation, TF binds with
coagulation factors that then triggers the extrinsic pathway (via Factor VII) which subsequently
triggers the intrinsic pathway (XII to XI to IX) of coagulation.

Causes

DIC can occur in the following conditions;

 Cancers of lung, pancreas, prostate and stomach, as well as acute myeloid leukemia
(particularly APML)
 Obstetric: abruptio placentae, pre-eclampsia, amniotic fluid embolism
 Massive tissue injury: Trauma, burns, extensive surgery
 Infections: Gram-negative sepsis, Neisseria meningitidis, Streptococcus pneumoniae,
malaria, histoplasmosis, aspergillosis, Rocky mountain spotted fever
 Miscellaneous: Liver disease, snake bite, giant hemangioma, shock, heat stroke, vasculitis,
aortic aneurysm, Serotonin syndrome[9]
 Viral: Arenaviruses causing Argentine hemorrhagic fever or Bolivian Hemorrhagic Fever

Diagnosis
Diagnosis is usually suggested by following conditions:[8]

 Severe cases with hemorrhage: The PT and APTT are usually very prolonged and the
fibrinogen level markedly reduced. High levels of fibrin degradation products, including D-
dimer, are found owing to the intense fibrinolytic activity stimulated by the presence of
fibrin in the circulation. There is severe thrombocytopenia. The blood film may show
fragmented red blood cells (schistocytes).
 Mild cases without bleeding: There is increased synthesis of coagulation factors and
platelets. PT, APTT, and platelet counts are normal. fibrin degradation products are raised.

Definitive diagnosis depends on the result of:

 Thrombocytopenia
 Prolongation of prothrombin time and activated partial thromboplastin time
 A low fibrinogen concentration
 Increased levels of fibrin degradation products

Conditi Prothrombin time Partia Bleeding time Platelet count


on l
throm
bopla
stin
time
Vitami prolonged prolo http://en.wikipedia.org/ http://en.wikipedia.org/
nK nged wiki/ wiki/
deficie Von_Willebrand_disease Von_Willebrand_disease
ncy or unaffected unaffected
warfari
n
Dissem prolonged prolo prolonged decreased
inated nged
intrav
ascula
r
coagul
ation
Von http://en.wikipedia.org/ prolo prolonged unaffected
Willebr wiki/ nged
and Von_Willebrand_disease
disease unaffected
Haemo http://en.wikipedia.org/ decre unaffected unaffected
philia wiki/ ased
Von_Willebrand_disease
unaffected
Aspirin http://en.wikipedia.org/ unaff prolonged decreased
wiki/ ected
Von_Willebrand_disease
unaffected
Throm http://en.wikipedia.org/ unaff prolonged decreased
bocyto wiki/ ected
penia Von_Willebrand_disease
unaffected
Early decreased unaff unaffected unaffected
Liver ected
failure

45. Which one of these is most benign?

a) kaposis sarcoma

b) adenolymphoma

c) burkitts lymphoma

Answer: adenolymphoma

A benign glandular tumor usually arising in the parotid gland and composed of two rows of
eosinophilic epithelial cells with a lymphoid stroma. Also called papillary cyst adenoma
lymphomatosum, Warthin's tumor.

46. which one of these tumors most likely to occur bilaterally


adenolymphoma 10% occur bilaterally.
a) Pleomorphic adenoma - 75% of parotid tumors, its mixed tumour ( fibrous, myxoid,
cartilaginous )***
Commom intraoral site – junction of hard and soft palate usually feels rubbery and lobulated on
palpation.

b) mucoepidermoid carcinoma- solid and more aggressive, can be invasive and occasionally
metastsise. 3-9%****

c) adenocystic carcinoma - cribriform or Swiss cheese pattern, infiltrate along nerve sheaths
Cause multiple cranial nerve lesions – especially lingual, facial or hypoglossal ***

d) Acinic cell carcinoma 1%


46. A curette may be inserted to the level of the attached gingiva with minimal trauma to the tissues
because of

A. Has a round base


B. Is easy to sharpen
C. Has rounded cutting edges
D. Provides good tactile sensitivity
E. Has two cutting edges

Answer: C

47. What type of hypersensitivity reaction causes amalgam restoration?

Answer: Type IV

48. Kiwi causes anaphylaxis & urticaria,


Answer: type I hypersensitivity.

48. As far as localised alveolar osteitis is concerned; which one of the following
is true

2. A. The incidence in the mandible and maxilla is similar


B. The prophylactic prescription of antibiotics prior to extraction reduces the
incidence.
C. Excessive fibrinolysis is the likely aetiology
D. Purulent exudate must be seen for a diagnosis and irrigation is mandatory
E. Zinc oxide eugenol and alvogyl dressing promote a rapid bone growth

Answer: C
Explanation: loss of cot was traditionally ascribed due to bacterial proteolytic enzymes. However,
it appears frequently due to excessive local fibrinolytic activity. The alveolar bone and other oral
tissues have a high content of fibrinolysin activators (plasmin) which are released when the
bone is traumatized.

** Oestrogen component of oral contraceptives enhances the serum fibrinolytic activity.


Alveolar osteotitis: affects females, few days after extraction may be delayed for week or
more. Pain is deep seated, severe and aching or throbbing.
Ethiology:
 In healthy person, it only affects lower molar region where the bone is more dense and less
vascular than elsewhere.
 Its an expected complication of extractions in Paget’s disease and after radiotherapy where
endarteritis causes ischemia of the bone.
 LA
49. How do you prepare floor of pulp chamber in molars

A. Swab and dry with cotton wool and excavate


B. Use round bur to flatten the floor
C. Under cut walls
D. Use flat end fissure bur to make it leveled

Answer: A

50. Which immunoglobin is the first to be found when the child is born?
Answer: IgM( first to be produced by the neonate and produced in primary immune response, 5-
10% inserum)
IgG crosses placneta and protects fetus, produced in secondary immune response. Most
abundant immunoglobulin 80% in serum.
IgA major secretry immunoglobulin (10 – 15%)

51. Distance of maxillary sinus from premolars and molars


Answer:
Canine 6.9 mm
1st PM – 3.8
2nd PM – 1.9
1st M – 1.8
2nd M – 1.7
3rd M – 2.8
The bony lamella separating the maxillary sinus and the teeth decreases in thickness from the upper
canine (6.9 mm) to the 2nd molar (1.7 mm) and further increases to the 3rd molar (2.8 mm).

52. Which of the following anomalies occurs during the initiation and proliferation stages of tooth
development

A. Amelogenesis imperfecta - histodifferentiation


B. Dentinogenesis imperfecta- histodifferentiation
C. Enamel hypoplasia – apposition stage
D. Oligodontia – initiation stage ( absence of single or multiple teeth)
E. Ankylosis
Hypocalcified/hypomineralised – normal organic matrix but defective mineralization.

53. How to disinfect a bridge before sending to lab?

54. Best treatment planning- to replace missing lateral incisor

55. 15yr old boy, 26year old with missing lateral, with good periodontium, bone levels, good oral
hygiene, absence of Dental Caries ?
Answer
15 yrs- Resin bonded bridge?
26years- Implant?

56.Treatment for pegshaped laterals? With Hybrid composite or laminate veneer the best option?

57.Type of crown placed after Nayyar core tec?

58.Cement used for Temporary cementaion? Is it Zincplycarboxylate cement or Zinc oxide eugnol
or both?
Answer: Zinc oxide eugnol ( temp bond)

59. How many mg of lidocaine in 3 cartridges of 2.2? please explain


Answer:
A. 2.2 ml of cartridge has 44mg of lidocaine.
132mg of LA in 3 cartridges

The max dose of L.A. is 4.4mg/kg body weight.


eg. weight of a young male 70 kg, hence max dose of LA will be 4.4 x 70 = 308mg.

2% Lignocaine comes to 20mg Lignocaine per ml.


1 cartridge = 1.8ml = 1.8 x 20 = 36mg of lignocaine in one cartridge.

Hence now 308 divided by 36 = 8.55


which means about 8 cartridges of L.A. are safe in this patient (wt. 70kg)

60. What is the most toxic LA?


Answer:
Cardiotoxic- bupivacaine also ( long.acting)

61. LA for a patient with congenital heart disease in a dental emergency


Answer:
for cardiac arrhythmias- Lignocaine
causes methhaemoglobinaemia - prilocine

62cells in acute and chronic infection


Answer:
Acute- PMN,
chronic- Lympocytes, plasma cells, macrophages

63. Cells in granulomatosis infection


Answer .Glaucomatous infection- epitheloid cells

64. What is the concentration of lignocaine in a topical anesthetic?


Answer: Intra oral 4% is available as an ointment or gel.
Spray formulations – 10% lidocaine is available for introral use.
Skin: lidocaine 2.5%, prilocine 2.5% or 4% tetracaine

Lidocaine is an effective LA and consequently the most commonly used in dentistry in the UK. It is
available in dental cartridges as a plain 2% solution or with adrenaline (epinephrine) added in a
concentration of 1:80 000.

Prilocaine
Prilocaine is available as a 4% plain solution or as a 3% solution with 0.02 lU/ml felypressin. The
latter is the usual alternative to lidocaine with adrenaline (epinephrine) in the UK. If a
vasoconstrictor must be avoided, then plain 4% prilocaine is more effective than plain 2% lidocaine.

2) What is the depth of action of a topical anesthetic?

3) Which is the most toxic anesthetic agent?

Most toxic – Bupivacine


Less toxic – prilocine compared to lidocaine

65. Bur used to remove enamel?


2. Bur used to remove dentine?
3. Treatment options for root canal treated upper molar wt buccal root canal failure and periapical
infection?
4. Percentage of children getting cavities after application of sealants?
5. Best restoration material for class 5 in sjogrens? - GIC
6. Cavity in post teeth in parkinsonism patient,restorative material of choice? - Patients with
Parkinson's disease are usually given anti-cholinergics, which causes dry mouth, hence increased risk of
caries.. GIC

66. What antibiotics and pain reliever would you prescribe for a patient who has prosthetic heart
valve and is on Walfarin anticoagulant after having a surgical extraction of an infected tooth?
Ans: Penicillin and Paracetamol.

How should postoperative pain control be managed? 

Patients should follow the advice of their anticoagulant clinic with regard to the choice of analgesia
for short-term mild to moderate pain. Generally paracetamol is considered the safest simple
analgesic for patients taking warfarin and it may be taken in normal doses if pain control is needed
and no contraindication exists. Patients should be advised not to take aspirin, aspirin containing
compound analgesic preparations or non-steroidal anti-inflammatory drugs (NSAIDs) e.g.
ibuprofen, which are considered less safe than Paracetamol in patients taking warfarin.

If analgesia is to be prescribed additional options include;


• Dihydrocodeine – an opioid analgesic with similar analgesic efficacy to codeine that can
be prescribed on an NHS prescription. Its use should be considered second line and only
when other drugs are unsuitable. It is effective for mild to moderate pain but has no anti-
inflammatory activity and is of limited value in pain of dental origin.

Are there any drug interactions that are relevant to this patient group undergoing dental surgical
procedures?

Amoxicillin - There are anecdotal reports that amoxicillin interacts with warfarin causing increased
INR and/or bleeding but documented cases of an interaction are relatively rare considering how
frequently the drug is used; the broad picture is that no clinically relevant interaction normally
occurs with amoxicillin and most penicillin’s. A single 3 gram dose given for endocarditis
prophylaxis has not been shown to produce a clinically relevant interaction. Prophylactic antibiotics
do not appear to affect the bleeding risk postoperatively.

Patients taking warfarin requiring a course of amoxicillin should be advised to be vigilant for any
signs of increased bleeding and concurrent use should be monitored so that the very occasional and
unpredictable cases INR increase or decrease can be identified.

Clindamycin - Clindamycin does not interact with warfarin when given as a single dose for
endocarditis prophylaxis. Prophylactic antibiotics do not appear to affect the bleeding risk
postoperatively.

Metronidazol – CAUTION: Metronidazol interacts with warfarin and should be avoided wherever
possible. If it cannot be avoided the warfarin dose may need to be reduced by a third to a half by the
GP or anticoagulant clinic. The patient must seek advice from the person managing their
anticoagulation before taking Metronidazol.

Erythromycin - Erythromycin interacts with warfarin unpredictably and only affects certain
individuals. Most are unlikely to develop a clinically important interaction. Patients should be
advised to be vigilant for any signs of increased bleeding, concurrent use should be monitored
especially in the elderly.

Paracetamol - The anticoagulant effect of warfarin is normally not affected, or only increased by a
small amount, by occasional doses of paracetamol (no more than about 2.5 to 3 g weekly).
Paracetamol is considered to be safer than aspirin as an analgesic in patients taking warfarin and is
the analgesic advised by anticoagulant clinics and the patient held ‘Oral Anticoagulant Therapy’
booklet. The anticoagulant effect of warfarin may be enhanced by prolonged regular use of
paracetamol.

Aspirin - AVOID use as an analgesic and anti-inflammatory agent. Concurrent aspirin increases the
likelihood of bleeding by 3-5 times, increases the bleeding time and may damage the stomach
lining. The interaction is well documented and clinically important.

Non-Steroidal Anti-Inflammatory Drugs (NSAIDs)


e.g. ibuprofen, diclofenac. – AVOID: Care should be taken when using NSAIDs because, to a
greater or lesser extent, they irritate the stomach lining, which can result in gastrointestinal
bleeding, which will be more severe in anticoagulated patients. Although no interaction usually
occurs with normal doses of ibuprofen and probably diclofenac, isolated cases of raised INRs have
been described. Some NSAIDs have effects on platelet activity, which can affect bleeding times.
 
http://www.dundee.ac.uk/tuith/Static/info/warfarin.pdfhttp://www.dundee.ac.uk/tuith/Static/info/
warfarin.pdf

67. What do we do with used files? are they disposed of in a sharps container or

are they sterilized ?


Answer: According to new guidelines you reuse it for the same patient once only (from
start to finish of rct) and have to dispose in sharps bin.

68. Which tooth if extracted can spread infection to cavernous sinus?

Answer: Mid facial infection spread infections to cavernous sinus

69. Patient, a known diabetic collapses in your dental chair, what wud is your first line of treatment?
Answer: If patient is conscious give glucose orally or if unconscious give 1mg glucagon IM.

70. Which measurement is taken with a single record block in the mouth??
Answer: Resting Vertical Dimension: taken with only the lower denture. Willis gauge used for
measurement.

71. Uses of face bow?


Answer: a device used in dentistry to record the positional relations of the maxillary arch to the
temporomandibular joints and to orient dental casts in this same relationship to the opening axis of
the articulator.

Or
Used to transfer the relationship of Maxilla to the intercondylar axis of the patient to the Articulator
and the hinge axis of articulator.

72. Cantilever Bridge


Answer: Suitable for Anterior teeth only

A Cantilever Bridge is a fixed bridge that is attached to adjacent teeth on one end only. They are
used in cases where a bridge can only be anchored to a tooth on one side of the gap. Thus,
Cantilever Bridges are best suited to areas of your mouth that are less prone to stress, such as the
front teeth. This dental procedure involves anchoring the false tooth to one side over one or more
natural and adjacent teeth. As a result, they are best suited and ideal for situations where a
traditional dental bridge or a dental implant isn’t an option.
73. Role of NaCl in saliva?
Answer: Preserve salt taste

2: role of Hco3 in saliva?


Answer: Acid-base balance

3: what makes saliva isotonic/hypotonic?


Answer: reduced concentration of HCo3- makes saliva hypotonic and has little buffering capacity.

Isotonic: Rich Na+ and HCO3- makes saliva isotonic produced by acinar cells (Primary saliva)

Final Saliva is less hypotonic and with greater acid buffering capacity

74. Saliva
Noraml resting saliva flow 0.1-0.6ml/min
Less than 0.1 cause xerostomia

75. A patient on examination was found to have swollen gingiva around a crown that had been
present for several years. The papillae were particularly enlarged.
What is the most important feature of a crown that may be responsible for this?
A. Material of the Crown
B. The occlusion
C. Proximal Contour
D. Labial Contour
E. Surface finish

Answer: C
Explanation: The keyword in the question is enlargement of the papilla. This enlargement is the
result of lack of space (under preparation) for the crown's material (MCC or GSC or Full porcelain)
and resulted in over contoured crown which in turn irritated the papilla and invaded its space
causing it to hypertrophy. Because the papilla are anatomically located proximal (mesial or distal)
to teeth, the best answer would then be proximal contour

Hypertrophy: increase in size of cells


hyperplasia: increase in number of cells (neoplasm=cancer)

76. Dental fee excemption


You do not have to pay for NHS dental treatment if, when the treatment starts, you are:
- Under 18
-Under 19 and receiving full-time education
-Pregnant or a mother who has had a baby in the previous 12 months
-Staying in an NHS hospital and your treatment is carried out by the hospital
dentist.
-An NHS hospital dental service outpatient (however, you may have to pay for your
dentures or bridges)
-Included in an award of:
- Income support
- Income-related Employment and Support Allowance
- Pension Credit guarantee credit
-You are named on a valid NHS tax credit exemption certificate,or you are
entitled to an NHS tax credit exemption certificate
-You are named on a valid HC2 certificate.
Ref :
- Pamphlet: NHS dental services in England
- www.dh.gov.uk/dentistry

77. A 55 year old female patient is missing her upper right second premolar and upper right first
molar and also is missing the upper left second molar. The upper right second molar is functional
and has an amalgam restoration (MOD and buccal wall) that requires replacing. The patient has no
functional or aesthetic concerns.
What would be the treatment of choice in this situation?
A. Provide an upper removable partial denture
B. Replace the amalgam in the upper right 7 only
C. Provide a full coverage crown in the upper right 7
D. Provide a fixed bridge in the upper right quadrant
E. Provide a full coverage crown in the upper right 7 with guide planes and occlusal rests.

Answer: B

78. A new filling material has been developed by the Dental School. After publication of laboratory
results, the researchers conducted a randomized clinical trial in general dental practice where
patients requiring one filling were randomly allocated either to old or new filling material group.
After five year follow up, the mean survival time between two materials was compared, and
P-value of 0.125 was reported.

Which answer is correct?


A. There are no important differences in the properties of the materials
B. There is no difference in restoration mean survival time between the two materials
C. The difference between two materials is not clinically important
D. The difference between the materials is very small
E. The new material is useless

Answer : B

Explanation: traditionally, one rejects the null hypothesis if the p-value is smaller than or equal to
the significance level,[2] often represented by the Greek letter α (alpha). If the level is 0.05, then
results that are only 5% likely or less, given that the null hypothesis is true, are deemed
extraordinary.
When we ask whether a given coin is fair, often we are interested in the deviation of our result from
the equality of numbers of heads and tails. In such a case, the deviation can be in either direction,
favoring either heads or tails. Thus, in this example of 14 heads and 6 tails, we may want to
calculate the probability of getting a result deviating by at least 4 from parity (two-sided test). This
is the probability of getting at least 14 heads or at least 14 tails. As the binomial distribution is
symmetrical for a fair coin, the two-sided p-value is simply twice the above calculated single-sided
p-value; i.e., the two-sided p-value is 0.115.

In the above example we thus have:

 null hypothesis (H0): fair coin;


 observation O: 14 heads out of 20 flips; and
 p-value of observation O given H0 = Prob(≥ 14 heads or ≥ 14 tails) = 0.115.

The calculated p-value exceeds 0.05, so the observation is consistent with the null hypothesis —
that the observed result of 14 heads out of 20 flips can be ascribed to chance alone — as it falls
within the range of what would happen 95% of the time were this in fact the case. In our example,
we fail to reject the null hypothesis at the 5% level. Although the coin did not fall evenly, the
deviation from expected outcome is small enough to be reported as being "not statistically
significant at the 5% level".

However, had a single extra head been obtained, the resulting p-value (two-tailed) would
be 0.0414 (4.14%). This time the null hypothesis – that the observed result of 15 heads out of 20
flips can be ascribed to chance alone – is rejected when using a 5% cut-off. Such a finding would be
described as being "statistically significant at the 5% level".

79. A 23 year old male presents to your surgery. He lost his upper lateral incisors some 10 years ago
in a swimming pool accident. Since then he has been wearing a `spoon' denture which he now feels
in aesthetically unacceptable. He has sought an opinion on dental implants but has been told that he
would need bone grafting for this to be successful and he is not prepared to undergo this.

His dentition is excellent with no restorations and a Class I occlusion. He wants some advice on
what the best treatment might be.
Which option would you put first on your list of possibilities?
A. Two fixed resin bonded bridges using the central and canine teeth
B. Two cantilever resin bonded bridges from the central incisors and canines.
C. Two conventional fixed bridges from the canine
D. Conventional cantilever bridges from the canines
E. Cobalt chrome partial denture

Answer: B

80. Universal curette and Gracey curette

Rake angle 90 and 70 degree.


 A universal curette has a blade that is perpendicular to its terminal shank. This orientation
allows the blade to be used against either the mesial or distal surface of a tooth. Because this
type of periodontal curette was developed at the Columbia University College of Dental
Medicine, it is also known as a Columbia curette.
 The Gracey curette, invented by Dr Clayton Gracey with the help of Hugo Friedman of
Hu-Friedy Manufacturing company in the early 1940's, has a blade that is laterally offset by
70 degrees relative to the shank. Consequently, a Gracey curette has a lower cutting edge
and an upper non-cutting edge. Because only one side of each blade can cut, Gracey curettes
are site-specific, and a posterior instrument used to clean mesial surfaces of teeth won't work
on distal surfaces , and vice versa. "Gracey" Christina Patrick blades used for mesial
surfaces of anterior teeth from the facial are only suitable for the distal surfaces of the same
teeth when access is performed from the lingual.Gracey Curettes 1/2 ,3/4 ,5/6 are used on
the anterior sextants of teeth.7/8 and 9/10 are used on the buccal and lingual portions of
posterior teeth.11/12 and 15/16 are used on the mesial portions of posterior teeth.13/14 and
17/18 are used on the distal portions of posterior teeth.

81. How to disinfect a bridge before sending to lab?

Answer: ultra sonic bath

82. Gingival enlargement due to phenytoin?


1. Affects males more then females
2. More in edentulous areas
3. Dose dependent
4. Occurs within 4-12 months after beginning of therapy

Answer: A
No sexual predilection exists for drug-induced gingival overgrowth, although in one study, males
were 3 times more likely than females to develop gingival overgrowth with calcium antagonists.
 Phenytoin has been shown to induce gingival overgrowth by its interaction with a
subpopulation of sensitive fibroblasts.
 Cyclosporine has been suggested to affect the metabolic function of fibroblast (eg, collagen
synthesis, breakdown), whereas nifedipine, which potentiates the effect of cyclosporine,
reduces protein synthesis of fibroblasts

82. Which antibiotic causes diarrhea and abdominal cramps at a high dose?
Answer: Mainly: clindamycin causes abdominal cramps and diarrhea.  (Antibiotic induced
colitis)ref- churchil book ( page no:497)

Others: less commonly erythromycin, penicillin, cephalosporin ( scully 5 th edi, page no:113)

83. which anion is present in bone?


Anion – phosphate and hydroxyl
Cation - calcium
2 significance of surface markers in hep b? – indicates acut or chronic infection, immunity etc.
3 which virus is stable outside the body? – hep A
4 how to treat a spillage of less then 30 ml of blood? – 10,000ppm of sodium hypochlorite.
5 which drug causes a fixed ulcer? – nicodril
6 who can give dental education?

84. GDC regulations


The GDC is an independent healthcare regulator established under the Dentists Act 1956 (since
replaced by the Dentists Act 1984, as amended). Its purpose is to protect the public by regulating
dentists and dental care professionals in the United Kingdom. In particular, the GDC:
• Registers qualified professionals (dentists, dental hygienists, dental therapists, dental nurses,
orthodontic therapists, dental technicians and clinical dental technicians).
• Sets standards of dental practice and conduct.
• Assures the quality of dental education.
• Ensures professionals keep up-to-date. (CPD)
• Helps patients with complaints about a dentist or a dental care professional.
• Works to strengthen patient protection.

85. How often should routine radiographs be taken?


Posterior bitewing radiographs
High caries risk: 6 monthly interval
Moderate risk: Annually
Low risk: primary dentition: 12-18 month
Permanent dentition: 2 years

86. Which x-ray for implant?


Conventional tomography or CT?

Answer: CT is best for implant.

87. What type of cells are seen in the early and later stages of periodontitis?
Answer:
Initial lesion: polymorphs
Early lesion: lymphocytes and polymorphs
Established: lymphocytes and plasma cells
Advanced: plasma cells dominate

88. The Isthmus diameter in cavity prep?


Answer: Dimensions of balance between adequate depth without risking pulpal exposure (1.5-
2mm) and adequate width without weakening the cusps(1/3-1/2distance between the cusps) in pedo
pg 84 pink book.

However in class II adults the width of the isthmus should not overcut (ideally 1/4 to1/3 intercuspal
width) pg 230 pink book.

89. Which area is more prone to develop cancer?


1 floor of the mouth is the single most common site followed by lingual sulcus and ventral surface
of tongue ( accounting for 75% of Cancer seen in European and America)

90. What is the pressure of BPE probe?


Answer: 25 grms

91. The management of patients taking corticosteroid for long time with dental infection, abscess or
high temperature, do we need double the dose of antibiotic or double the dose of cortisone ?

Answer; Adrenal insuffiency may follow long term administration of oral corticosteroids and can
persist for years after stopping therapy. A pt with adr.insuf. can become hypotensive when under
physiological stress. Acute adr.insuf. can be prevented by administration of an increased dose of
corticosteroids prior to treatment.

92. Do we need to change the antibiotic dose for patient with rheumatoid arthritis taking 15 mg
prednisolone for 6 months or 3 months had dental abscess?

Answer: Antiobiotic prophylaxis- is not recommended for prevention of infective endocarditis in


adults or children undergoing dental procedures - ( scully ) except PATIENTS ON PROSTHETIC
JOINTS undergoing ( pg 396 Scully -6th edition )

1. dental at risk procedures - ( tooth removal, flaps, perio surgery - subgingival scaling, root
planing, intraligament inj, reimplant avulsed teeth, endo beyond root apex, ortho banding )

2. Joint at risk- placed within 2 yrs, h/0 0f infection, joints in hemophiliacs, type1 diabetics,
rheumatoid. a, under immunosuppressive therapy.

3. Antibiotic prophylaxis- 1-1.5hours AMOXICILLIN 2 G


CLINDAMYCIN 600 mg
CEFALEXIN 2 G
AZITHROMYCIN 500 mg
CLARITHROMYCIN 500 mg

93. Why the patient with heart failure bleed easily?. How to mange such bleeding?
Answer: pts with heart failure are on blood thinners that help prevent clots forming in the blood.
Blood thinners may cause to bleed easily or bruise easily. 3. Refer to cardiologist, check INR.

94. What is the main cause of collapsing a patient with congestive heart failure in the dental clinic?
Answer: any surgical procedures may cause undue stress resulting in cardiac dysfunction
(workload increase of the heart) which surpasses the functional ability of the heart followed by
potential acute pulmonary edema. Pts with this condition present with extreme dyspnea,
hyperventilation, cough, hemoptysis, great difficulty in breathing, murmurs due to cardiac asthma
and cyanosis. The pt prefers the sitting position, is anxious and might feel he/she is choking and if
death is imminent.
The preventives measures are: 1. Written consent from the pt's cardiologist and consultation is
desirable 2. Oral premedication: 5-10 mg diazepam 1 hour before the surg procedure 3. Small
amounts of vasoconstrictors in LA with particular importance of aspiration 4. Short appointments
and as painless as possible

95. Which immunoglobin exists as both a monomer and diamer?


Answer: IgA

96. about k files all are true except


a has more number of flutes then reamer
b is more flexible then reamer
c used to machine the dentin
d made up of triangular/square blank

Answer: b

k-files

1. have more flutes than reamers.


2.made up of triangular or sqare blank ( Reamers - triangle)
3. USed to machine dentine ( like reamers )

k-flex files are more flexible.

99. Fluoride in blood is carried by - calcium ions


Answer: The mechanism of toxicity involves the combination of the fluoride anion with the
calcium ions in the blood to form insoluble calcium fluoride, resulting in hypocalcaemia

100. What is the extracranial cause of nerve palsy?


a. cerebral malignancy (Intracranial palsy)
b. stroke (Intracranial palsy)
c. MS (Intracranial palsy)
d. Ramsay hunt syndrome (Intracranial palsy)
e. Melkersson-Rosenthal syndrome

Answer: E

101. A young girl complains of swelling of cheeks both sides. On doing various tests the salivary
glands were found normal. What may be the reason?

Answer: Cherubism

102. A 43year old patient is missing on the upper right first premolar and molar. He has good oral
hygiene and requests a fixed replacement for these teeth. The other teeth on the same side are all
moderately restored with MOD amalgam restorations and are vital, except the canine, which has a
very large restoration and is root-filled. He has group function. Radiographs show a large sinus
cavity and no peri-apical pathology.
What would be the restoration of choice for replacement of the missing teeth?
A. Implant supported crowns
B. A conventional fixed bridge using the 7 and 5 as abutments
C. Two conventional cantilevered bridges, using the 7 and 3 as abutments
D. A resin-bonded bridge, using the 7 and 5 as abutments
E. A conventional fixed-moveable bridge using the 7 and 5 as abutments

Answer: B
Explnation: a fixed bridge with 7 and 5 as abutments.

 Cantilever bridges are not preferred because they put a heavy load on the retainer and distal
cantilever bridges are rarely indicated due to high occlusal loads.
Resin bonded bridges are generally for the anterior teeth and they again cant take up high
loads.
Fixed movable bridges..in this case the cantilever effect of the non rigid component will
put extra load on the major retainer and these are also not indicated for posterior teeth with
heavy loading .
Implants are best for single tooth replacement.

103. An 80 year old patient presents with an ulcer in the floor of the mouth. This has been present
for several months and has not responded to conventional treatment. An incisional biopsy is taken.
Which of the following histological changes in the epithelium confirm a diagnosis of squamous cell
carcinoma?
A. Hyperkeratosis
B. Acanthosis
C. Dysplasia
D. Invasion
E. Discontinuous epithelium

Answer: D

104. Cigarette smoking is considered to be the most important factor next to microbial plaque in
periodontal disease progression.
Which of the following is the most important factor in the disease progression in smokers?
A. Smokers have drier mouths than non-smokers
B. Smokers have poorer oral hygiene than non-smokers.
C. Nicotine will impair the chemotactic and phagocytic properties of PMNs.
D. The gingival blood flow is reduced in smokers.
E. Smokers alter the oral environment encouraging the growth of anaerobic bacteria.
 Answer: D
 http://www.tobaccoinduceddiseases.com/content/pdf/1617-9625-8-4.pdf

105. A 35 year old male patient who admits to grinding his teeth at night has a number of wedge-
shaped cervical (Class V) lesions on his upper premolar teeth. These are causing some sensitivity
and are approximately 3mm deep.
What is the correct management option?
A. Provide tooth brushing instruction and fluoride
B. Restore the lesions with compomer
C. Restore the lesions with micro-filled composite
D. Restore the lesions with a hybrid composite
 Answer: D

106 Cell which not present in healthy periodontium? 


Answer: Cementoclasts are found only in pathological conditions such as application of excessive
orthodontic forces, roots of deciduous teeth undergoing resorption prior to exfoliation.
Blue book pg 103

107. A patient gives a history of rheumatic fever. Which of the following procedures require
prophylactic antibiotic cover.
a. scale and polish
b. extraction of tooth
c. inferior dental nerve block
d. impression for a new lower complete denture
e. placing a class 1 amalgam restoration
Answer: none

108. Best method to diagnose below caries


1) caries half away through enamel on proximal surface - bitewing
2) caries halfway through enamel on occlusal surface - straight probe
3) caries half way through dentin not cavitated – bitewing
4) stained fissure – straight probe
5) hidden occlusal caries – wet tooth

Option: traslumination, bitewing, dry tooth, wet tooth, briault prob, Williams’s prob, straight sharp
prob

Answer: Bitewing

109. What is required to increase efficacy and reduce fatigue of universal curettes?

Answer: Sharpness of the instrument, Chair position, and instrument Grasp and finger rest all
are required to increase efficiency and reduce fatigue of universal curettes.

110. Ideal angle required for sub gingival curettage from universal curettes?

Answer: Ideal angle required for sub gingival curettage from universal curette is 45 degrees.
If 0 degrees then embedding the calculus to root surface.

If 90 degrees Laceration of soft issues will occur.


111. Suppuration is mainly the result of the combined action of four factors which of the following
is not one of them?

1. necrosis

2. presence of lymphocytes

3. collection of neutrophils

4. accumulation of tissue fluid

5. autolysis by proteolytic enzymes

Answer: is 2. Presence of lymphocytes

112. What make scaling easier and less exhausting?


A. length of instrument

B. breadth

C. grip
Answer: C

2) Same question about tiredness?

A. diameter of curette

B length

C thickness

Answer: C

113. Which immunoglobin will increase in gingival inflammation?


Answer: IgG

114. 8 year old, high caries rate prescribe fluoride content

500ppm, 800ppm, 1000ppm, 1250 ppm.

Answer: 1250. (1350 – 1400ppm)


Risks Adult >16 years 6 -16 years 3-6 years 0-3 years

High 5000 ppm 2800ppm 1500ppm 1000 ppm


Medium 2800 ppm 1500 ppm 1500 ppm 500 ppm
Low 1500 ppm 1500 ppm 1000-1500 ppm 500 ppm

115. What is gingival index?


Score 0- normal gingiva
score 1- Mild inflammation, slight change in colour, slight oedema, No bleeding
on Probing
Score2- Moderate inflammation, redness, oedema, Bleeding on probing
score3- Severe inflammation, marked redness, oedema, ulceration. spontaneous
bleeding

116. What would over trituration of silver and mercury cause and what would under
trituration cause?
Answer: over trituration causes: hard and hot amalgam

under trituration causes: grainy amalgam.

117. A patient on examination was found to have swollen gingiva around a crown that had been
present for several years. The papillae were particularly enlarged.

What is the most important feature of a crown that may be responsible for this?
A. Material of the Crown
B. The occlusion
C. Proximal Contour
D. Labial Contour
E. Surface finish

Answer: C

118. What causes Amalgam expansion? What makes Amalgam black after sometime?
Answer:
Amalgam Expansion:
1- Moisture contamination during mixing and condensation operation.
2- Moisture in the saliva is the potential contaminant for the amalgum.
3- Zinc containing amalgum , the presence of saliva on the amalgum during
condensation probably a posible source of expansion.

Amalgum Black:
1- corrosion.
2- reaction of the phases.
3- marginal leakage.

119. A patient presents with a history of a post-crown having fallen out. The post-crown was
originally placed fifteen years ago and had been successful up until four months ago since when it
has come out and been recemented four times. At recementation there was no evidence of any
caries. The patient had been a regular attendee and not needed any restorative treatment for the last
eight years.
Which of the following is the most likely cause for the failure of this crown?
A. The post was to narrow
B. The post was to short.
C. The root canal treatment was failing.
D. A vertical root fracture was present.
E. There were excessive occlusal loads on the tooth.
Answer: D

Explanation: Post crown was fixed 15 years ago & was fine till 4 months ago which means there is
some drastic change after such a long time which is not gradual. Change in occlusal load is
normally a gradual process & occurs especially after some restorations or extraction are done or due
to some perio issues. Pt had no treatment in mouth for last 8 years which indicates very good
hygiene. The only reason in this case is a fracture in root & is also supported by the fact that post
crown needed recementation 4 times in last 4 months coz root is not supporting the post due to
fracture.

120. A 50 year old male patient has a Class III jaw relationship with an anterior open bite. It is
planned to restore his lower right second molar, which has suffered tooth wear and fracture, with an
indirect restoration. This tooth has approximately 2mm of coronal height.
What would be the most suitable approach to restore this tooth?
A. Provide an adhesively retained gold onlay
B. Provide a conventional full crown
C. Increase the vertical dimension and provide a full crown
D. Surgically crown lengthen and provide a gold crown
E. Provide an adhesively retained ceramic onlay

Answer: A

121. In dental epidemiology,


Indices are used to measure the oral health of a population. The DMF index is commonly used to
measure the prevalence and severity of dental caries.

122. You want to evaluate the effectiveness of tretracycline as an adjunct to scaling & root planning
for the treatment of chronic periodontitis
What is the primary study design most appreciate for addressing this topic?
A. Cohort
B. Non randomised controlled trial
C. Randomised controlled trial
D. Control case study
E. Case series

Answer: C

123. Which LA is neurotoxic at 4%


Answer: 4% is articane is neurotoxic
124. .Which LA will u use in a child to do pulpotomy?
Answer: 2% LIGNOCAINE

125. Cyst in which white paste is seen?


Answer: OKC

126. What is the amount of gas in lungs at the end of the tidal volume?
Answer: 1000ml is residual vol

127. Anesthetic for patient with congenital heart disease in an emergency appointment?
Answer: 2%LIGNOCAINE WITHOUT 1:80 000 ADR

128. Pt. with dentures that don't fit,


Answer: macroglossia+ space in lower central incisors?

129. Minimum percentage of leucocytes for an extraction?


Answer: 50,000

130. Cement used for cantilever with little preparation?


Answer: GIC LUTING

131. % of fissure sealants that fail and get carious?


Answer: RETENTION OF F/S AT 48 MTHS= 52% SO FAILURE = 48% (Nunn et al 2000,
cochrane review, beauchamp 2008)

132. What is the % of transmission of these viruses?

Is it HBV 30%

HCV 3%

HIV 0.3%

which is the most stable virus outside the body?

Hepatitis A Virus
The hepatitis A virus, or HAV, is relatively hardy.In good conditions, it can survive outside the
body for months. HAV can survive certain acids and some heat. For a period of time and under
certain conditions, HAV can survive in sea water, dried feces and live oysters.

 Hepatitis B Virus
The hepatitis B virus, or HBV, can still be infectious for up to a week outside the body.

How long does the


hepatitis C virus survive outside the body?
The hepatitis C virus can survive outside the body at room temperature, on environmental surfaces,
for at least 16 hours but no longer than 4 days.

Hepatitis D Virus

This virus needs help from the hepatitis B virus to be infectious, so it only infects someone with
hepatitis B.

Hepatitis E Virus

This virus is spread similar to the hepatitis A virus and causes acute disease similar to the others.

3 in 1000 get HIV after needle stick injury


4 in 1000 get HEBc  after needle stick injury
the risk is less than that in hep b because we are immunized that from odell book 2010.
** HIV viruse has the least ability to survive outside the body as it cannot survive outside the body
for more than a maximum of 30 to 60 seconds

133. Which have most smooth muscle??

Answer: arterioles

134. Child dental survey 2003


Answer:
There has been a decrease in the average number of filled primary teeth in both five and eight
year-olds. The average number of filled primary teeth in five-year-olds decreased from 0.5 teeth in
1983, to 0.3 teeth in 1993 and 0.2 teeth in 2003
And in eight-year-olds decreased from 1.2 teeth in 1983 to 0.7 teeth in 1993 and 0.5 teeth in 2003.
• In 2003, 62% of 12-year-olds and 50% of 15-year-olds were free from any obvious decay
experience in the permanent teeth. Among both 12 and 15-year olds 87% were free from cavities
into dentine in the permanent teeth.

135. A 30 year old man with unknown allergy to latex goes into anaphylactic shock whilst being
treated in the dental surgery.
Which drug and route of administration is of most benefit in this situation?
A. Hydrocortisone - orally
B. Chlorphenamine – intramuscularly
C. Chlorphenamine - orally
D. Epinephrine - intravenously
E. Epinephrine – intramuscularly

Answer: E
Explnation:
The first line of management for anaphylaxis is 500micrograms or 0.5ml of epinephrine 1:1000
given intramuscularly. It is fast acting whilst hydrocortisone and chlorpheniramine are slower.
Hydrocortisone and chlorpheniramine are not relevant to a dentist in genereal dental practice. When
the paramedics arrive, the patient may be given these other two drugs which give a longer duration
of action than epinephrine. Refer rhesus guidelines and Scully for more explanation on this.

136. What causes difficulty in pronouncing the letters?


F, V – Incisors too far palatally
D,S,T – incorrect overjet or bite, alteration in palatal contour
S becomes ‘th’ - Incisors too far palatally, palate too thick.
Whisteling – palatal vault too high behind incisors
Clicking teeth – OVD, lack of retention

137. What is the irrigant that could be used in root canal irrigation?
What concentration?

Answer: sodium hypochlorite - 2.5%

138. What combines irreversible to Hb (hemoglobin)?


Answer: Carbon monoxide (compound formed is carboxyhaemoglobin)

139. What are the success rate of first time RCT and the success rate of Re-RCT?
Answer: Success rate for first time RCT is 90% whereas for reRCT is 70%.

2- Should a re-RCT always be referred? Even if the patient is otherwise fit and well?

Answer: Safe bet would be to refer but at the same time explain the patient what the

endodontist will do.

140. BPE score * : is that pocket >7 mm

Answer: when attachment loss (recession and pocket depth) is more than 7mm or there
is furcation involvement.

141. What type of fluoride (topical or systemic) you will prescribe for a 6 year and a 12 year old
living in an area with water fluoridation of less than 0.3 ppm.
Answer: for <3- 0.25mg
For 6 year old - 0.5mg
For 12 year old - 1mg
For > 3 years 1350 – 1400ppm of toothpaste is advised.

142. .best method to detect sickle cell anemia:


a. full blood count – iron def
b.siderex - if results needed urgently
c. Hb electrophoresis – sickle cell anaemia.(on all afro Caribbean, Mediterranean, middle eastern
and Indian)
Answer: C

143. .iron def anemia is: Microcytic hypochromic- iron def (chronic blood loss, inadequate diet)

A. Macrocytic anaemia – B12 (pernicious anemia -deficit intrinsic factor). Folate deficiency –
usually dietary, illness (e.g celiac disease, skin disease) or drugs (phenytoin, methotrexate,
trimothoprim and co-trimoxazole)
Management: IM hydroxocobalamin 1mg 3monthly to treat B12 deficiency and folic acid 5mg od
for folate deficiency
MCV > 100fl

B. Microcytic hypochromic- iron def (chronic blood loss, inadequate diet)


Diagnosis: FBC (low serum iron and increased total iron binding capacity (TIBC))
Increased RBC and zinc protoporphyrin is a fast and sensitive early test.
Management: ferrous sulfate 200mg tds.
MCV<78fl

C. Noromochromic noromocytic – anaemia of chronic disease. Other causes pregnancy, acute blood
loss, hemolytic anemia and aplastic anaemia.
Diagnosis TIBC is reduced

Answer: B

144. Last deciduous tooth to be replaced by permanent one:


a.max canine
b.max 2nd molar
c.mand canine

Answer: A

145. Elongation of pulp chamber seen in:


a.gemination
b.turner teeth
c.taurodontism
d.dilaceration

Answer: C
Taurodontism is found in association with amelogenesis imperfecta , ectodermal dysplasia and
tricho-dento-osseous syndrome. The term means "bull like" teeth derived from similarity of these
teeth to those of ungulate or cud-chewing animals.
Turner's hypoplasia is an abnormality found in teeth. Its appearance is variable, though usually is
manifested as a portion of missing or diminished enamel on permanent teeth. Unlike other
abnormalities which affect a vast number of teeth, Turner's hypoplasia usually affects only one
tooth in the mouth and, it is referred to as a Turner's tooth.
If Turner's hypoplasia is found on a canine or a premolar, the most likely cause is an infection that
was present when the primary (baby) tooth was still in the mouth. Most likely, the primary tooth
was heavily decayed and an area of inflamed tissues around the root of the tooth (called a periapical
inflammation), affecting the development of the permanent tooth. The tooth most likely affected by
this cause is the canine tooth. The appearance of the abnormality will depend on the severity and
longevity of the infection.

Gemination arises when two teeth develop from one tooth bud and, as a result, the patient has an
extra tooth, in contrast to fusion, where the patient would appear to be missing one tooth. Fused
teeth arise through union of two normally separated tooth germs, and depending upon the stage of
development of the teeth at the time of union, it may be either complete or incomplete. On some
occasions, two independent pulp chambers and root canals can be seen. However, fusion can also be
the union of a normal tooth bud to a supernumerary tooth germ. In these cases, the number of teeth
is also normal and differentiation from gemination may be very difficult, if not impossible. In
geminated teeth, division is usually incomplete and results in a large tooth crown that has a single
root and a single canal. Both gemination and fusion are prevalent in primary dentition, with
incisors being more affected

Concrescence is a condition of teeth where the cementum overlying the roots of at least two teeth
join together. The cause can sometimes be attributed to trauma or crowding of teeth. Surgical
separation of the teeth may be necessary if one is to be extracted.

146. Potassium sulfate is added to gypsum to


a. increase setting expansion
b.decrease working time
Answer: B

147. The component of bacteria that attaches to toll process:


a. prop ionic acid
b. lipopolysachharide
Answer: B

148. the maximum cartridges of lignocaine that can be given to a pt


a.7
b.14.
C.6
Answer: A

149. Hb e antigen means:


a. high risk infection
b. recent infection
c. immune to infection

Answer: A
150.question of candidiasis which is caused in particular group/races(dont remember the exact
ethnic group)
a.c.tropicalis
b.c.albicans

151.the approach for Gilles method in zygomatic #,


A.vertical incision is made in the temporal hairline, and the elevator is then tunneled underneath the
temporalis fascia,

b.the same between the temporal fascia and temporalis muscle

Gilles method

A 3-cm incision placed 4 cm superior to the zygomatic arch and posterior to the temporal hairline
can be fashioned to allow direct access to the arch. This approach (ie, Gillies approach) allows
accurate fracture reduction by means of a bimanual technique. The surgeon creates a skin incision,
the surgeon carries down a dissection through the superficial temporal fascia and the temporalis
muscle fascia (deep temporal fascia). A plane is carried forward, superior to the temporalis muscle
to the zygomatic arch.

See the image below.

Gillies approach to reduction of a zygomatic arch fracture.

Once this conduit is created, a periosteal elevator is positioned beneath the zygoma. Lateral traction
is placed on the elevator while the surgeon's free hand palpates the fracture site during reduction.
Once hemostasis is ensured, the fascia and skin are closed in the usual fashion. Take care to close
the wound with all layers reanastomosed to their respective anatomic partners. Advantages of the
Gillies approach include a scar camouflaged by the patient's hair, accurate bimanual fracture
reduction, and a remote chance of injury to the temporal branch of cranial nerve VII.

Fluroscopy may aid in proper fracture reduction with closed reduction


152.double blind method:
12.lot of ques based on histological features of phemphigus
Pemphigus vulgaris: pathology
• Loss of intercellular adherence of suprabasal spinous cells (acantholysis)
• Formation of clefts immediately superficial to the basal cells
• Extension of clefts to form intraepithelial vesicles (Fig. 13.22)
• Rupture of vesicles to form ulcers
• High titre of circulating antibodies to epithelial 'intercellular cement substance' (desmoglein 3)
• Binding of antibodies to intercellular substance detectable by fluorescence microscopy

13.trigeminal neuralgia features:


a.trigger zones
b.night pain

153.the amt of adrenailne used 1:X,X is


a.100
b.1000
c.10000
Answer: B

154.ques based on TNM classfn


THE TNM SYSTEM OF STAGING
Tl less than 2 cm greatest dimension
T2 2- 4 cm greatest dimension
T3 > 4 cm greatest dimension
T4 extending to adjacent structures, e.g. bone, sinus, skin

NO No regional lymph node metastases


Nl One ipsilateral node < 3 cm diameter
N2 Ipsilateral or contralateral nodes 3-6 cm diameter
N3 Lymph node metastasis > 6 cm diameter

MO No distant metastases
Ml Distant metastasis (e.g. liver, lung)

Scores are compiled to designate the stage as follows:


Stage 1 Tl NO MO
Stage 2 T2NOMO
Stage 3 T3NOMO
Stage 4 any T4
any N2 or N3
any Ml

SITES OF ORAL CANCER


The lower lip is the most frequent site of oral cancer overall, while the tongue is the most frequently
affected site within the mouth. In the oral cavity, the majority of cancers are concentrated in the
lower part of the mouth, particularly the lateral borders of the tongue, the adjacent floor of the
mouth and lingual aspect of the alveolar margin, forming a U-shaped area extending back towards
the oropharynx (Fig. 17.2). This accounts for only about 20% of the whole area of the interior of the
oral cavity, but 70% of oral cancers are concentrated there. This distribution may be due to the
likelihood that carcinogens could pool and concentrate in the lower mouth before swallowing.
For the same reason, the hard palate and central dorsum of tongue are very rarely affected.

16.Ques based on BPE

155.phemphigus shows:
a.acantholysis
b..acanthosis
Answer:A

156. IgG and C3 are seen in:


a. lichen planus
b. phemphigoid
c. erythema multiform
d. phemphigus vulgaris
Answer: D

157. Commonly used treatment of candidiasis:


a. flucanozole
b. micanazole
c. nystatin
Answer: C

158. actinomycosis is differentiated from osetomyelitis by the presence of:


a. sequestrem
b. involucrum
c. sulfpur granules
d. pain
Answer: C

159. wharf’s assessment is used in


a. fractures
b.3rd molar assessment

160.AVPU is used in
a.ABCDE
b.ABC

161.defn epidemiology
Epidemiology is defined as the study of distribution and determinants of health related states in
populations and use of this study to address health related problems.

162.histopath lesion closest to advancing edge in enamel caries:


a. dark zone
b. surface zone
c. translucent zone
Answer: C

163. pogonion,menton,and gonion are the bony landmarks in


a. mandible
b. max
c. bony chin
Answer: A

164. best method of fluoridation – systemic

165.simplest way to correct a lateral incisor cross bite :


a.removal appliance
b.fixed appliance
Answer: A

167. case scenarios where one central incisor is missing in a child who has had a previous history
of trauma with an avulsed primary incisor, the cause is
a.scar tissue
b.supernumerary teeth
c.tubercle
d.frenum

168. based on AAS(American anesthesia association) when can u treat a dental pt:
a. type /class 1
b. type 1 or 2
c.type 5
d.type 4 and 5
ASA Physical Status Classification System
ASA Physical Status 1 - A normal healthy patient
ASA Physical Status 2 - A patient with mild systemic disease
ASA Physical Status 3 - A patient with severe systemic disease
ASA Physical Status 4 - A patient with severe systemic disease that is a constant threat to life
ASA Physical Status 5 - A moribund patient who is not expected to survive without the operation
ASA Physical Status 6 - A declared brain-dead patient whose organs are being removed for donor
purposes

169.ques on pulpotomy

170.pontic

171.when can u say lkp is turning malignant:

172.ques on epi dysplasia

173.What is the concentration of chlorhexidine in a gel?

0.2%, 0.5%, 1%, 5%

Answer: Chlorhexidine as a mouthwash is 0.2% and gel is 1% (Corsodyl)


Ref : pink book page no. 32

171. Q1.
3 1 3

1 4 3

174. A 46-year-old male smoker presents as a new patient complaining of bleeding gums, bad
breath and his BPE score is given as above:

 
Select the most appropriate initial radiographic examination.

A. Bitewings

B. Bitewings and periapical views of selected teeth

C. Full mouth periapicals

D. Periapicals of the lower incisors

E. Vertical bitewings

Answer: B

175. Q2.

A. 1 month

B. 3 months

C. 6 months

D. 12 months

E. 24 months

F. 36 months

Choose from the options above the period of time which should elapse before the next radiographic
review in the scenarios below. Each option may be used once, more than once, or not at all.

1. A 13-year-old patient designated as having a high caries risk. - 6 months

2. A 15-year-old patient considered to be at moderate risk of future

caries. - 12 months

3. A 32-year-old patient still considered as at high risk of future caries. - 6 months

4. A 9-year-old patient at low caries risk. 12-18 month

5. A 25-year-old patient at moderate risk of future caries. Annually


6. A 38-year-old patient who has had a full coverage crown placed. Annually

7. A 27-year-old patient who has had orthograde endodontic treatment to UL6. Annually

8. A 7-year-old who has had a vital pulpotomy following trauma to UL1. Annually

 High caries risk: 6 monthly interval


Moderate risk: Annually
Low risk: primary dentition: 12-18 month
Permanent dentition: 2 years

176.Q3. 

A. Clubbing

B. Erythematous palms

C. Evidence of widespread scratching

D. Flattened nails (koilonychias)

E. Keratotic striations

F. Pitted nails

G. Purpura

H. 'Target' lesions.

For each of the following clinical scenarios identify the most appropriate skin/nail condtion from
those provided. Each option may be used once, more than once, or not at all

1. A 45 year old woman with known liver disease. - H

2. A 56-year-old psoriasis sufferer. B (can also be ‘C’ Due to itching)

3. A woman with a hypochromic microcytic anaemia. - D

4. A heavy smoker with haemoptysis. - A

5. A patient with a history of gallstones presenting with dark urine and jaundice. - C
Explnation for Q 5- Post-hepatic jaundice, also called obstructive jaundice, is caused by an
interruption to the drainage of bile in the biliary system. The most common causes are gallstones in
the common bile duct, and pancreatic cancer in the head of the pancreas
Patients also can present with elevated serum cholesterol, and often complain of severe itching or
"pruritus" because of the deposition of bile salts.

177. Ankylosis

Our teeth stay anchored to our jawbone, thanks to healthy tissue and ligaments. Normally, the roots
and ligaments for primary ("baby") teeth will dissolve, allowing the baby teeth to come out and the
permanent teeth to emerge from underneath.

Sometimes the roots don't dissolve properly, and instead they fuse directly to the jawbone. The
fusion may occur because the ligament that normally surrounds the tooth in the jawbone is lost. This
condition is called ankylosis (ang-kil-LO-sis).

Causes of ankylosis

The causes for ankylosis vary; for example, the healthy ligament cells may dry up and die because
of:

 Inflammation or infection
 Problems with metabolism or normal bone growth
 Congenital tendencies
 Gaps in the membranes around the tooth
 Abnormal pressure from the tongue
 Crushing or other damage

Ankylosis also may develop in children or adults when an injury (such as with sports or accidents)
causes the top of the tooth to be broken off, leaving the roots behind.

Some studies have indicated that 1 to 3 percent of children may have ankylosis with one or more
primary teeth, while other studies have indicated more than 38 percent of children may have
ankylosis. Studies also have shown that children or adolescents with 1 or 2 ankylosed teeth are more
likely to have additional ankylosis later.

These variances in statistics may be due to different ways of diagnosing ankylosis, ethnic factors, or
other reasons. (For example, a study at the Pediatric Dentistry Clinic of the University of
Minnesota's School of Dentistry focused only on Caucasian children.) But the studies all come to
the same conclusion: while it may not be an unusual condition, especially with lower first molars,
ankylosis is a serious condition. To ensure the healthy development of your child's teeth, ankylosis
must be addressed.

Signs and sounds of ankylosis


Some of the signs of ankylosis may include:

 The ankylosed tooth appears submerged because its roots don't grow at same rate as other
teeth.
 The opposing tooth visibly loses alignment as it grows out of its socket.
 The permanent tooth is blocked out of position because the primary tooth's roots failed to
dissolve normally.

Often we can diagnose ankylosis simply through observation. For example, a tooth noticeably lower
in growth is a tell-tale sign of ankylosis. Also, with x-rays taken at regular checkups, we can view
the development of permanent teeth that may be blocked by the primary teeth's roots.

Special, sophisticated instruments can be used to identify ankylosis. But one of the most reliable
instruments is our ears! Simply by tapping on the teeth, we can hear a distinct difference between an
ankylosed tooth and a normally growing tooth. That's why we may use the clean handle of a dental
mirror to tap lightly on children's teeth as part of their regular dental exam. (An ankylosed tooth has
a higher pitched or dulled sound as opposed to the more cushioned sound of a normal tooth.)

Solutions for ankylosis

We may recommend several solutions for ankylosis, depending on the specific tooth and your
child's situation. For example, with younger children, we may simply recommend monitoring the
situation for a period of time, to watch how the primary and permanent teeth develop.

Sooner or later we may recommend removing the ankylosed tooth, to ensure that your child's
permanent teeth can develop straight and strong. We also may recommend surgery to expose,
protect, or reposition the emerging tooth.

If we must remove a primary tooth before the permanent tooth is ready to emerge, we may use
space maintainers to ensure that surrounding teeth do not crowd out the emerging permanent tooth.
Sometimes orthodontic steps may be taken to ensure that your child's teeth align and the bite is
correct. In general, the sooner we can deal with your child's ankylosed tooth, the fewer problems
that are likely to develop later.

178. A mother brings her child to the after hours clinic suffering spontaneous excess bleeding from
his gingival, what is the expected diagnosis?
- acute leukemia
- injury
- stress
- scurvey
Answer: A

179. The most common type of injury caused by a Non accidental Injury is:
- burnt tip of tongue
- ulcer on the gingiva
- lacerated labial frenum

Answer: B

180. The most common scale used for diagnosing a brain injury is:
Answer: Glasgow Coma Scale (GCS).

181. The most important feature to differentiate between an upper neuron motor lesion and a lower
neuron motor lesion is:
- eye involvement
- ear involvement
- forehead involvement
- anaesthesia of the facial nerve
Answer: C
Main difference: frontalis ans orbicularis oculi muscle is less paralysed, the facial muscle may
appear non-paralysed during emotional reactions and there is usually a degree of paralysis of
ipsilateral arm and leg or aphasia. (Scully page no – 578)

182. A supernumerary tooth next to the maxillary lateral incisor is called a:


- conical tooth
- supplementary
- tuberculate

Answer: B

183. A patient attends your clinic for swollen lumph nodes, after examination, the diagnosis
revieled T2N1M0, what does that mean?
- A tumor of 1 cm, with one lymph node involved and no metastasis.
- A tumor of 2 cm, with one lymph node of 1 cm involved and no metastasis.
- A tumor of 2-4 cm, with ipsilateral lymph nodes involved and no metastasis.

Answer: C

184. You suspect the patient suffers hepatitis B symptoms, after inestigation, the test show HBe Ag
antibodies, what does that indicate?
- the patient is getting better
- the patient is in a transmitting state of the disease
Answer:B
185. Peptic ulcers are caused by the following type of bacteria:
- mycobacterium tuberculosis
- sterptococus oralis
- helicobacter pylori

Answer: C
186.According to the modified Anxiety Scale, a dental phobic patient is on a scale of:
- 9-14
- 15-20
- 20-27
- 27-35

10- The age range for the following diseases are: pemphigus vulgaris, mucouse membrane
pemphigoid,

lichean planus
PEMPHIGUS VULGARIS
Management
The diagnosis must be confirmed as early as possible. Biopsy is essential and the changes are
sufficiently characteristic to make a diagnosis. Immunofluorescence microscopy should be used to
exclude similar but less common diseases. Once the diagnosis has been confirmed,
immunosuppressive treatment is required. There is little consensus about dosage but a typical
regimen is 80-100 mg/day of predisolone plus azathioprine (1-1.5 mg/kg daily). Azathioprine is
given to allow doses of the corticosteroid to be lowered and reduce their side-effects.

Pathology
Histologically there is loss of attachment and separation of the full thickness of the epithelium from
the connective tissue at basement membrane level. Epithelium, though separated, remains for a time
intact and forms the roof of a bulla (Fig. 13.26). The floor of the bulla is formed by connective
tissue alone, infiltrated with inflammatory cells. The disease is immunologically mediated,
and binding of immunoglobulin or more frequently of complement components along the basement
membrane zone can be demonstrated (Fig. 13.27). Circulating autoantibodies are detectable by
sensitive techniques.
Management
The diagnosis is confirmed by biopsy and immunofluorescence microscopy but it is preferable to
obtain an intact vesicle or bulla. Oral mucous membrane pemphigoid can often be effectively
controlled with topical corticosteroids. Doses are small and without systemic effects. Because of the
possible risk to sight, ocular examination is necessary if early changes in the eyes are suspected. If
the eyes become involved, systemic corticosteroids have to be given and are effective.
11- The percentage of people surviving a cardiac arrest in the UK is…
12- Winters lines on an OPT is used to:….
13- Verifiable and non-verifiable CPD hours, and core CPD hours
14- Maximum dose allowed for: amoxicillin, aspirin, ibuprofen, paracetamol
15- The action of these drugs
16- What do the following trade names indicate: Panadol, Dactarin, Fusidin, Augmentin..
17- Endodontics: K-files, Hedstrom files, EDTA, Gutta percha, Chloroform
18- Root filling techniques: step down, step back, vertical condensation, thermal condensation
19- Kennedy classification,
20- Eruption times and numbering systems for teeth
21- Medical emergencies: anaphylactic shock, hypoglycaemia, MI, asthma
22- A patient suddenly collapses, what is the first thing you need to do: call 999, check the airway is
clear, Give him glucagon..
23- Types of suturing needles, their sizes and for which wounds they are used
24- You are a VT working in a dental clinic, to which departments do you refer the following
patients: a patient suffering Addison's disease, a patient suddenly becomes confused with slurred
speech and not had breakfast, a patient needs his third molar to be extracted under sedation. A
patient who looks anaemic..
25- Which of the following materials has an alkenoic reaction: calcium hydroxide, ZOE, GIC
26- To increase the strength of the amalgam and decrease its corrosion which of the following needs
to be added: mercury, copper, silver..

204. A man has diet-controlled type 2 diabetes. He consumes 30 glasses of alcohol and smokes 40
cigarettes per day. He is otherwise healthy. What may be the reason of dry mouth in this person?

Sjogren's sydrome, candidosis, dehydration, drug-induced.

Answer. C
205. Which among the following is least important in taking consent?

The trainee understands the procedure of taking consent, he has done it previously, he understands
the risks associated with the procedure, he explains the risk to the patient, he follows the procedure
from a textbook .
Answer: trainee done it previously

205. What colour bag for disposing waste in the waiting room of a clinic
Answer: Black

206. Laws and regulations governing different scenarios e.g Dental Nurse wanting to take
radiograph. Options included IRMER, Health and Safety, COSHH.

206 Which sealant is more likely to cause tissue damage if extruded – AH plus, grossman's sealer,
tubliseal.

207Which LA becomes neurotoxic at 4%.


Answer: Articine

207Cellularity of exocrine glands-Uni,multi,bi etc as choices.

209. What is the kind of epithelium lining exocrine ducts?

210. What affects cardiac output-Stroke vol and heart rate.

211. .what decreases when u exercise


Answer: total peripheral resistance

212. Patients with hypothyroidism should not receive conscious sedation with benzodiazepines, but
is possible to use nitrous oxide and oxygen. The problem with benzodiazepines and opioid
analgesics is the risk of myxoedema coma. I think this applies for both treated and untreated
patients, because thyroxine is only given for symptomatic patients.

213. A 64 year old man with type 2 diabetes and raised blood glucose and sodium
levels is already being treated with iv insulin and saline what drug should be
added?

Answer: IS HEPARIN BECAUSE THEY ARE MORE LIKELY TO DEVELOP BLOOD CLOTS
(THROMBUS ) IN THE BLOOD .SO TO AVOID THAT HEPARIN IS GIVEN
REF: EMQ IN DENTISTRY

214. cells in herpetic stomatitis? options lymphos ,monos,neutrophils,macros,plasma


cells
215.. percantage of patients indicated to hospitals by infection? options
5%,10%,15%,20%
216. muscle involved in snoring?
Answer; genioglossus, palatoglossus

217. How many cpd hrs do the nurses need from 1st August 2008 (started compulsory)
Answer: 150 in 5 year cycle, 50 verifiable.

218.. How many cpd hrs does the dentist need?


Answer: 250 in 5 year cycle, 75 verifiable, 50 approximately per year

219. Core subject for CPD


Answer:
Medical emergencies - 10 hrs
Disinfection/decontamination – 5hrs
Radigraphy/radiation protection – 5 hrs
For dentists working in clinical environment, CPD in legal/ethical issues and handling complaints
should be carried out (verifiable and non verifiable)

220. When an individual is exposed to radiation a certain amount of radiation is needed before
clinical signs of damage to somatic cells appear.
For these effects to occur a minimum radiation dose has to be exceeded and this is known as:
A. Background radiation dose
B. Threshold dose
C. Equivalent dose
D. Absorbed dose
E. Effective dose
Answer: D

221. A panoramic cassette was opened in the darkroom to remove and process the exposed film. On
opening the cassette, a piece of paper was discovered on the surface of the intensifying screen.
What kind of artefact would the presence of paper in the cassette most likely produce?
A. A black artefact
B. No artefact
C. A white artifact
D. Reticulation
E. Dichroic fog
Answer: A

222. You arrive at a new practice and notice that almost every radiograph in the patient's notes has
turned brown. Your nurse confirms that this is a widespread problem that no-one has ever remedied
and she also remarks that the films tend to get browner with age.
What corrective action will you take to remedy the problem?
A. Develop the films for the correct period of time
B. Heat the developer to a higher temperature
C. Ensure films are fixed for the correct period of time
D. Wash the films properly after fixing
E. Change the solutions more regularly
Answer: D

223. The size of the wire used for splinting an avulsed tooth and why we do bucally.
Answer: It is 0.6 mm (or basically a semi-soft wire). Wire placed buccally because it is easier
during treating trauma and better moisture control for bonding the wire to the teeth.

224. The gutta percha becomes soft at 65c and melts at 100c

225. The management of herpes labialis is by topical antivirals NOT systemic (tablets) as follows:
Aciclovir cream 5% 4 hourly ( is used during the prodromal symptoms stage, before ulcers appear,
otherwise it won't be effective)
Penciclovir cream 1% 2 hourly ( it can be used after the appearance of the ulcers and it accelerates
the healing process)
Systemic aciclovir (tablets) are only used in immunocompromised patients with hepres labialis and
in such case the lesions are more severe and appear intra orally as well...

226. basically there are three types of hand hygiene proc:

1,social for 10 to 15 sec using liquid soap for non clinical activities like decontamination
2.hygienic for 15 - 30 sec using antimicrobial disinfectants before and after using gloves(clinical
proc)
3.surgical scrub 2-3 mins using 4% chlorhexidinegluconate or 7.5% povidone iodine after
oral,periodontal or implant surgery
ref: BDA may 09

227. MAX.RECOMMENDED ALCOHOL UNITS FOR MEN PER WEEK


4. MAX.RECOMEMMENDED ALCOHOL UNITS FOR WOMAN PER WEEK

 Men should drink no more than 21 units of alcohol per week (and no more than four units in any
one day).
 Women should drink no more than 14 units of alcohol per week (and no more than three units in
any one day).
Man should not regularly drink more than 3-4 alcohol units a day (equivalent to a pint and a half
of 4% beer) and a woman should not regularly exceed 2-3 units a day (equivalent to a 175ml glass
of 13% wine).
1unit of alcohol = 1/2 pint or 10 ml or 8gm

228. 15 YEAR BOY LOST HIS CENTRAL TOOTH IN SKIING-WHAT IS THE BEST OPT ?

Answer: 15 yrs is still growing, so offer only Dt or resin bonded and you should inform him about
the space that going to happen below the pontic.

229. What part of the nervous system causes increased salivary secretion?

Answer: parasympathetic
230. What part of the nervous system increases heart rate?

Answer: Sympathetic

231. What hormone increases blood glucose?


Answer: glucagon

232. What hormone reduces blood glucose?


Answer: insulin

233. What hormone increases blood glucose and potassium?


Answer: Cortisol.

234. 10% of UK population receives fluoridated water.

235. Is Myasthenia Gravis and Grave's disease type 2 or type 5 hypersensitivity reaction?
Answer: Both, mediated by IgM and IgG

236. Which immunoglobulin increases in gingival inflammation?


Answer: IgG

237. What type of cell junctions are seen in oral mucosa?


Answer: desmosomes,gap junctions and zona occiudens

238. Percentage of caries in children in u.k


Answer: 43% 5 years old and 57% 8 years old by children heath survey 2003

239. Most abundant extracellular cation?


Answer: Na is main extracellular cation while K is main intracellular cation.

240. ig present in periodontitis?


Answer: IgG especially IgG2

241. Which immunoglobulin is dimeric?

Answer: secrotory IgA

242. Antibiotic contraindicated in lactating mothers?


Answer: QUINOLONES, TETRACYLINES, METRANIDAZOLE, SULFONAMIDES

243. Which nerve makes a pt. look upwards n downwards?


Answer: OCULOMOTOR NERVE (3RD NERVE)

244. Which has softer walls? veins, artery or arterioles?


Answer: Veins
245. Most common failure of maryland bridge?
Answer: Retention failure is the most common prob associated with Maryland bridges.
246. % of alcoholics in uk?
Answer: 5% ADULTS ARE ALCHOLICS

247. %children with caries less than 5years old?


Answer: 43% OF 5 YEARS OLD

248. Cement used to temporary cement a crown?


Answer: Cementation of temporary crowns is with the help of ZOE.(tempbond)

249. Which cement is more susceptible to fail if contaminated with saliva?


Answer: Cement most susceptible to failure if contaminated with saliva  is GIC

250. % of ppl in uk who drink too much?


Answer: 50% OF MEN AND 30% OF WOMEN EXCEEDS THE RECOMMENDED WEEKLY
GUIDELINES

251. Measles caused by rna or dna virus?


Answer: RNA virus

252. cjd/ prions most infectious? n that dnt get sterilised.


Answer: PRIONS RESISTANT TO INACTIVATION BY CONVENTIONAL STRLIZATION
METHODS

253. Which is elastic artery or vein?


Answer: Artery

254. The layers of the skin are as follows:


from in to outwards
stratum basale
stratum spinosum
stratum granulosum
stratum lucidum
stratum corneum

The stratum lucidum is a layer of the epidermis found throughout the body, but is thickest on the
palms of the hands and the soles of the feet.

255. What is the main function of EDTA?


Answer: Is a chelating agent, Negotiate sclerosed canals, dissolve inorganic component of smear
layer which sodium hypochlorite can’t do.
Sodium hypochlorite: dissolve organic debris and it’s bactericidal, 2.5% conc available chlorine.
256. A 22 year old woman has acute gingival hypertrophy, spontaneous bleeding from the gingiva
and complains of weakness and anorexia. Her blood analysis was as follows: HB=12gm,
Neutrophils=90%, Monocytes=1%, Platelets=250000, WBC=100000, Lymphocytes=9%,
Eosinophils=0%
The most likely diagnosis is:
a. Myelogenous leukaemia
b. Infectious mononucleosis /glandular fever/
c. Thrombocytopenic purpura
d. Gingivitis of local aetiological origin
e. Pernicious anaemia /Vitamin B12 deficiency/

Answer: B

257. Which of the following organisms are pathognomonic of acute necrotic ulcerative gingivitis?
a. Spirochaetes and fusobacterium SP
b. Spirochaetes and eikenella corrodes
c. Polymorphs and lymphocytes
d. Actinobacillus actinomycetes comitans oral capnocytophaga
e. Porphyromonas gingivalis and prevotella intermedia

Answer: A

258. Which of the following is true regarding gingivosis (Desquamative gingivitis)


a. It is caused by hormononal imbalance
b. Is seen only at or after menopause
c. Is frequently caused by lichen planus
d. Is a variant pregnancy gingivitis
e. Is related to nutritional disturbance

Answer: C

259. The treatment of Localised Juvenile Periodontitis is frequently supplemented with


tetracycline
a. because flora involved is predominant:
b. Aerobic
c. Strictly anaerobic
d. Facultative or microaerophilic
e. Resistant to other antibiotic
Answer: D

Prevotella intermedia - Localised Juvenile Periodontitis, necrotizing periodontal disease.


Prophyromonas gingivalis – chronic periodontitis and aggressive periodontitis
Actinomycetemcomitans – microaerophilic, capnophilic gram –ve rod, pathogen in aggressive
periodontitis
260. The most accurate way to evaluate the effectiveness of root planning is by:
a. Inspect the root surface with an instrument for root smoothness
b. Use air for visual inspection
c. Activate a curette against root surface and listen for a high pitched sound which indicates a
smooth, hard surface.
d. Evaluate the soft tissue at the end of the appointment for a decrease oedema and bleeding
e. Evaluate the soft tissues 10 to 14 days later.
Answer: A

261. Probe pressure at the sulculus of pocket should be enough to:


a. Feel the top of the crestal bone
b. Balance the pressure between fulcrum and grasp
c. Define the location of the calculus deposit
d. Feel the coronal end of the attached tissues
e. Limit the lateral pressure
Answer: D

262) A curette may be inserted to the level of the attached gingiva with minimal trauma to the
tissues because of:
a. Has a round base
b. Is easy to sharpen
c. Has rounded cutting edges
d. Provides good tactile sensitivity
e. Has two cutting edges
Answer: C
Explnation: A periodontal curette is a dental instrument used primarily in the prophylactic and
periodontal care of human teeth. The working tips are fashioned in a variety of shapes and sizes, but
they are always rounded at the tip in order to make subgingival cleansing less traumatic to the
gingiva.

263) Tetracycline hydrochloride conditioning of root surface in periodontal surgery is to:


a. Sterilise the root surface
b. May enhance binding of fibronectin and fibroblast
c. Aids in re-mineralising the root surface
d. Assist the biding of lamina dura
e. Prevents post operative infections
Answer: B

264. A cold stimulus applied to a tooth will produce a hypersensitive response if the tooth
A. is nonvital.
B. has a periodontal pocket.
C. has a hyperemic pulp.
D. has chronic proliferative pulpitis.
Answer: C

265. The location and extent of subgingival calculus is most accurately determined clinically by
A. radiopaque solution used in conjunction with radiographs.
B. disclosing solution.
C. probing with a fine instrument.
D. visual inspection.
Answer: C

266. A characteristic sign of aggressive periodontitis in an adolescent (juvenile periodontitis) is


A. marginal gingivitis.
B. painful, burning gingivae.
C. hyperplastic gingivitis.
D. drifting of the teeth.
Answer: A

267. Which treatment procedure is indicated for a patient with asymptomatic age related gingival
recession?
A. Connective tissue graft.
B. Gingivoplasty.
C. Lateral sliding flap.
D. Gingival graft.
E. No treatment.
Answer: E

268. The absence of adequate drainage in a periodontal pocket may result in


A. cyst formation.
B. abscess formation.
C. epithelial hyperplasia.
D. increased calculus formation.
Answer: B

269. For an otherwise healthy patient, with an acute localized periodontal abscess, initial treatment
must include
A. scaling and root planing.
B. occlusal adjustment.
C. prescription of an antibiotic.
D. prescription of an analgesic.
Answer: A

270. The most likely cause of tooth loss following a tunneling procedure to provide complete access
for a mandibular Class III furcation involvement is
A. root caries.
B. root sensitivity.
C. pulpal involvement.
D. recurrent pocketing.
Answer: A
271. Maximum shrinkage after gingival curettage can be expected from tissue that is
A. fibroedematous.
B. edematous.
C. fibrotic.
D. formed within an infrabony pocket.
E. associated with exudate formation.
Answer: B
272. Apart from Dentigerous Cyst, which other lesions are associated with absence of teeth?
Answer:
Adenomatod odontogenic tumor(canines)
Keratocyst
Ameloblastoma
Calcified Odont tumor

273. Regarding Hand washing


Answer: Chlorhexidine gel apart from alcohol and povidone iodine as a surgical hand washing gel.
 
Apart from MRSA, it prevents the spread of clostridium difficle.

274. What is the normal resting flow rate for saliva?


Answer: vary b/w 0.1-0.6ml per min
Stimulated saliva 1-2ml per min

275. .Which nerve is affected if a patient is unable to gaze laterally to the left?
rt abducent-
rt trochlear-
lft abducent-
lft trochlear
Answer: A Left abducent as it supply lateral rectus of the eye which is responsible for lateral
movements

276. development of the parotid gland in utero in which month?


Answer development of parotid gland in utero(at what months)---40-44 days in utero
which would be approx 1.5 months

277. Muscles used in swallowing?

278. Bone formed totally intramembranously?


frontal-parietal- occipital- mandible- sphenoid- temporal
Answer: All
Intramembranous ossificationis:
1. parietal
2. maxilla
3. frontal
4. nasal bone
5.vomer
6. lacrimal

Mixed is: (TOMS)


1. temporal
2. occipital
3. mandible
4. sphenoid

Endocondrial OSSIFICATION:
1. HYOID
2. INFERIOR NASAL CONCHAE
3. Ethmoid bone

Explanation:

Intramembranous ossification

Intramembranous ossification mainly occurs during formation of the flat bones of the skull but also
the mandible, maxilla, and clavicles; the bone is formed from connective tissue such as
mesenchyme tissue rather than from cartilage. The steps in intramembranous ossification are:

1. Development of ossification center


2. Calcification
3. Formation of trabeculae
4. Development of periosteum

Endochondrial ossification

Endochondral ossification, on the other hand, occurs in long bones and most of the rest of the bones
in the body; it involves an initial hyaline cartilage that continues to grow. The steps in endochondral
ossification are:

1. Development of cartilage model


2. Growth of cartilage model
3. Development of the primary ossification center
4. Development of the secondary ossification center
5. Formation of articular cartilage and epiphyseal plate

Endochondral ossification begins with points in the cartilage called "primary ossification centers."
They mostly appear during fetal development, though a few short bones begin their primary
ossification after birth. They are responsible for the formation of the diaphyses of long bones, short
bones and certain parts of irregular bones. Secondary ossification occurs after birth, and forms the
epiphyses of long bones and the extremities of irregular and flat bones. The diaphysis and both
epiphyses of a long bone are separated by a growing zone of cartilage (the epiphyseal plate). When
the child reaches skeletal maturity (18 to 25 years of age), all of the cartilage is replaced by bone,
fusing the diaphysis and both epiphyses together (epiphyseal closure).

279. 5.what condition is caused by failure of closure of the vertebral arches?


Answer: Spina bifida

280.If the gland on 1 side are stimulated,what will the response be like?
Answer: Ipsilateral

281. Muscles attached to the auditory tube?


Answer: Tensor tympani

282. 1. Bitewing radiography is the main special text used to help in diagnosis of proximal caries.
The performance (accuracy) of a diagnostic test like bitewing radiography can be expressed in
terms of sensitivity and specificity.
Which of the following is a reasonable summary of the diagnostic accuracy of bitewing radiography
for proximal caries diagnosis?
A. Moderate sensitivity and low specificity
B. Moderate sensitivity and moderate specificity
C. Moderate sensitivity and high specificity
D. High sensitivity and moderate specificity
E. High sensitivity and high specificity
Answer:E

283. You are interested in finding out what the risk indicators are for a rare form of oral cancer and
decide to undertake a study to examine this.
What type of study would be the most appropriate for addressing this issue?
A. Cohort
B. Prevalence study
C. Clinical trial
D. Case-control study
E. Case-series

Answer: A

284. Sex ratio for oral cancer is 2:3 or 1:2?


Answer: In England and Wales Male: female ratio of oral cancer is 2:1 and in Scotland is 3:1

285. What is mean mode n median


Mean: average
median is described as the numeric value separating the higher half of a sample, a population, or a
probability distribution, from the lower half.
Mode: the mode is the value that occurs most frequently in a data set or a probability distribution
286. What is least important for success of post?
A. diameter,
B. material,
C. luting agent,
D. length
Answer: C

287. Which type of cells is seen in the early stage of chronic periodontitis?

Basophils, eosinophils, lymphocytes, neutrophils, macrophages

4. Which type of cells is seen in late stage of chronic periodontitis?


5. Which type of cells is seen after early stage of chronid periodontitis?

Answer: the term chronic indicates presence of chronic inflammatory cells like macrophages,
monocytes n lymphocytes.

288. Cellularity of exocrine gland?multi, bi,unicellular


Answer: almost all the exocrine in human body are multicelluar except goblet cells are unicellular

289. Type of epithelium lining exocrine gland?


Answer: columnar or cuboidal secretory cells.

290. Which is NOT TRUE in relation to the prescription of 5mg or 10mg of


diazepam for* *sedation*

A.   Patient commonly complain of post operative headache


B.   An acceptable level of anxiolytic action is obtained when the drug is given one
hour preoperatively
C.  There is a profound amnesic action and no side affects
D. Active metabolites can give a level of sedation up to 8 hours post
operatively
E. As Benzodiazepine the action can be reversed with Flumazepil

Answer: C

Explanation: Diazepam has a range of side-effects that are common to most benzodiazepines. Most
common side-effects include:

 Suppression of REM sleep


 Impaired motor function
o Impaired coordination
o Impaired balance
o Dizziness and nausea
 Depression[54]
 Reflex tachycardia[55]

Less commonly paradoxical side-effects can occur and include nervousness, irritability, excitement,
worsening of seizures, insomnia, muscle cramps, changes in libido (increased or decreased libido)
and in some cases, rage, and violence. These adverse reactions are more likely to occur in children,
the elderly, individuals with a history of drug or alcohol abuse and people with a history of
aggression.[4][56][57][58] Diazepam may increase, in some people, the propensity toward self-harming
behaviours and, in extreme cases, may provoke suicidal tendencies or acts.[59] Very rarely dystonia
can occur.[60]

Diazepam may impair the ability to drive vehicles or operate machinery. The impairment is
worsened by consumption of alcohol, because both act as central nervous system depressants.[33]

During the course of therapy, tolerance to the sedative effects usually develops, but not to the
anxiolytic and myorelaxant effects.[61]

Patients with severe attacks of apnea during sleep may suffer respiratory depression
(hypoventilation) leading to respiratory arrest and death.

Diazepam in doses of 5 mg or more causes significant deterioration in alertness performance


combined with increased feelings of sleepiness.[62

291. Which is the danger zone in dental surgery


Answer: the danger zone of the face is the region of the face where infections could pass to the
cavernous sinus and to the brain possibly via the pterygoid plexus and deep facial vein. This could
occur when giving la to the posterior superior alveolar nerves. This area is triangular and extends
from the corners of the mouth to the bridge of the nose, including the nose and maxilla.

292. %ge of children suffering from dental trauma


Answer:
8 year olds- 5%

12 year olds- 11%

15 year olds- 13%

this is for trauma to permanent teeth.

293. what is BMI( body mass index) for? How do we measure it?
Answer:
Its to check if someone is overweight/obese/ underweight. It is calculated with height in meters and
weight in kgs. weight (kgs)/(height in metres)2
 
20-25= normal weight for height, 26-30= overweight, 31-35= obese, 36-40 morbidly obese
BMI= 76 kgs/ 1.76*1.76 = 25

294. Antibiotic for chronic sinusitis


Answer: .doxicycline

295. Depth of topical anesthesia


Answer: 2mm

296. Where bone is lost most in perio disease


Answer: inter dental (approximal)

298. Suture material for lip trauma,oroantral fistula


Answer: black silk

299. Pulp tester, its readings and what they signify


Answer: Exaggerated response to pulp testing is seen in reversible pulpitis and no response or a
reduced response is seen in irreversible pulpitis.
300. Nerve fibers in pulpal pain
Answer:
Delta nerve fiber is responsible for sharp, stabbing pain of toothache.
Polymodal nerve fibers give rise to dull, aching and poorly Localised.

301. what does the translucent/ sclerotic zone in dentin represnt?


Hisopathological zone in the advancing edge of the carious lesion
The four zones of dentin in order from the advancing edge are
1. zone of sclerosis
2. zone of demineralization
3. zone of bacterial invasion
4. zone of destruction
The four zones of enamel in order from the advancing edge are
1. zone of translucency
2. dark zone
3. body lesion
4. surface zone

302. Medical condition in which patient starts using 3-4 pillows at night?
Answer: left sided heart failure because they come down with breathlesness with pulmonary
edema.

303. A pat complaints of loss of taste sensation on the lateral border of tongue after extraction of
3rd molar.which nerves would be damaged?
Answer: lingual nerve
304. a nurse injures her finger with a needle prick.whts the first step she needs to do?
Answer: encourage bleeding, wash hands under running water ,do not scrub.

305. An old man with loose lower denture comes to u, making every yr a new one. will u make a
new denture, suggest implants, wait for 3 months, refer to specialist, alveoloplasty?
Answer: refer to specialist

306. Gingival pocket lining is formed from; reduced E E, hertwigs, dental papilla
Answer: REE

307. Where to discard a face mask?


Answer: clinical waste

308. What’s the color of bin for special waste?


Answer:

309. Do we give antibiotic prophylaxis before dental procedure if pat has prosthetic heart valve?
Answer: No antibiotics required anymore for routine dental procedures.

310. Membranes of expanded polytetraflourethylen have been designed for periodontal regenerative
techniques.
Which of the following defects will respond most predictably to regenerative therapy?

a shallow, wide 1-walled defects

b shallow 2- wall defect

c deep narrow 3- walled defect

d deep narrow 1- walled defect

e shallow narrow 2- walled defect


Answer: C

311. You examine and a patient and find bpe score of 4 in all sextants. radiographs show
generalised bone loss with minimum of 50% of bone support remaining on all teeth.which of the
following factors is important in considering the prognosis of teeth

a age of pt

b oral hygiene status

c bleeding on probing score


d mobilty

e gingival recession.
Answer: B

312 you decide to refer an 8 yr old pt to your local oral surgery dpt for extraction under General
anesthesia

what key reason for asking for GA would you put in the referral letter

a Parents requests GA

b failed to complete treatment under inhalation sedation

c child wouldn’t accept la

d parent thinks hospital is more convenient.


Answer: D

313. pt presents with clinking in their tmj joint, the clink is present midway the opening cycle and is
consistent. There is some preauricular pain and the lateral pterygoid mucscle is tender to resisted
movement test. There is no trismus and the click is absent when the pt closes edege to edge instead
of her normal occlusion

the pt would like rx

the most appriopate occlusal splint is

a stabilization splint

b Localised occlusal interferences splint

c bite raiser

d soft bite guard

e anterior repositioned splint


Answer: D

314. The subodontoblastic plexus of Raschkow occurs:


a. Below the cell bodies of odontoblasts
b. In the root region of the pulp
c. Within the central pulp core
d. Within the cell-rich zone of Weil
Answer: A
315. Which of the following is NOT a major mechanism of action for fluoride in caries inhibition?
a. Increases remineralization of enamel.
b. Inhibits carbohydrate metabolism
c. Reduces enamel solubility.
D. ability to prevent reduction of the pH of plaque
Answer:B

316. The best approach for diagnosis of odontogenic pain is which of the following?
a. Radiographic examination
b. Percussion
c. Visual examination
d. A step-by-step, sequenced examination and testing approach
Answer: D

317. A patient with no positive history came along for scaling. The moment you pick up the scaler
you punch your finger, what should you do?
A. Complete the procedure as nothing has happened
B. Check patient's blood for Hepatitis B antibody HBsAb
C. Check patient's blood for Hepatitis B antigen HBsAg
D. Check dentist's blood for Hepatitis B antibody HBsAb and HIV antigen HIVAg
E. Check dentist's blood for Hepatitis B antigen HBsAg and HIV antibody HIVAb
F. Dentist should go and take a HBsAb vaccine
Answer: A

318. What is least important for success of post?

A. diameter,

B. material,

C. luting agent,

D. length

Answer: C

319. which antibody is found in bacterial and viral infection?

Answer: IgG
level of IgA?
A) Saliva>serum>breastmilk>tears

b) serum>saliva>breastmilk>tears

c) saliva>breastmilk>tears>serum
Answer: B
Alternate answer : SERUM>TEARS>SALIVA>BREAST MILK     OR if to give in figures 
SERUM--40-400mg/dl,TEARS--60-85mg/dl,  SALIVA--6.2-14.5 mg/dl,  BREAST
MILK--approx 0.708 mg/dl                                  

320. Tumor that spread along the neural sheath of the submandibular or parotid duct?
Answer: Adenocystic carcinoma has tendency to spread along nerve pathways. Appear on palate
from minor salivary glands. H/F – cribiform or Swiss cheese pattern.

321. What solution is used for alginate impression disinfection?


Answer: 2% gluteraldehyde with various trade names.

322. How often should you review the smoking pattern


a)6months
b)12months
c)24months
d)3months
Answer:A

323. What is the concentration of sodium hypochloride to clean a blood splatter in the dental
surgery?
Answer: 10,000ppm Sodium hypochlorite, cover it with a disposable cloth and dispose it as
clinical waste.

324. Which antibody is used for dental caries vaccine?is it IgA or IgG?
Answer: IgA INHIBITS THE ADHERENCE of microbes and neutralizes toxins and viruses.

325. what are the bones which grow by endochondral ossification?


Answer: Mandibular condyle, Coronoid process, angle of the mandible all undergoes
endochondrial ossification. (long bones of the body)

326. Which is the most adhesive dental cement?


Answer: glass ionomer cement is adhesive to both dentine and enamel.

327. Is it mercury that causes black staining in amalgam or silver?


Answer: silver for the blackness

328. Impression taking in an apprehensive adult with gag reflex?


Answer: For patients having gag reflex...inhalation anesthesia is the best option. We can also use a
lower tray for upper impression to reduce the contact of impression material with the palate.

329. What we use instead of formcresol now in one visit pulpotomy..


Answer: Ferric sulphate

330. Which one is more aggressive between Okc and ameloblastoma?


Answer: Ameloblastoma is more aggressive in terms of tendency to metastsize and how locally
invasive the tumour is. I havnt read anytext book where ameloblastoma and okc were compared.
you just have to look at the clinical features and decide for yourself. It also depends on what you
mean by aggressive. do you mean recurrence rate, rate of local invasion or ability to metastasize. the
recurrence rate for okc is higher than that of ameloblastoma (11-60%; soames and southam) but
ameloblastoma has a higher tendency to metastasize and I would say its local invasion of bone is
more than that of okc( ameloblastoma causes buccolingual expansion while okc spreads anterio-
posteriorly but can cause mesio buccal expansion as well; soames and southam, cawson and odell).

331. What is the BULL rule??


Answer: BULL rule as you said is for Buccal Upper and Lower Lingual cusps, which are
supporting cusps. The Upper palatal and lower buccal cusps are functional
cusps which bear most of the masticatory stresses.

332. patient with grade 2 mobility you did root planning? When should we again recall patient?
1 week, 1 month or 3 months?
Answer: 3 months

333. Writing 5 faults/defects in a try-in denture/wax- up......what could be the possible causes of
that?
Answer: mid line shift
2.the molar relation not in class 1 on both sides
3.on one side the teeth may not be in arch,for example the lateral may be palatally inclined
4.instaed of a molar a premolar is put
5.the central incisors are put other way like the right and left side wrongly put.
6.there could be open bite on either side.

334. In a patient with chronic periodontitis with BPE score of 4 or * how many times per year do
we have to take radiographs?
Answer.
 Patients with perio disease not under good control - - iopa and/or vertical bitewings of
problem areas every 12-24 mnths n full mouth every 3-5 yrs.
 
 Patients with perio disease under good control - - iopa and/or vertical bitewings of problem
areas every 24- 36 mnths n full mouth every 5 yrs.
 
 dental implants-- 6, 12 n 36 mnths after prosthetic replacement, then every 36 mnths if there
is no clinical problem
 
 Maintenance pt -- full mouth if not avilable, if taken within 24 mnths then only problematic
areas....

335. An apical radiolucency (2mm) is noticed as an incidental radiographic finding associated with
the apex of the Mesiobuccal root of the lower right first molar. The tooth has been root filled but is
2mm short of the radiographic apex. There are no other clinical or radiographic findings and the
patient is fit and well.
What is the most appropriate course of action?
A. Extract the tooth.
B. Redo the root filling
C. Perform periapical surgery.
D. Advise the patient of the situation and monitor clinically and radiographically
E. Prescribe antibiotics then review
Answer. D

336. Which condition is relieved by sleeping on 3-4 pillows?

Answer: orthopnoea
Explnation:
Orthopnea or orthopnoea (Greek from ortho, straight + pnoia, breath) is shortness of breath
(dyspnea) which occurs when lying flat,[1] causing the person to have to sleep propped up in bed or
sitting in a chair. It is the opposite of platypnea.
It is commonly measured according to the number of pillows needed to prop the patient up to enable
breathing (Example: "3 pillow orthopnea").[citation needed]

[edit] Causes
Orthopnea is due to increased distribution of blood to the pulmonary circulation while recumbent,[2]
but usually can be attributed to a more fundamental cause.
Orthopnea is often a symptom of left ventricular heart failure and/or pulmonary edema.[3][4] It can
also occur in those with asthma and chronic bronchitis, as well as those with sleep apnea or panic
disorder.[citation needed] It is also associated with polycystic liver disease

337. What is the minimal preparation cantilever and how is it different from conventional
cantilever.

Answer: A cantilever is a bridge suspended from one end only. For eg, minimal preparation usually
involves no or minor palatal tooth reduction and use of adhesive cements (resembles a tooth
suspended off a wing).  A conventional cantilever preparation involves full crown prep and the
tooth suspended off it.

338. What is the radiation exposure when taking an OPG


    
a 0.001mSv

B 0.01 mSv

c.0.01mSv

d. 1.0mSv

e. 10 mSv
Answer: B and C

339. The use of rectangular collimator reduces the dose of radiation by 50% or 65%??
Answer: 65%

340. what is the optimal shape for collimator


     
     a. square
     b.rectangular
     c. round
     d. hexagonal
     e. circular
Answer: b

341. what is the radiation dose in milli-Sieverts for a patient associated with having an OPG??

A. 0.034
B 0.34
C 34
D 3.4
E 340

Answer: B

342. Which cells are found in hepatic stomatitis


A, lymphcytes
B, monocytes
C,neutrophils
D, macrophages
E, plasma cells

Answer: There are multinucleate giant cells present in herpes labialis which closely relates to
macrophages in the given options.

343. Which is the most common site for Mandible Fracture - Is it the Condyle or Angle?
Answer: condyle is the commonest site followed by angle...30% vs 25%

344. Radiograph for RCT follow up.


Answer:
Further follow-up radiographs should be taken at one year and four years after completion of
treatment – for asymptomatic teeth. Large periapical radiolucencies should be monitored more
frequently.

345. Dental survey 2009 results


The survey consisted of a questionnaire interview with all adults aged over 16 years at all sampled
households, and an oral examination of the mouth and teeth of all those adults who had at least one
natural tooth.

The sample size for the survey was 13,400 households (1,150 in each English Strategic Health
Authority and Wales, and 750 households in Northern Ireland). Data collection for the survey
took place between October 2009 and April 2010, and the final household interview response rate
was 60 per cent.

A total of 11,380 individuals were interviewed, and 6,469 dentate adults were examined, making
this the largest ever epidemiological survey of adult dental health in the United Kingdom.

This report covers the following topics:

 Loss of all natural teeth


 Functional dentition - adults with 21 or more teeth
 Summary of tooth condition

Sound and untreated teeth

Restored, otherwise sound teeth

Decayed or unsound teeth

Decay on the crowns of the teeth

 Regular dental attendance


 Levels of dental anxiety.

Key facts

 Over the last 30 years the proportion of adults in England who had no natural teeth
("edentate") has fallen by 22 percentage points, from 28 per cent in 1978 to 6 per cent in
2009.
 In Wales, the proportion of adults who were edentate has fallen 27 percentage points from
37 per cent in 1978 to 10% in 2009.

 In Northern Ireland, the proportion of adults who were edentate has fallen 26 percentage
points from 33 per cent in 1979 to 7% in 2009


 Eighty-six per cent of dentate adults had 21 or more natural teeth.
 The average number of teeth among all dentate adults was 25.6.
 The average number of sound and untreated teeth was 17.8.

 Over three-fifths (61 per cent) of dentate adults said they attended the dentist for regular
check-ups;
 Twelve per cent of all adults (who had ever been to the dentist) were classified as having
extreme dental anxiety.

The proportion of dentate adults with 21 or more natural teeth varied by country. In England 86 per
cent of dentate adults had 21 or more natural teeth compared with 80 per cent of dentate adults in
Wales. Eighty-four per cent of dentate adults in Northern Ireland had 21 or more natural teeth.

There was a clear socio-economic gradient in the proportion of adults who had 21 or more natural
teeth ranging from 91 per cent of adults from managerial and professional occupation households
to 79 per cent of adults from routine and manual occupation households.
• The majority of dentate adults (71 per cent) had no visible coronal caries.

Over three-fifths of dentate adults said they attended the dentist for regular check-ups.
 The proportion of adults attending the dentist for regular check-ups increased in all three
countries between 1978 and 2009 (1988 to 2009 in Northern Ireland5). The greatest increase
was observed in Wales, up 30 percentage points from 39 per cent in 1978 to 69 per cent in
2009.
Twelve per cent of all adults (who had ever been to the dentist) were classified as having extreme
dental anxiety.
 Extreme dental anxiety was more prevalent among women than men, 17 per cent compared
with 8 per cent respectively.
 There was a clear pattern of higher levels of dental anxiety among younger adults. Levels of
extreme dental anxiety ranged from 15 per cent of adults aged 16 to 24 to 9 per cent of
adults over 85.
346. Cancer- referral in how many days should be seen in secondary care?
Answer: within 2 weeks

347. Which of the following is the most important factor in the disease progression in smokers?

A. Smokers have drier mouths than non-smokers


B. Smokers have poorer oral hygiene than non-smokers.
C. Nicotine will impair the chemotactic and phagocytic properties of PMNs.
D. The gingival blood flow is reduced in smokers.
E. Smokers alter the oral environment encouraging the growth of anaerobic
bacteria

Answer: D

348. What are notifiable disease and examples pls.

Answer: notifiable diseases in England and Wales under public health act 1984, public health
regulations 1988
acute encephalitis,leptosprosis,malaria,measles,meningitis,meningococcal septicemia, mumps,
ophthalmic neonatorum, paratyphoid, plague acute poliomyelitis, rabies, relapsing fever, rubella,
scarlet fever, smallpox, tetanus, TB, typhoid, typhus, viral hemorrhagic fever, viral hepatitis,
anthrax,whoophing cough.
Cholera
diphtheria
dysentery
food poisoning
leprosy

others like SARS, Avian flu.

349. Salivary gland secretions?


Answer:
Parotid - serous
Submandibular - mixed
Sublingual – mucous

350. The percentage of possible damage to the lingual and dental inferior nerve in surgical
extractions of wisdom tooth

Answer:
inferior alveolar nerve - temporary 4-5 %
                                     - permanent 0.2%
Lingual nerve     - temporary 0.2%
                           - permanent 2%
Check the Odell page 116, the blue chart there. They give all the information about nerve damage
and accidents pos surgery

351. Abundant immunoglobulin?


Answer:
IgG is the most abundant in body followed by IgA
Immunoglobulins, quantitative serum
IgA 68 - 378 mg/dL
IgG 768 - 1632 mg/dL
IgM 60 - 263 mg/dL
IgE 10 - 180 IU/L

351. HUMAN TISSUE THAT HEALS WITHOUT SCARRING ?


Answer: Gingiva

352. WHAT IS REQUIRED TO INCREASE THE EFFICIACY AND DECREASE THE


FATIGUE OF UNIVERSAL CURETTE?

Answer: efficacy is increased by sharpness


353. what the maximum period imposed on a dentist on grounds of fitness to practice?
Answer: 12 months

354. The 'SPLINT TIMES' for avulsion, intrusion, extrusion, lateral luxation, root fractures,
alveolar bone fracture?
Concussion – Ressuarance and soft diet
Avulsion – 7- 14 days
Intrusion, extrusion – 1-2 weeks
Luxation – 2-3 weeks
alveolar bone fracture – 3-4 weeks
subluxation – 1-2 weeks
Middle 3rd root fracture – 4 weeks
cervical 1/3 root fracture – 4 months

355. WHICH SALIVARY GLAND TUMOUR  OCCURRS BILATERALLY?


Answer: Warthin's tumor is bilateral, but the two masses usually are at different times. Warthin's
tumor is highly unlikely to become malignant.
http://en.wikipedia.org/wiki/Warthin%27s_tumor

356. An apical radiolucency (2mm) is noticed as an incidental radiographic finding associated with
the apex of the mesiobuccal root of the lower right first molar. The tooth has been root filled but is
2mm short of the radiographic apex. There are no other clinical or radiographic findings and the
patient is fit and well.
What is the most appropriate course of action?
A. Extract the tooth.
B. Redo the root filling
C. Perform periapical surgery.
D. Advise the patient of the situation and monitor clinically and radiographically
E. Prescribe antibiotics then review

Answer: D

357. What is the normal bleeding and clotting time?


Answer: Normal value of clotting time is 5-8 minutes
normal bleeding - 2 - 9 minutes depending on the method used.

358. how many years are we supposed to tell the patient that the following treatments last in
average?
Answer:
Amalgam = class 1,2,& 4  .............. 5-25 years [ median/ average 11 years ].
Anterior composite...........................3-10 years [ median/average   6 years ].
Direct posterior composite=...............3-9 years   [ median/average  5 years  ].
Glass ionomer= class1,3 &5..............3-5 years  [ median/average   4 years]
Cast metal- precious/gold/noble metal=10-25 years[ median/average  17 years].
PFM = .............................................10-25 years [ median/average 18 years ]
 
359. What Local anesthetic is:
Answer:
- used in children  lignocaine
-used in cardiac arrythmias lignocaine without adrenaline 
-neurotoxic at 4%.,    articaine

360. .During swallowing, a) suprahyoid muscles relax b) masseter contract c)


tongue touches the palate
A. a and b
B. a and c
C. b and c
D. none of the above
E. all of the above
Answer: D
Explanation: http://en.wikipedia.org/wiki/Swallowing

361. .Endogenous morphine like substance which can control pain is known as

A. Bradykinins

B.Peptides

C.Prostaglandins

D.Serotonins

E.Enkephalins

Answer: B

Explanation: Endorphins ("endogenous morphine") are endogenous opioid peptides that function
as neurotransmitters. They are produced by the pituitary gland and the hypothalamus in vertebrates
during exercise, excitement, pain, consumption of spicy food, love and orgasm, and they resemble
the opiates in their abilities to produce analgesia and a feeling of well-being.

The term "endorphin" implies a pharmacological activity (analogous to the activity of the
corticosteroid category of biochemicals) as opposed to a specific chemical formulation. It consists
of two parts: endo- and -orphin; these are short forms of the words endogenous and morphine,
intended to mean "a morphine-like substance originating from within the body

362. What is the golden time for the tooth to be out of the socket.Is it 30 min or 45 min.
Answer: 30 minutes

363. What is the taper of a standard GP cone ?

A) 0%
B) 1%

C) 2%

D) 5%

E) 10%
Answer: C

364. A 73 year old man taking fluoexetine for depression and now taking misoprostol and
diclofenac for arthritis
Answer: Diahorrea
365. A 55 year old woman taking inhaled salbutamol for asthma, now prescribed timolol eye drops
for glaucoma
Answer: dyspnoea and wheeze
366. A 70 year old man taking bendrofluazide and enalapril started on doxazosin
Answer: postural hypotension

367. A 23 year old woman with Hodgkins lymphoma, being treated with vincristine and
cyclophosphamide
Answer: neutropenia

368. A 50 year old treated with oxybutynin for bladder over-activity and dihydrocodeine for pain.
Answer: constipation

369 .Which of the following is true regarding lathe cut silver alloy?

A. Has low creep.

B. Requires least amount of mercury.

C. Has tensile strength, both at 15 mins & 7 days comparable to high copper Unicompositional
alloys.

D. Achieves lowest compressive strength at 1 hour.

Answer: A

370. Which antibiotic is safe to prescribe to patient on warfarin,?


Answer:
Warfarin's anticoagulant effect was potentiated by 6 antibiotics (cotrimoxazole, erythromycin,
fluconazole, isoniazid, metronidazole, and miconazole)
Warfarin's anticoagulant effect was inhibited by 3 antibiotics (griseofulvin, rifampin, and nafcillin)

 Safe to prescribe penicillin v (penicillin or amoxicillin)


371. .Bilateral swellings not associated with food smell associated with lymphadenopathy?
Answer:

372. How is acetyl choline released?


Answer:

373. You have recently read a paper in the Guardian about the best treatment for oral candidosis.
This you have decided to use, as the new colleagues have been using. Which of the following
methods will you choose to evaluate the results.
a. Mean
b. Median
c. Standard deviation
d. Standard reduction
Answer: c
374. In which of the following case you will require post-op antibiotics?
a. Radiotherapy for head and neck region
b. Valvular heart disease
c. Pacemaker
Answer: None

375. Parts of the audit.Which is not one of them?


Options:
a. Collect control group
b. Implement change
c. Compare set standards with clinic standards
d. Re audit
Answer: A

376. Which of the following is the best method of clinical evidence?


a. Case control
b. Cohort
c. Systematic review
d. RCT (Randomised control trial)

Answer: C

377. Reason for tingling of hands and feet during treatment of long standing procedures ?

Answer: shortage of blood to the feet or any nerve damage,lyme disease or tarsal
tunnel syndrome causes tingling and numbness in the feet
65Person with swollen salivary glands+swollen lips+ swollen gingiva…diagnosis
Answer: sarcoidosis

378. Photo of two canines completely abraded/attrited with gingival recession and cerv.abrasion
- Give 1 advantage of having these two teeth alone
- Give 2 disadvantages to having these two teeth alone
- T/t options for this patient other than RPD
- If overdentures are given the how would you obtain stability for this
denture- 2 ways
- 4 advantages of overdentures for this patient.
Answer:

379. Fracture (mid-palatal) of denture


- 4 patient related factors leading to it
- 4 dentist related factors leading to it
- Give 2 ways of overcoming this fracture
Answer:

380. Picture of 14 yr old girl with erosion of Buccal surfaces of molar


- 3 types of non carious tooth loss
- What do you think this is
- What disorder would you expect in this
- 4 ways in which you will monitor tooth wear
- What is the most important advice that you will give in this?
Answer:
A.3 non carious lesions:abrasion,attrition,erosion
b. it is bullimia or anorexia nervosa-since they have clearly mentioned that it is a
disorder
c.4 ways to monitor tooth wear:
study models, photos, clinical picture and history, tooth wear indices
d.Management: Referral to medical care, psychosocial interventions, nutritional
counseling

381.Which fibers are most resistant to periodontitis?


a. Transeptal fibers
b. Intergingival
c. Circular
d. Dentogingival
Answer: A

382. If the root filled tooth has no symptoms. When do you take the first xray for the follow up. Is it
6,12 or 18monts?
Answer: 12 months

383. A dentist is carrying out electro surgery on a patient and patient sneezes with his head moving
forward. Which is most likely to occur?
a GINGIVAL TRAUMA
b mucosal burn
c mucosal trauma
d. trauma to adjacent tooth
e. gingival laceration

Answer: c

384. Bilateral swellings not associated with food smell associated with lymphadenopathy?   
Answer: Sarcoidosis

385. How is acetyl choline released?


Answer: exocytosis

386. What age the sphenoid and mastoid fontanels are closed?

Answer:
 The posterior fontanel usually closes about two months after birth;
 sphenoid fontanel closes at about three months, the mastoid fontanel closes near

the end of the first year,


 Anterior one may not close until the middle
or end of the second year.

387. .A dentist is using soflex disc on an upper molar and lacks finger support. What is most likely
to occur?
a GINGIVAL TRAUMA
b mucosal burn
c mucosal trauma
d taruma to adjacent tooth
e gingival laceration

Answer: E

388. What hormone increases blood glucose and potassium?


Answer: Cortisol 

389. What antibiotic is used to manage a super infected herpetic lip lesion?
a.Ciprofloxacin
b.Metronidazole
c.Cefuroxime

Answer: c

390. Most abundant immunoglobulin in saliva during infections?


Answer: IgA

391. Consent is needed from a patient to share information, which of the following needs explicit
consent?
a.To share information with an insurance company
b.To share information with other doctors in the practice
c.To share information with the therapist treating the patient
d.To share information with the patients GMP
Answer: A

392. Which immunoglobin will increase in gingival inflammation?


Answer: IgG

394. Which immunoglobulin increase in periodontitis?


Answer: IgG

395. These phonetics are in relation to complete denture phonetics.


Answer:
 Mechanism of speech
 The voice is produced in the larynx: the muscles of the thorax and abdomen control the
flow of the air with nasal cavity act as resonant chamber 
 The air from the larynx divided into 2 streams by the velum:
a) Upper stream: the air expelled entirely through the nose to produce the nasal sounds: N-
M-Ng.
b) Lower stream: the air expelled through the oral cavity and altered by the palate, tongue
and position of the teeth and lips to produce all other sounds.

 Types of sound:
I. Vowels: 
 Produced by vibration of the vocal cords and not affected by oral structures. The tongue is
positioned in the floor of the mouth and contact lingual surface of anterior teeth. 
 Types: Vowels are: a-e-i-o-u.
II. Consonants: 
 Produced by constriction, obstruction and direction of the air stream when the air pass
through the mouth 
 Types: 
a) According to the manner of production:
1. Nasal sounds: N-M-Ng
 Produced through the nose. When the nasal cavity is blocked (adenoid hypertrophy-
deviated nasal septum), hypernasality occurs.

2. Plosives sounds: P-B-T-D-K


 Produced by complete stop of air stream, build up of pressure in the oral cavity then
sudden release and explosion of air
3. Sibilant (fricative) sounds: S-Ch-H-X-Z
 Produced by friction of the air stream when forced through narrow path way
b) According to the site of production:
1. Bilabial sounds: B-P-M
 Formed by lip only. The air from the lung builds up pressure behind the closed lip,
explosion produced when the lip suddenly opened
2. Labiodentals sounds: F-V
 Formed by lips and teeth. Produced by the contact between the upper incisors and the
lower lip
3. Lingudental sounds: Th
 Formed when the tip of the tongue is positioned between upper and lower incisors
4. Lingualveolar sounds: 
a. Tongue and the anterior portion of the hard palate: S, T-D
 S: the tongue form a slit like channel into which the air hisses and the air escape from the
median grove of the tongue when it is positioned behind maxillary incisors. If this groove is
flattened, lisping occurs (S is pronounced Ch), and if the groove is deepened whistling
occurs 
 T-D: the sided of the tongue contact the teeth, the air stops and sudden release (explode) 
b. Tongue and the intermediate portion of the hard palate: Sh-Ch-J
 The tongue is pressed against large area of the hard palate and alveolar process
c. Back of the tongue and soft palate: K,G
5. Nasal sounds: N-M-Ng 

Effect of complete denture on speech (prosthetic factors affecting speech): 


A. Denture base:
1- Denture base thickness:
Thin well adapted denture base (1mm thickness) not greatly affect the speech 
 Increasing the thickness of the denture base leading to cramping of denture space,
decrease air volume and obstruction air channels 
 Thickening the denture base in the anterior palatal → lisping (S pronounced Ch), and T
pronounced D
 Thickening the denture base in posterior palatal border → defect in vowels (e,i) and
consonants (k,g), so the border should be smooth tapered and merge with the soft palate (not
form a square edge)
 Thickening the denture base at lower lingual flange → cramping of tongue space →
lisping
 Decreasing the thickness of the denture base → whistling, D pronounced T

2- Extension of the denture base


 Proper extension of the denture flanges aid retention and stability of denture which help in
proper articulation of sounds as with poor retention, the tongue try to reseat the denture
against the palate during the speech.
 Avoid overextension of the flanges to decrease interference with muscle movement
during speech → indistinct speech especially if the lip affected

3- Polished surface
 Reproduction of incisive papilla and rugea area (by wax carving –tin foil) on the polished surface of the
anterior palate aid in correct production of anterior palatal sounds. 

B. Denture relations:
1- Occlusal plane
 Too high occlusal plane: tongue spread on the lower teeth→ lisping (S pronounced Ch), and F pronounced V
 Too low occlusal plane: difficulty in correct positioning of the lower lip and tongue contact occlusal surface
during the speech → V pronounced F
2- Vertical dimension
 Increased vertical dimension: denture teeth make contact during speech→ clicking, defect in Ch-C-J sounds,
whistling, Th pronounced T due to failure of the tongue to be placed between anterior teeth 
 Decreased vertical dimension: leading to lisping (S pronounced Ch)
 M sound: used as an aid to obtain correct vertical dimension. When the patient say M, if the lips are
straightened and unable to make contact, the record blocks are occluded prematurely and the VD is high 
 S sound: also used as an aid to obtain correct vertical dimension. When the patient say S (sixty-six), the
upper and lower teeth should be separated 2mm from each other (closest speaking space method)
3- Teeth arrangements: 
1- Width of the dental arch: 
 Too narrow dental arches→ the tongue cramped and the size of air channel decreased → faulty articulation
of consonants (T-D-N-K-C), therefore, the teeth should be placed in the position previously occupied by natural
teeth 
2- Antro-posterior position of the anterior teeth
 Upper anterior teeth
 Too far palatally:
- Upper incisors difficult to contact the upper lip → affect labiodentals sounds (F-V)
- Tongue make contact with the teeth prematurely → affect lingupalatal sounds→ lisping (S pronounced Ch), T
pronounced D
 Too far labially: whistling and D pronounced T
 Lower anterior teeth:
 Too far lingually: Th pronounced T and the tongue rested in the floor of the mouth behind lower anterior teeth
in pronunciation of vowels
 Too far labially: affect pronunciation of vowels.
3- The relationship of upper and lower anterior teeth
 Abnormal protrusive or retrusive Jaw relations (class II, class III angle classification) associated with increase
or decrease the overjet leading to difficulty in pronunciation of S sound (increase overjet→ whistling) 

396.which of the following gypsum products is commonly used for pouring up the primary dental
impressions......

a.TYPE I
b.TYPE II
c.TYPE III
d.TYPE IV
e.TYPE V

Answer: B

397. Most reliable test to indicate the presence of Active Hepatitis -


a. HBs antigen
b. HBc antigen
c. HBe antigen
d. Antibody to HBs antigen
Answer: C

398. -What vessels are the capacitance vessels and hold most of the blood volume? a. Arteries

b. Arterioles

c. Capillaries
d. Veinules

e. Veins

Answer: C (have 5% of blood volume always.) or (systemic veins carry most of blood volume)

399. What enzyme is found in dentinal caries? Lactoferrin, collagenase, or enolase?


Answer: enolase
400. Blood supply interferes in salivation. How? Sympathetic, parasymphatetic, etc(Type of
stimulation on blood vessels in salivary glands)
Answer: increase in salivary glands secretion is a result of parasympathetic and sympathetic
stimuli.
Sympathetic nervous system affects s.gland secretions inversly by innervating blood vessels that
supply gland.

401. Lots of neurophysiology (even alpha, beta, gamma, delta something), do you know what they
mean?
Answer: alpha, beta, Gamma, DELTA,B,C are nerve fibres which are myelinated except c which is
non myelinated.

402. What’s increased in elderly? Systole, diastole, stroke volume, etc


Answer: Diastole

403. Function of intermediate zone of adrenal Respiratory


Answer:

404. How you describe saliva in relation to plasma?-options, hypotonic, isotonic, hypermolar?

Answer: Hypotonic

405. physiology in cardiac arrest, angina??/


Answer: angina and MI are common causes of IHD
Angina pectoris is caused by an imbalance b/w myocardial oxygen requirements and myocardial
oxygen supply.
pain of angina is due to metabolic changes in ischemic myocardium.chemicals like
adenosine,histamine release 4rm ischemic cells which act on intracardiac sympthateic nerves which
goes 2 cardiac plexus nd symp.ganglia.impulses den transmitted throu spinal chord to thalamus nd
cerebral cortex.
myocardial oxygen req.increases as workload of heart inc.if coronary arteries are unable 2 deliver
req.o2 myocardial ischemia inc. leadin 2 MI.

406. Irreversible shock

407. what increases pulse rate during sex? Answer: Adrenaline


408. What neurotransmitter is responsible for closing pre capillary sphincters?

a.Acetylcholine

b.Serotonin

c.Substance p

d.Nor-adrenaline
Answer: D
409. What hormone increases blood glucose and potassium?
Answer: Cortisol

410. .best restoration in a missing incisor in a healthy caries free mouth?


Answer:

411. Radiation burns in cheek, what will u find intraorally?


Answer:

412. Antibiotic prophylaxis required in which case?


Answer: renal dialysis, immune compromised case.

413. antibiotic given in chronic sinusitis .


Answer: Amoxicillin/erythromycin/cephalosporin plus metronidazole

414. which drug u give to a Patient coming 2 days after extraction of 3rd molar with 2 extraoral
draining sinuses?
Answer: Metronidazole 400mg 3 time for 5 days

415. What s the nerve block for upper 4?


Answer: Infra-orbital nerve block

416. What cement for cementing temporary bridge?


Answer: Temp bond...ie zinc oxide eugenol

417. What s the best restoration for a peg shaped lateral?


Answer: composite

418. What should be added to beta lactams to be more efficient against anaerobes?

419. what does fracture of the 2 condyles cause?


Answer: Anterior open bite

420. What s the 2ry impression material for resorbed ridge?


Answer: Zinc oxide eugenol paste
421. What material should be used in a lingual cavity in a patient with Parkinson's disease?
Answer: GIC

422. What anaesthesia should be given in a flap for upper lateral Apicectomy?
Answer: infraorbital nerve

423. .You arrive at a new practice and notice that almost every radiograph in the
patient’s notes has turned brown. Your nurse confirms that this is a widespread
problem that no-one has ever remedied and she also remarks that the films tend
to get browner with age.

What corrective action will you take to remedy the problem?


A. Develop the films for the correct period of time
B. Heat the developer to a higher temperature
C. Ensure films are fixed for the correct period of time
D. Wash the films properly after fixing
E. Change the solutions more regularly
answer: D

424. A 10½ year old boy with an uncrowded sound dentition attends your practice.
His permanent canines are unerupted and not palpable and primary canines are retained. On
radiographic examination, you find the canines are mesially inclined and in the line of the arch.

What is the most appropriate management option?


A. Arrange surgical removal of permanent canines.
B. Extract primary canines.
C. Extract upper first premolars.
D. Grind primary canines.
E. Monitor occlusal development.
answer: E

425. What gives the black stein to amalgam tattoo? Silver or mercury?
Answer: Silver

426. A patient says that he does not like the appearance of his previously root
filled upper central incisor tooth. His dentition is otherwise well maintained
and his periodontal health is good. The tooth appears to be darker than the
adjacent teeth.
What is the most appropriate approach to treatment?
A. Provision of a post crown
B. Provision of an all ceramic crown
C. Provision of a metal bonded to ceramic crown
D. Carry out a non vital bleaching procedure
E. Provision of a porcelain veneer
Answer: D
427. A 35 year old male patient who admits to grinding his teeth at night has a
number of wedge-shaped cervical (Class V) lesions on his upper premolar teeth.
These are causing some sensitivity and are approximately 3mm deep.
What is the correct management option?
A. Provide tooth brushing instruction and fluoride
B. Restore the lesions with compomer
C. Restore the lesions with micro-filled composite
D. Restore the lesions with a hybrid composite
E. Restore the lesions with conventional glass-ionomer
Answer: D

428. You are trying in a partial chrome denture framework which fails to seat
properly. It fits the master cast.
What is the most likely cause of this problem?
A. Insufficient expansion of the investment material
B. Distortion of the impression
C. Contraction of the metal framework during casting
D. Failure to block out unwanted undercuts
E. Complex denture design
Answer: b

429. Bitewing radiography is the main special text used to help in diagnosis of
proximal caries. The performance (accuracy) of a diagnostic test like bitewing
radiography can be expressed in terms of sensitivity and specificity.
Which of the following is a reasonable summary of the diagnostic accuracy of
bitewing radiography for proximal caries diagnosis?
A. Moderate sensitivity and low specificity
B. Moderate sensitivity and moderate specificity
C. Moderate sensitivity and high specificity
D. High sensitivity and moderate specificity
E. High sensitivity and high specificity
Answer: E

430. Types of cells that proliferate in later stages of life


a Odontoblast
b Cementoblast
c Rest of Malassez
d Undifferentiaied Mesenchymal Cells

Answer: D

431. Dentist did RCT for child patient, now he does Crown for the same patient. What band the
dentist should claim?
a Band 1
b Band 2
c Band 3
d No Claim

Answer: C

432. A patient with a removable partial denture is dissatisfied with the false appearance of the
mandibular anterior teeth. The dentist could CORRECT this appearance by

I.Moving the teeth farther lingually so that they are not so obvious
II.Varying the inclinations of the incisors so that alternate teeth appear tilted
III.Moving the teeth farther facially so that their appearance is enhanced
IV.Separating the teeth slightly to make each one look distinct
Choose one answer.
a. III only
b. Any of these options
c. II, III
d. II, IV Correct
e. I, III
Answer: E

433. A PRIMARY consideration for full coverage of abutment teeth to be used in the construction
of a fixed partial denture is the
Choose one answer.
a. Health of the mucous membrane
b. Caries susceptibility of the patient
c. Amount of supporting bone remaining
d. Health of the gingival tissues
e. None of these options
Answer: E

434. What is the % of de oxygenated blood coming from lungs?

Answer: 75%

Explanation: Oxygen transport


Basic hemoglobin saturation curve. It is moved to the right in higher acidity (more dissolved carbon
dioxide) and to the left in lower acidity (less dissolved carbon dioxide)

About 98.5% of the oxygen in a sample of arterial blood in a healthy human breathing air at sea-
level pressure is chemically combined with the Hgb. About 1.5% is physically dissolved in the other
blood liquids and not connected to Hgb. The hemoglobin molecule is the primary transporter of
oxygen in mammals and many other species (for exceptions, see below). Hemoglobin has an
oxygen binding capacity of between 1.36 and 1.37 ml O2 per gram Hemoglobin,[14] which increases
the total blood oxygen capacity seventyfold,[15] compared to if oxygen solely was carried by its
solubility of 0.03 mL O2 per liter blood per mmHg partial pressure of oxygen (approximately 100
mmHg in arteries).[15]

With the exception of pulmonary and umbilical arteries and their corresponding veins, arteries carry
oxygenated blood away from the heart and deliver it to the body via arterioles and capillaries, where
the oxygen is consumed; afterwards, venules, and veins carry deoxygenated blood back to the heart.

Under normal conditions in adult humans at rest; hemoglobin in blood leaving the lungs is about
98–99% saturated with oxygen, achieving an oxygen delivery of between 950 - 1150 mL/min[16] to
the body. In a healthy adult at rest, oxygen consumption is approximately 200 - 250 mL/min ,[16]
and deoxygenated blood returning to the lungs is still approximately 75% (70 to 78%)[16] saturated.
Increased oxygen consumption during sustained exercise reduces the oxygen saturation of venous
blood, which can reach less than 15% in a trained athlete; although breathing rate and blood flow
increase to compensate, oxygen saturation in arterial blood can drop to 95% or less under these
conditions.[19] Oxygen saturation this low is considered dangerous in an individual at rest (for
instance, during surgery under anesthesia. Sustained hypoxia (oxygenation of less than 90%), is
dangerous to health, and severe hypoxia (saturations of less than 30%) may be rapidly fatal.[20]

A fetus, receiving oxygen via the placenta, is exposed to much lower oxygen pressures (about 21%
of the level found in an adult's lungs), and, so, fetuses produce another form of hemoglobin with a
much higher affinity for oxygen (hemoglobin F) in order to function under these conditions.[21]

Carbon dioxide transport


When blood flows through capillaries, carbon dioxide diffuses from the tissues into the blood. Some
carbon dioxide is dissolved in the blood. A part of CO2 reacts with hemoglobin and other proteins to
form carbamino compounds. The remaining carbon dioxide is converted to bicarbonate and
hydrogen ions through the action of RBC carbonic anhydrase. Most carbon dioxide is transported
through the blood in the form of bicarbonate ions.

Carbon dioxide (CO2), the main cellular waste product is carried in blood mainly dissolved in
plasma, in equilibrium with bicarbonate (HCO3-) and carbonic acid (H2CO3). 86–90% of CO2 in the
body is converted into carbonic acid, which can quickly turn into bicarbonate, the chemical
equilibrium being important in the pH buffering of plasma.[22] Blood pH is kept in a narrow range
(pH between 7.35 and 7.45).[9]

435. How would you treat a person with TB, HIV, hepatitis , MRSA
 send them to the hospital
 use single use instruments
 treat with sterile instruments
 treat using standard condition for infection control
Answer: treat using standard condition for infection control

436. .Type of Hepatitis that doesn’t become chronic – Hep A?

Answer: Hep A and E

437. Which fluoride varnish aggravates effects of ANUG

Answer:

438. Which Antifungal should not be given with Diazepam

Answer: fluconazole

439. Upper missing Laterals and Pointed Canines, gaps between centrals and canines. What would
be the best Rx options

a Orthodontically move the Canines

b Give bridges

c Crowns on Canines

Answer: A

440. Upper denture loose. A root beneath the lower CD. Which X-ray will you take?

Answer: Periapical
441. Condition most commonly affectedby Dry Socket

a. Smoker

b. Pt. On warfarin

c. Elderly

Answer: A

442. QUESTION 1
A. COPPER
B. PALLADIUM
C. PLATINUM
D. SILVER
E. ZINC
CHOOSE FROM THE ABOVE FOR THE FOLLOWING:

1. INCREASED TARNISH RESISTANCE


2. SCAVENGER, PREVENTS OXODATION OF OTHER METALS
3. INCREASED STRENGTH AND HARDNESS
4. DECREASED DENSITY AND MELTING POINT
5. INCREASED DENSITY AND MELTING POINT
6. INCREASED CORROSION
7. INCREASED POROSITY

Answer:
Copper increases strength and hardness
Palladium increased density and melting point
Silver increase tarnish resistance/ Corrosion resistance
platinum is added as scavenger
Zinc is used at times for the same purpose of scavenging

443. The most important feature to differentiate between an upper neuron motor
lesion and a lower neuron motor lesion is:
a- eye involvement
b- ear involvement
c- forehead involvement
d- anaesthesia of the facial nerve

Answer: C
Explanation: it is because in upper motor neuron lesions, there’s no affect on voluntary control of
muscles of forehead due 2 bilateral innervations of portion of motor nucleus of cranial nerve vii that
innervates upper muscle of facial expression.

444. What are D1 D2 D3 lesions?


Answer:
That’s the WHO classification System: The caries lesion shape and depth is scored in a 4 points
scale.

D1: Clinically detectable enamel lesions with intact (non-cavitated) surfaces.

D2: Clinically detectable cavity limited to enamel

D3: Clinically detectable cavity involving dentin

D4: Lesions extending into pulp

445. A PARKINSON DISEASE PATIENT COMES TO YOUR SURGERY AND WITH WHAT
MATERIAL WILL YOU RESTORE A LINGUAL CAVITY IN HIM?
Answer: Parkinson dis : glass ionomer as they suffer from dry mouth and fluride releases from GIC

446. PARAESTHESIA OF LINGUAL N. LASTS FOR HOW MANY MONTHS?


Answer: 2-3 months

447. Which part of the cranium is considered as the most stable area,
A. Frankfort plane
B. Occlusal plane
C. Anterior cranial plane
D.Anterior nasal to gnathion
Answer:

448. .A 9 yr old child requires extraction of their upper right first permanent molar under local
anesthesia. Her medical history is unremarkable except that she has had rheumatic fever at 3 yrs of
age and a chest infection 4 months ago, which was treated with penicillin. What is the correct
precaution for this child?

449. Appointment for asthmatic patient should be given in:


1. Morning.
2. Afternoon.
3. Evening.
4. Any time.
Answer: Afternoon.

450. Chi square test is:


1. Measures qualitative data.
2. Measures both qualitative and quantitative data.
3. Measures the qualitative data between two proportions.
4. Measures the quantitative data between two proportions
Answer: 4
Chi-square is a statistical test commonly used to compare observed data with data we would expect
to obtain according to a specific hypothesis. For example, if, according to Mendel's laws, you
expected 10 of 20 offspring from a cross to be male and the actual observed number was 8 males,
then you might want to know about the "goodness to fit" between the observed and expected. Were
the deviations (differences between observed and expected) the result of chance, or were they due to
other factors. How much deviation can occur before you, the investigator, must conclude that
something other than chance is at work, causing the observed to differ from the expected. The chi-
square test is always testing what scientists call the null hypothesis, which states that there is no
significant difference between the expected and observed result. 

451. Radio graphically lingual developmental groove is seen as:


1. Blurring of root canal.
2. Small pulp chamber size and constriction of root canal compare to contralateral tooth.
3. Constriction of pulp canal only.
4. Large pulp chamber size and constriction compare to ipsilateral tooth.
Answer:

452.   1.  A 15 year old girl in boarding school comes to your surgery for an extraction which of the
following cannot give consent on her behalf?
a. Her grandfather with legal guardianship
b. Her biological father who has separated from her mother
c. The girl herself
d. Her older sister
Answer: C

453. Consent is needed from a patient to share information, which of the following needs explicit
consent?
a. To share information with an insurance company
b. To share information with other doctors in the practice
c. To share information with the therapist treating the patient
d. To share information with the patients GMP     
Answer: A

454.    EMQ about muscle fibres’-


a. what is the A band made of-Myosin
b. what combines with calcium- Proporin
c. what protein has I,T and c subcomponents.-????

455.  What enzyme assists microorganisms when they are causing dentine caries?
a.       Collagenase
b.      Enolase
c.       Latoferin
Answer: C

456. What neurotransmitter is responsible for closing pre capillary sphincters?


          a.       Acetylcholine
          b.      Serotonin
          c.       Nor-adrenaline
          d.      Substance p

457.    What antibiotic is used to manage a super infected herpetic lip lesion?
   a.       Ciprofloxacin
   b.      Metronidazol
   c.       Cefuroxime
 Answer:C
           
458.  What component of amalgam gives strength- 
a. Copper    
b. Silver
Answer:     A                                                                                                                                                     
                   
459.    Picture of the tongue with an ulcer on the lateral border- what drug causes it
a. Ace inhibitors                                                                                                                                      
b.  Calcium channel blockers
c. Potassium sparing drugs
Answer:     A

460. 1. Fluoride treatment for a child with high caries rate


a. 2,800 ppm Tooth paste
b.   5,000 ppm toothpaste
c.   Application of 2.2% duraphat 2 time yearly
d.  Application of 2.2% duraphat 3-4 times yearly

Answer:   D  

461.   Average number of 15 year olds who have lost 6-7 teeth
a. 1.1
b. 3.1
c. 4.7
Answer:     

462. A man who smokes 2 packets of cigarettes a day and drinks would have an increase risk of
developing oral cancer of
a. 11 times
b. 13 times
c. 47 times
Answer:     C

463. A picture of the gingival of a black person with melanin pigmentation and a white line across
the central incisors alone
1.       What is the cause of the white line
a.       Amelogenesis imperfecta
b.      Dentinogenesis imperfecta
c.       Hypo mineralization
d.      Flourosis
Answer:    C

464.  64-year-old patient with a past medical history of metastatic breast cancer presents to your
surgery complaining of loose teeth, dental pain and swollen gums.

A. What appearance is shown in the above X-ray?

B. What is the most likely causative agent?

C. How would you manage this patient?

Answer:

A. Osteonecrosis of the left mandible.

B. Long-term use of bisphosphonates to treat hypercalcaemia secondary to metastastic disease.

C. Polite introduction. Take a history. Ask about trauma. Take detailed personal past medical
history (PMH) regarding breast cancer diagnosis and subsequent management and length of
bisphosphonate use. Diagnosis history:

 examination further investigations – CT scan, inflammatory markers


 treatment iv antibiotics

liaise with oncology team concerning bisphosphonate cessation.


465. which of the following is derived entirely from meckel cartilage ..? incus , malleus. stapes,
mandible.
Answer: Malleus

Explnation:

Meckel's cartilage

Meckel's cartilage forms in the mesoderm of the mandibular process and eventually regresses to
form the incus and malleus of the middle ear; the anterior ligament of the malleus and the
sphenomandibular ligament. The mandible or lower jaw forms by perichondral ossification using
Meckel's cartilage as a 'template', but the mandible does not arise from direct ossification of
Meckel's cartilage.

466. Which antibiotic causes abdominal cramps in therapeutic doses?


Answer: clindamycin

467. Which antibiotic causes abdominal cramps in toxic doses?


Answer: cephalosporins

468.
1-carious haif away thorogh enamel on proximal surface - bitewing
2-carious half away through enamel on occlusal surface – visual exam
3-carious half way through dentine not cavited - wet surface,
4- stained fissure – bite wing and visual exam
5-hidden/occlut caries – bitewing and beam aiming devices
choice (traslumination, bitewing, dry surface, wet surface, briuat prop wilames prob, stright sharp
prob

469. .the cells in cementum that line its boundary with PDL with cytoplasmic processes directed
towards cementum?

2.cells that line the cemental boader with cytoplasmic processes directed towards PDL ?

3) pt. came from holiday and have diarrhea and vomiting and claim that he has eaten something
from street?
 Hep A

4 )which condition that phagocyte kill bacteria is more difficult(tbc etc.)

6)severly disabled pt sedation – GA

471.
1. %of edentulous pts in England nd Scotland
2. %of people with decay b/w 10-12 yrs old in 2003

4.Antibiotic used to manage superinfected herpetic lip cefuroxime or metronidazole?

5.%of bacterial load that will decrease by cleaning

6.condition that prevents to maintain oral hygiene


Answer: down syndrome

7. % of children getting cavities after pit nd fissure sealant application

8.what to do when a pt comes with acute pain the very next day he had been discharged after
myocardial infarction

9. Three weeks ulcer present after denture insertion what to do? Trim and observe or refer

10. Maternity leave allowed – 6 weeks

472. suture material used on lips?


Answer: prolene, nylon....

2.suture material for use in oroantral fistula?


Answer: vicryl

473. Which ion acts as a second messenger?


Asnwer; Calcium.

474. most common reason of failure of approximal amalgam filling?

FAILURES OF AMALGAM
FRACTURE OF THE RESTORATION

CAUSE EFFECT
Too shallow cavity
Too thick cavity liner Amalgam thin
Too thick cement base
Inadequate cuspal reduction
Amalgam thin (over the cusp)
Giving cavosurface bevel
Sharp axio-pulpal line angle
Stress concentration (fracture thro'
Sharp angles in occlusal outline form of isthmus)
Class II
Sloping gingival step No resistance form (fracture thro'
Too narrow gingival step isthmus)
Insufficient Hg
Excess Hg Amalgam weak
Undertrituration
Moisture contamination (Zn having
Delayed expansion (flow over margins)
alloys)
Insufficient condensation pressure
Increased residual Hg
Not squeezing out excess Hg
Mix squeezed too dry
Condensation of partially crystallized Lack of cohesion (amalgam weak)
amalgam
Overfilling Thin amalgam over margins
Failure to warn patient not to chew on the
restoration for first few hours
Early strength not high enough
Failure to support proximal part of the
restoration while removing matrix band

FRACTURE OF THE TOOTH

CAUSE EFFECT
Sharp angles in occlusal outline of Class
Stress concentration
II
Excess removal of tooth structure Enamel undermined and tooth weakened
INADEQUATE RETENTION

CAUSE EFFECT
Absence of undercuts No retention form
Dovetail with only one cornu No resistance to dislodgement
Too thick liner that is lost subsequently Lack of adaptation to cavity walls
Undercuts and margins not filled - No
Using large condensers initially
retention

MARGINAL LEAKAGE

CAUSE EFFECT
Excess Sn (tin) in the alloy Too much shrinkage
Overtrituration Shrinkage on setting
Excessive pestle pressure Slow setting with shrinkage
Failure to condense towards margins Marginal gaps
Using large condensers initially Deficient margins and undercuts
Carving from amalgam to tooth Marginal defects and gaps
Excess amalgam left beyond cavosurface
Breaks away leaving deficient margins
angles

POROUS AND WEAK AMALGAM

CAUSE EFFECT
Increased γ2 Phase (low Cu alloys) Weak phase
Porosities and voids; less coherence of
Irregularly shaped particles in the alloy
phases
Same as above (Incomplete
Too less Hg
amalgamation; non-plastic)
Increased residual Hg (Increased γ2 Phase
Too much Hg
, decreased γ1 Phase phases)
Undertrituration Porosities & voids
Decreased coherence (due to cracking of
Trituration beyond limits
crystals)
Delayed insertion after trituration
Porosities and voids
Insertion of too large increments
Porosities and voids (increased residual
Decreased condensation pressure
Hg)
Moisture contamination Porous amalgam
Mix squeezed too dry Decreased coherence
Mix not squeezed (with high Hg:Alloy Porosities and voids (increased residual
ratio) Hg)
Condensation of partially crystallized
Porous amalgam
amalgam
Condensing with serrated pluggers with Old amalgam contaminates restoration
set amalgam in the serrations and weakens it
'Burns' amalgam and releases Hg
Overheating while polishing
resulting in porosity
Breaks up superficial crystalline structure
Burnishing set amalgam
releasing Hg causing porosity

TARNISH AND CORROSION

CAUSE EFFECT
Alloy with excess γ2 Phase Has least resistance to corrosion
Food stagnation leading to tarnish and
Fissures carved too deep
corrosion
Rough surface causing crevicular
Failure to polish
corrosion
Contact with dissimilar metallic
Galvanic corrosion
restoration

GINGIVITIS AND PERIODONTITIS

CAUSE EFFECT
No wedge used Gross overhang; Contact area deficiency
Surface left high in bite High point causing periodontitis
Failure to polish proximal surface Food stagnation resulting in gingivitis
Lack of proximal contact and periodonitis

LACK OF FUNCTIONAL EFFICIENCY

CAUSE EFFECT
Fissures carved too deep
Reduced masticatory efficiency
Underfilling
Failure to carve Decreased masticatory efficiency (Tooth
anatomy not simulated).

PAIN AFTER PLACING RESTORATION

CAUSE EFFECT
Failure to use liner and base Thermal conduction
High point causing periodontitis resulting
Overfilling
in pain
Moisture contamination Delayed expansion with pressure on pulp
Mercuroscopic expansion with pressure
Increased Hg: alloy ratio
on pulp
Failure to squeeze out excess Hg Mercuroscopic expansion (increased
Inadequate condensation pressure residual Hg)
Cavity preparation without water coolant
Pulpitis resulting in pain
Microscopic pulp exposure

TOOTH DISCOLOURATION

Sometimes excess Hg within the restoration may seep through the dentinal tubules, discolour dentin
and result in blackish or greyish staining of teeth. Since enamel is semi-translucent, this
discolouration is not inconspicuous.

475. Who checks if there is written protocol put up in the clinic for radiation protection
Answer: written protocol set by legal person monitored by radiation protection supervisor

476. Whom to contact if there is 20s of extra radiation dose


Answer. Radiation protection advisor

477. Most common reason of failure of approximal amalgam filling?


Answer. Isthmus fracture

478. what is more likely to happen to an interproximal composite filling rather than amalgam it's
feature or under contouring?
Answer. Polymerization shrinkage

479. Largest possible cavosurface angle?


Answer. 110 degree

480. What cement you use to glue alumina core ceramic crown? it's panavia or resin modified gic?
Answer. Resin modified gic

481. Pigmented fissure, which is the best way of caries diagnosis?


Answer. Investigate the area with round bur

483. What is the sensory nerve supply to temporalis muscle?


Answer. The Sensory and motor are both supply by the V3 , (Sensory by Auriculotemporal branch
of V3 and motor by ant. and post. deep temporal branch of V3).

484. What’s the percentage of >25 years with periodontitis in UK


Answer. 64%

485. What’s the percentage of tooth erosion in 6 years-old and 15 years-old in UK?
Answer. 5-52% and 15--27%

486. Do you know what enzyme assists micro-organisms in dentin caries process?


I'm in doubt between Enolase and Collagenase.

Answer. Enolase
Enolase, also known as phosphopyruvate hydratase, is a metalloenzyme responsible for the
catalysis of the conversion of 2-phosphoglycerate (2-PG) tophosphoenolpyruvate (PEP), the ninth
and penultimate step of glycolysis. Enolase belongs to the class Lyase. Enolase can also catalyze the
reverse reaction, depending on environmental concentrations of substrates. [3] The optimum pH for
this enzyme is 6.5.[4] Enolase is present in all tissues and organisms capable of glycolysis
or fermentation. The enzyme was discovered by Lohmann and Meyerhof in 1934,[5] and has since
been isolated from a variety of sources including human muscle and erythrocytes.[4]
Fluoride is a known competitor of enolase’s substrate 2-PG. The fluoride is part of a complex with
magnesium and phosphate, which binds in the active site instead of 2-PG. [4] As such,
drinkingfluoridated water provides fluoride at a level that inhibits oral bacteria enolase activity
without harming humans. Disruption of the bacteria’s glycolytic pathway - and, thus, its normal
metabolic functioning - prevents dental caries from forming.
http://en.wikipedia.org/wiki/Enolase

487. % of leukocytes required for extraction to be carried out?


Answer. 50X10^9

489. Which kind of prevention are these?

1. Scaling and tooth polishing - secondary prevention


2.curettage – tertiary
3.deep curettage - tertiary
4.correction of tooth malalignment - secondary
options(primary,secondary.tertiary?

Levels of Prevention (Dentistry)


• Primary – Prenatal counseling, infant oral health program, pre-disease tooth brushing/flossing,
routine fluoride use, etc.

• Secondary – Remineralization of enamel caries, restoration of carious teeth, treatment of


gingivitis, correction of malocclusions, etc.

• Tertiary – Dental prosthesis, implants, surgical reconstruction following trauma, periodontitis,


cancer, etc.
PRIMARY
1.health education
2.plaque control advice
3.diet counseling
4.all forms of flouride
5.caries activity test
6.pit and fissure sealants
7.prophylactic odontomy
8.daily brushing and flossing
9.mouthguards in contact sports
10.supervised brushing

SECONDARY
1.periodic screening of oral disese
2.PRR
3.S&P
4.simple restorative procedures- eg amalgam, pulp capping

TERTIARY
1.pulpotomy
2.rct
3.Ext
4.rpd fpd implants
5. Minor tooth mvmts
6.deep curettage root planing
7.splinting
8.perio surgery

490. .First line of treatment of a pt with lower lateral extracted while the alveolus heals to cover the
gap?
1. Conventional cantilever,
2. Partial denture.
Answer. Partial denture.

491. Which is the most common jaww cyst in UK?


Answer. Radicular cyst (75%)

492.
1. What time do you give BCG vaccine -
2. What time do you give MMR vaccine
BCG at 13years
MMR at 1 and 4 years

United Kingdom Vaccine Schedule: 2010[24]

2 12 3–4 12–13
Vaccine 3 mo. 4 mo. 13–18 yrs
mo. mo. yrs yrs

Diphtheria, pertussis, and


DTaP DTaP DTaP DTaP Td
tetanus

Haemophilus influenzae type


Hib Hib Hib Hib
b

Pneumococcal PCV PCV PCV

Polio vaccine (inactivated) IPV IPV IPV IPV IPV

Measles, mumps, and rubella MMR MMR

Meningococcus MenC MenC MenC

Human papillomavirus vaccine HPV1

1. ^ Females only

493.
pt 16 years old with bleeding gums and bad smell which antibiotic? - metronidazol
2.Patient with osteitis + pus after 3rd molar extraction which antibiotic?. - metronidazol
3. pt with pain after extraction:which analgesic – NSAID

494. Which nerve affected if tongue deviates to right when protruded?

A. Right hypoglossal
B. Left hypoglossal
C. Glossopharyngeal

Answer. A

495.
Which nerve is affected if patient is unable to gaze laterally to left?

A. Right abducens
B. Left abducens

Answer. B

496. What's the most frequent site of endocarditis?


In other patients without a history of intravenous exposure, endocarditis is more frequently left-
sided.
Patients who inject narcotics or other drugs intravenously may introduce infection which will travel
to the right side of the heart classically affecting the tricuspid valve, and most often caused by S.
aureus.

Aortic valve – 5-36%


Tricuspid – less than 10%
Bicuspid
Mitral – 28-45% (mitral and aortic both 0-35%)

497. Diet advice for a child. What's the most effective method?
A. tell the parents not to give sweets for the child,
B. dentist tells the child which are good and bad foods,
C. nurse talk to the child,
D. send the child to a dietician, etc

498. What protein has I, T and C subcomponents in muscle fibres?


Answer. Troponin

499. Cells found in herpetic stomatitis and lichen planus?

500. Patient who had epilepsy 10 years ago but now in control. Which tech will u use?

A. Inhalation sedation
B. IV sedation
C. Carry n with treatment

501. What enzyme assists the micro organism in causing dentine caries?
1. collagenase
2. enolase
3. lactoferrin
502. Is rheumatoid arthritis type 3 hypersensitivity ???
Yes
503. Which of the following is the most critical step in root canal therapy?
A.Cleaning and shaping.
B.Obturation.
C.A hermetic apical seal.
D. A hermetic coronal seal.
E. C and D.

504. At which angle to the external surface of proximal cavity walls in a class II preparation for
amalgam should be finished
A. An acute angle
B. An obtuse angle
C. A right angle
D. An angle of 45°

505. If the sealant of bonding agent is not placed on part of enamel that has been etched by an acid
solution; you would expect:

A. Arrest of enamel carries by organic sulphides


B. The enamel is to return to normal within 7 days
C. Continued enamel declassification in the etched area
D. Slight attrition of the opposing tooth

506. In the inferior alveolar block the needle goes through or close to which muscles:
A. Buccinator and superior constrictor
B. Medial and lateral pterygoid
C. Medial pterygoid and superior constrictor
D. Temporal and lateral pterygoid
E. Temporal and medial pterygoid

507. Which of the following may be caused by newly placed restoration which interferes with the
occlusion
A. Apical abscess
B. Pulpal necrosis
C. Apical periodontitis

508. Rank the following impressions materials according to their flexibility

A. Alginate> Polysulphide> Silicone> Zinc Oxide Eugenol


B. Silicone> Alginate> Polysulphide> Zinc Oxide Eugenol
C. Alginate> Polysulphide> Zinc Oxide Eugenol>Silicone
D. Alginate> Silicone> Polysulfide> Zinc Oxide Eugenol
E. Alginate> Zinc Oxide Eugenol> Silicone> Polysulphide
509. Mercury is dangerous when it turns into vapour form because of,
A. It is accumulative and cause liver poison
B. It is accumulative and cause kidney poison
C. It induces neoplasia in the liver
D. It is accumulative and cause brain poison
E. It induces neoplasia in the brain

510. implant

sometime back i posted this doubt, as to re implant tooth in cardiac patients and IF yes, do u
prescribe antibiotics.

luckily i got this question answered by. Dr. James, who was one of teachers for us on the DTP
session day, he was also a previous ORE examiner.

He said u should implant the tooth back or ask it to be put back even if its a cardiac patient . and
there is no need to give antibiotics now according to the recent guidelines, coz in cardiac patients it
was noted that even With antibiotics they carried the rsik of infection, so there was no point in
prescribing antibiotics to them, it was nt helping in anyways.

so, thou it is given in Odell as we read that DO NOT re implant avulsed tooth in cardiac patient ,
that is outdated , the above is the answer now.

511. WHAT FRACTURE OF THE JAW IN 8 YAER OLD WOULD AFFECT GROWTH?

Ans. the condyle fracture would affect the growth.

512. Do we see tight junctions in oral mucosa?

Ans. less frequently observed

513. FLUORIDE SUPPLEMENT FOR A 4 YEAR CHILD,WHEN WATER FLUORIDE LEVEL


IS LESS THAN 0.1?

Ans. Toothpaste 1350-1500ppm pea-size (Alternative - 0.5 mg)


Fluoride varnish 2.2%F twice yearly

514. A MAN HAS DIET CONTROLLED TYPE 2 DIABETES.HE CONSUMES 30 GLASSES


OF ALCOHOL AND SMOKES 40 CIGARETTES PER DAY.HE IS OTHERWISE
HEALTHY.WHAT MAY BE THE REASON OF DRY MOUTH ?
Ans. Dehydration

515. Alginate after impression if you put soaked gauzes on it what happens (contraction, expansion,
shrinkage)

516. Ideal rake angle required for the universal curette in subgingival curettage?
Ans. 70degrees

517. Which immunoglobin will increase in gingival inflammation?

Ans. IgG

518. Mercury is dangerous when it turns into vapour form because of,
A. It is accumulative and cause liver poison
B. It is accumulative and cause kidney poison
C. It induces neoplasia in the liver
D. It is accumulative and cause brain poison
E. It induces neoplasia in the brain
Ans. B

Summary of toxicology

Effects on Humans: Mercury vapor can cause effects in the central and peripheral nervous systems,
lungs, kidneys, skin and eyes in humans. It is also mutagenic and affects the immune system
[Hathaway et al. 1991; Clayton and Clayton 1981; Rom 1992]. Acute exposure to high
concentrations of mercury vapor causes severe respiratory damage, while chronic exposure to lower
levels is primarily associated with central nervous system damage [Hathaway et al. 1991]. Chronic
exposure to mercury is also associated with behavioral changes and alterations in peripheral nervous
system [ACGIH 1991]. Pulmonary effects of mercury vapor inhalation include diffuse interstitial
pneumonitis with profuse fibrinous exudation [Gosselin 1984]. Glomerular dysfunction and
proteinuria have been observed mercury exposed workers [ACGIH 1991]. Chronic mercury
exposure can cause discoloration of the cornea and lens, eyelid tremor and, rarely, disturbances of
vision and extraocular muscles [Grant 1986]. Delayed hypersensitivity reactions have been reported
in individuals exposed to mercury vapor [Clayton and Clayton 1981]. Mercury vapor is reported to
be mutagenic in humans, causing aneuploidy in lymphocytes of exposed workers [Hathaway et al.
1991].

519.
what is the percentage of plaque in the uk? - plaque...66%
what is the percentage of calculus in the uk? - calculus...69%
What % of population are allergic –

Statistics: 20-30% of people claim to have a food allergy, 5-8% of children and 1-2% of adults
have a food allergy.

Peanut allergy - 0.4-0.6%,

3% or 1.5 million of the British population who are at risk – latex allergy

What % of population are allergic in UK? – 30%


What % of bacterial load will decrease by cleaning

520. What bone lines alveolus?What happens to alveolus after extraction?

Ans. cortical bone (lamina dura)

521. Nerves that may be damaged during lower molar extraction? How to avoid?

Ans. lingual nerve and inf alveolar nerve

522. Extraction of lower first molar- nerves to be anaesthetised, after anaesthesia , pt can't close
eyes .. Reason?

Ans. injury to facial nerve

523. 3 year-old child, baby teeth to be extracted. You want to check permanent germs.Which
radiograph?
Panoramic,
periapical,
bimolar,
occlusal,
lateral skull view

Projection Function
   
DPT or Bimolar: Identification of the devlopeing dentition
  Confirmation of the presence/absence of teeth
  Preliminary assessment of caries, apical condition. periodontal state
   
Standard occlusal
Identification of abnormal pathology
view:
  To show presence of unerupted teeth
  Vertical parallax localisation either with DPT or periapical film
  To supplement bimolar film
   
Mandibular
Localisation of unerupted teeth
occlusal:
   
Periapicals: To assess root morphology
  To assess root resporption
  To assess apical pathology
In combination with a standard occlusal or second periapical to localise
 
unerupted teeth by horizontal parallax
   
Bitewings: To assesss teeth of dobtful prognosis
  Caries identification and periodontal bone levels
   
Lateral
To assess skeltal pattern and labial segment angulation
Cephalometric view:
  To aid assessment of unerupted teeth

524.

Which Xray is needed for implants? – CBCT

2.Diagnosis of interproximal caries, what would you use? - bitewing


3.What do you find in the caries tissue?
4.Bacteria that most commonly cause infection in digestive system in the UK? Helicobacter pylori
common bacteria in stomach lining and duodenum

5.Percentage of edentulous patients in England and Scotland.? - England 6% Scotland 12


6.Percentage of tooth loss (wear) in 2003? - 36% in 16 to 24, 89% in 65 and over
7.Percentage of people without decay between 10 to 12 years old in 2003? - 66.6%
8.% of 12 year old with caries? - 38%
9.%of 16 year old with non carious tooth loss? -
10.%of edentulous in the uk? – 13%

525. Which cells r seen in TB and RA? - T lymphocytes

2 A growing plaque like pigmented patch on thigh. What investigation is this? - purpura not very
sure of this ans

3 Which hepatitis doesnt become chronic? - Hepatitis A

4 Saliva drooling, swelling in the floor of mouth something wrong with finger. Wt diagnosis? -
Ranula but no clue abt this question related to fingers
5 Sharp shooting pain radiating to shoulders? -
6 Angioneurotic edema is characterized by which esterase inhibitor? - C1 levels low
C1 c2 c3 c4?

http://en.wikipedia.org/wiki/Angioedema
7. multiple lumps on neck, they are sensitive, and their skin is pigmented. Daignosis? - Chronic
myeloid leukaemia

526. someone who drinks 14 units in a week and smokes 30 cig a day has diagnosis of precancrous
lesion after biopsy of palate...wats da best action to reduce chances of malignancy?
(excise lesion ,evaporate by laser ,avoid to stop smoking, avoide to stop drinking)

Ans. stop smocking

527. In regards to carbide burs; the more number of cutting blades and low speed will result in:
A. Less efficient cutting and a smoother surface
B. Less efficient cutting and a rougher surface
C. More efficient cutting and a smoother surface
D. More efficient cutting and a rougher surface (correct)

528. 1. What part of the nervous system is responsible for fight or flight Sympathetic Nervous
System
2. What part of the nervous system causes increased salivary secretion?Autonomic Nervous System
3. What part of the nervous system increases heart rate?
Autonomic Nervous System (Sympathetic Nervous System)
Maybe you wil find this article useful:
 
The nervous system is divided into the somatic nervous system which controls organs under
voluntary control (mainly muscles) and the Autonomic Nervous System (ANS) which regulates
individual organ function and homeostasis, and for the most part is not subject to voluntary control.
It is also known as the visceral or automatic system.
The ANS is predominantly an efferent system transmitting impulses from the Central Nervous
System (CNS) to peripheral organ systems. Its effects include control of heart rate and force of
contraction, constriction and dilatation of blood vessels, contraction and relaxation of smooth
muscle in various organs, visual accommodation, pupillary size and secretions from exocrine and
endocrine glands. Autonomic nerves constitute all of the efferent fibres which leave the CNS,
except for those which innervate skeletal muscle. There are some afferent autonomic fibres (i.e.
transmit information from the periphery to the CNS) which are concerned with the mediation of
visceral sensation and the regulation of vasomotor and respiratory reflexes, for example the
baroreceptors and chemoreceptors in the carotid sinus and aortic arch which are important in the
control of heart rate, blood pressure and respiratory activity. These afferent fibres are usually
carried to the CNS by major autonomic nerves such as the vagus, splanchnic or pelvic nerves,
although afferent pain fibres from blood vessels may be carried by somatic nerves.
The ANS is primarily involved in reflex arcs, involving an autonomic or somatic afferent limb, and
then autonomic and somatic efferent limbs. For instance, afferent fibres may convey stimuli from
pain receptors, or mechanoreceptors and chemoreceptors in the heart, lungs, gastrointestinal tract
etc.
There may then be a reflex response to this involving autonomic efferent fibres causing contraction
of smooth muscle in certain organs (e.g. blood vessels, eyes, lungs, bladder, gastrointestinal tract)
and influencing the function of the heart and glands. The efferent limbs of these reflexes may also
involve the somatic nervous system (e.g. coughing and vomiting). Simple reflexes are completed
entirely within the organ concerned, whereas more complex reflexes are controlled by the higher
autonomic centres in the CNS, principally the hypothalamus.
The ANS is divided into two separate divisions called the Parasympathetic and Sympathetic
Systems, on the basis of anatomical and functional differences. Both of these systems consist of
myelinated preganglionic fibres which make synaptic connections with unmyelinated
postganglionic fibres, and it is these which then innervate the effector organ. These synapses
usually occur in clusters called ganglia. Most organs are innervated by fibres from both divisions of
the ANS, and the influence is usually opposing (e.g.the vagus slows the heart, whilst the
sympathetic nerves increase its rate and contractility), although it may be parallel (e.g. the salivary
glands). The responses of major effector organs to autonomic nerve impulses are summarised in
Table 1.
 Parasympathetic Nervous System
The preganglionic outflow of the parasympathetic nervous system arises from the cell bodies of the
motor nuclei of the cranial nerves III, VII, IX and X in the brain stem and from the second, third
and fourth sacral segments of the spinal cord. It is therefore also known as the cranio-sacral
outflow.
Preganglionic fibres run almost to the organ which is innervated, and synapse in ganglia close to or
within that organ, giving rise to postganglionic fibres which then innervate the relevant tissue. The
ganglion cells may be either well organised (e.g. myenteric plexus of the intestine) or diffuse (e.g.
bladder, blood vessels).
The cranial nerves III, VII and IX affect the pupil and salivary gland secretion, whilst the vagus
nerve (X) carries fibres to the heart, lungs, stomach, upper intestine and ureter. The sacral fibres
form pelvic plexuses which innervate the distal colon, rectum, bladder and reproductive organs.
In physiological terms, the parasympathetic system is concerned with conservation and restoration
of energy, as it causes a reduction in heart rate and blood pressure, and facilitates digestion and
absorption of nutrients, and consequently the excretion of waste products.
The chemical transmitter at both pre and postganglionic synapses in the parasympathetic system is
Acetylcholine (Ach). Ach is also the neurotransmitter at sympathetic preganglionic synapses, some
sympathetic postganglionic synapses, the neuromuscular junction (somatic nervous system), and at
some sites in the CNS. Nerve fibres that release Ach from their endings are described as cholinergic
fibres.
The synthesis of Ach occurs in the cytoplasm of nerve endings and is stored in vesicles in the
presynaptic terminal. The arrival of a presynaptic action potential causes an influx of calcium ions
and the release of the contents of several hundred vesicles into the synaptic cleft. The Ach then
binds to specific receptors on the postsynaptic membrane and increases the membrane permeability
to sodium, potassium and calcium ions, which results in an excitatory post-synaptic potential. The
action of Ach is terminated by hydrolysis with the enzyme Acetyl Cholinesterase.
The specific Ach receptors have been subdivided pharmacologically by the actions of the alkaloids
muscarine and nicotine. The actions of Ach at the preganglionic synapses in both the
parasympathetic and sympathetic systems is mimicked by nicotine, and all autonomic ganglia are
therefore termed nicotinic. Nicotinic transmission also occurs at the neuromuscular junction, in the
CNS, the adrenal medulla and at some sympathetic postganglionic sites (see later). However, the
actions of Ach at the parasympathetic postganglionic nerve ending is mimicked by muscarine.
Muscarinic transmission also occurs at certain sites in the CNS.

If you are interested, it is continued on http://www.nda.ox.ac.uk/wfsa/html/u05/u05_011.htm


4. What hormone increases blood glucose?Glucagon
5. What hormone reduces blood glucose? Insulin
6. What hormone increases blood glucose and potassium? Cortisol
 
529. A 30 year old man with unknown allergy to latex goes into anaphylactic shock whilst being
treated in the dental surgery.
Which drug and route of administration is of most benefit in this situation?
A. Hydrocortisone - orally
B. Chlorphenamine – intramuscularly
C. Chlorphenamine - orally
D. Epinephrine - intravenously
E. Epinephrine – intramuscularly

530.
75. What material is LEAST UNUSABLE for impression of PFM? - Alginate
72. What makes that the post does not til - Ferrule effect
Statistics: % of fissure sealants that fail and get carious

53. How far from the papilla would you place teeth in a denture? - 10mm

51. Which x-ray to use for implant? in eric whaites it mentions cone beam ct scan is it that or
another one?

46. Periapical with approximal caries in a molar the question was: what is the best access to the
cavity? Direct access. Oclussal access

39. Sarcoidosis is: hereditary, autoimmune


34. Cement used for temporary crowns
36. Cement used for cantilever with little preparation
57. Immunoglobuline present in mumps?is it igm
3. Minimum percentage of leucocytes for an extraction?
531. Goldish crust in the corner of tge moutg of a complete denture wearer patient( candida) not
included) - Angular chelitis

532. patient with recent complete denture, has ulcer in buccal sulcus? ( is it overextended flange? -
Overextended

533. Matrix band poor technique. what happen?


a. marginal over construction
b .marginal under construction
c. overhang
d. open contact point

 ans: improper placement of both wedge and matrix band cause overhang.
 "improper placement of wedge will cause?" answer is overhang.
 if it says "not placing the wedge at all will cause?" - answer is both overhang and open contact
 if quest is "improper matrix band placement will cause?"
answer could be a, b or c.. as the function of the matrix band is to restore anatomical contours
and establish good contact. but a marginal overconstruction can be corrected easily and a
marginal underconstruction is mostly due to inadequate filling but if it is due to improper band
placement, it would mean that the band has been placed too gingivally(which usually does not
happen) and would also lead to a gingival overhang. hence i opted for c.

534. How to decrease polymerization shrinkage of a composite.


Major ways of reducing polymerization shrinkage are:
1.Incremental placing of composite by two techniques: herring bone and lateral filling
methods .These two minimize the wall to wall effects of shrinkage.
2. Directinal curing technique:VLC materials start to cure at surface closest to curing light and then
shrink towards light. At the base of the box, curing can be initiated by using light transmitting
wedge to cure the increment closest to cervical margins first. Remainder of increment cured from
occlusal aspect and follows again herring bone method of increment placing.
3. Increasing the effective filler loading

535. MOST POTENT VASOCOSTRICTOR OF SKIN – ADRENALINE/ NOR ADRENALINE /

2.OTHERS-PRODUCT WHICH IS FIRST FORMED- CO2/ UREA/ OTHERS


3.DEVELOPMENT OF PERMANENT TOOTH GERM BY WHICH WEEK / 20 weeks - 5
months in utero
4- WHICH MUSCLE IS PIERCED IN INFIRIOR ALVEOLAR NERVE BLOCK /temporalis
&medial pteregiod - buccinator and superior constrictor, if it pierces medial pterygoid it causes
trismus

5.BEST POST OPERATIVE ANALGESIC - paracetamol 500mg 4 hrly  upto 4gm maximum
dose
6-FIBRIN STABILIZING FACTOR - factor xiii is called fibrin stabilising factor
7-CLOTTING FACTOR DECRASED IN PATIENT WITH LIVER DISEASE - vit k is required
for facor ii, vii, ix and x so it could be any of them becoz vit k is synthesized in liver

536. What happen if we use old diamond bur? - you have to apply excessive pressure and more heat
will be produced which is not good for pulp
2. What is best protection for pulp? -its dentine and if material then Zno eugenol ( eugenol content
of ZOE acts as obtundent and analgesic and thrfore used as sedative dressing over reversibly
inflamed pulp. calcium hydroxide can induce mineralization of adjacent pulp)
3. What happen to interproximal composite filling? (Over contouring, under contouring, fracture)

537. Which lung volume doesnt change after exercise? - residual volume

 What happens to your lung capacity when you smoke?


Smoking doesn't reduce lung capacity, it does increase the dead space as it reduces the surface area
available for the exchange of gases.
 What happens to stroke volume during exercise?
it increases with exercise
 What happens to muscle during exercise?
The muscle cells burn off ATP (Really fast) and they get much tighter to deal with the strain. Make
sure that you always stretch and warm up before running as the muscles could strain!
 What happens to your residual volume during exercise?
In contrast to Inspiratory Reserve volume,Tidal volume and Expiratory reserve volume, residual
volume does not change with exercise
 What is the difference between lung volume and lung capacity?
lung capacity/volume: the volume of the air in the lung after maximal inspiration there is no
difference

538. Immediate denture stages are:


1- assessment
2- primary imp.
3- sec. imp.
4- record occlusion
5- try-in
6- extraction
7- finish

539. The correct order of steps of partial denture design..??


1.- surveying
2.- outline saddles
3.- plan support
4.-obtain retention
5.-assess bracing required
6.- choose connector...
540. Which of the following ahs the highest sucrose content:
A. Ice cream
B. Canned juice
C. Cough syrups
D. Breakfast cereal
E. Sweet potato

541.  penultimate tooth to erupting permanent dentition? - second molar


542. DOSAGE OF 8 YEAR OLD CHILD WITH ANAPHYLACTIC REACTION? - 0.3 ml of
1:1000 adrenalin and if less than 6 than .15ml

543. DEVELOPMENT OF PAROTID GLAND BEGINS IN UTERO AT WHAT MONTHS? - 6


weeks

544. After the age of 6 years, the greatest increase in the size of the mandible occurs:
A. At the symphysis
B. Between canines
C. Distal to the first molar

545. Which of the following does state BEST the morphology of periodontal ligament fibres:
A. Elastic
B. Striated
C. Non striated
D. Levity
E. Wavy

546. Ten years old amalgam filling raised on tooth surface. Reason why its raised? – creep

547. Patient whose hands fell warm and moist is MOST likely to be suffering from:**

A. Anxiety
B. Congestive cardiac failure
C. Thyrotoxicosis

548. What kind of neurons supply mastocatory muscles ? - Slow conducting or fast conducting.

549. 1 what is most likely to happen to an interproximal composite filling rather than amalgam?
Overcontouring, undercontouring ,overhang.fracture)

2.Amalgam MOD on molar.you did rct and palatal cusps are missing..how do you restore? Full
gold crown , ¾ gold crown.?

550. 1. radiograph of choice for new edentulous patients in surgery? - periapical for edentulous
patient in problem area
2. method of diagnosing interproximal caries in anterior teeth? - its periapical by parallel technique
551. X-RAY TO DETECT CARIES IN PRIMARY MOLARS OF A THREE YEAR OLD CHILD
OPTIONS: BIMOLAR TECHNIQUE,PA,BITEWING ETC

552. WHICH ANAESTHETIC PENETRATES THE BONE BETTER – Articane

553. WHICH FRACTURE WILL LEAD TO MENINGITIS - Lefort 3 leads to meningitis

554. WHICH FRACTURE WILL LEAD TO EMPHYSEMA - Orbital emphysema is typically a


benign condition that occurs following forceful injection of air into the orbital soft tissue spaces. In
many cases there is a history of trauma and fracture of an orbital bone, which permits air entry.
However, other mechanisms of orbital emphysema have been reported including infection,
pulmonary barotrauma, injury from compressed-air hoses, and complications from surgery
including dental procedures

555. A 58 year old male presents at your surgery complaining of a sharp pain of no more than 30
minutes duration arising from his upper left molar region. The pain is brought on by cold stimuli but
persists after the stimulus is removed. It does not seem to occur spontaneously. He has tried taking
paracetemol and this does temporarily stop the pain from recurring. The upper left 6 reacted to a
lower current on electronic pulp testing than the upper right 6, upper left 7 or the lower left molars.

What is the most likely cause of the patient's pain?


A. Acute/reversible pulpitis
B. Dentine sensitivity
C. Chronic/irreversible pulpitis
D. Periapical periodontitis
E. Trigeminal neuralgia
Explanation: importantly, there is a reduced response to pulp testing. Hence, the answer would be
Chronic Irreversible Pulpitis.
But reversible pulpitis doesn't persist after removal of the stimulus. I think the answer is
chronic/irreversible pulpitis based on the history given.

556. Vaccination which doesn’t include deactivated micro organism? - polio vaccine

557. Which immunoglobin is present after a bacterial and viral infection? - IgM and IgG

558. Which immunoglobulin is pentameric? – IgM

559. fluorides

Prescribing high concentration fluoride toothpaste

Sodium fluoride 2,800 ppm toothpaste


Indications: high caries risk patients aged 10 years and over.

Sodium fluoride 5,000 ppm toothpaste


Indications: patients aged 16 years and over with high caries risk, present or potential for root caries, dry
mouth, orthodontic appliances, overdentures, those with highly cariogenic diet or medication.

560. In periodontal ligament where are fibers are thinner? - middle third of root.tencate

561. Jugular venous pressure is best described as

A. Pulse pressure
B. 10 mmHg more than ventricular pressure
C. 10mmHg less
D. 20 mm hg less than VP
E. 20mmhg more
Ans: Jugular Venous pressure is right atrium pressure and it is  20 mm Hg less than ventricular pressue.

562. PH of venous blood - 7.3-7,4

563. 1. Woman with hiatus hernia with tooth surface loss. Best treatment option( surgery wasn't an
option)
A. Medicine
B. Dietary advice
c. Occlusal splint
well in patient with hiatus hernia it depends how sever is the condition. if mild can be controlled by
medicine and dietary advice and if sever may require surgery. usually in most cases, sufferers
experience no discomfort and no treatment is required. However, when the hiatal hernia is large, or
is of the paraesophageal type, it is likely to cause esophageal stricture and discomfort. Symptomatic
patients should elevate the head of their beds and avoid lying down directly after meals until
treatment is rendered. If the condition has been brought on by stress, stress reduction techniques
may be prescribed, or if overweight, weight loss may be indicated. Medications that reduce
the lower esophageal sphincter (or LES) pressure should be avoided. Antisecretory drugs
like proton pump inhibitors and H2 receptor blockers can be used to reduce acid secretion.

Where hernia symptoms are severe and chronic acid reflux is involved, surgery is sometimes
recommended, as chronic reflux can severely injure the esophagus and even lead to esophageal
cancer.

2.young adult with tooth surface loss. Best treatment option

removal of iatrogenic cause. And taking photographs and study models and comparing. usually its
known as smith index.

3. Patient with bulemia. Best treatment option

referring to gp and gp will decide to have a psychiatric concealing or not    

4.patient with anorexia nervosa, high caries rate, high sugar intake
control of sugar intake, dietary advice. ohi which include toothbrushing and use of mouth wash, and
regular appointment with dentist. and referring to gp for anorexia nervosa.

564.  What is TRUE


A. Boiling point of acrylic > boiling point of water
B. Boiling point of acrylic is similar to that of water
C. Boiling point of acrylic < boiling point of water

565. which radiograph for the following BPE scores


1..
232
222

2.
*14
423

3..
*34
42*

4..
*2*
*1*

5..
424
423
Ans. 1.horizontal bitewing
2.upper sextant=full mouth periapical long cone technique with vertical or
horizontal bitewing
lower=vertical bitewing
3.same as upper in 2 for both upper and lower
4.same
5.vertical bitewing for both upper and lower
 
566. Hormone that control calcium metabolism?(parathyroid hormone, vit D, calcitonin)
Ans. Answer is Parathyroid hormone.
Vitamin D is essential for the ACTION of parathyroid
Calcitonin is antagonist of parathyroid hormone.

567. What are the end products of co2 metabolism? is it HCO3 and H+ ?

568. Teenager has swelling involving his upper lip, the corner of his nose and a region under his left
eye. The swollen area is soft, fluctuant and pointed on the labial plate under his lips on the left side.
His body temperature is 39°. What is the first thing you would do after taking history and
temperature
A. Refer him to physician
B. Anaesthetise all of the maxillary left anterior teeth to provide instant
relief
C. Give him an ice pack to be placed on the area to control the swelling
D. Take radiograph and test vitality of his teeth
E. Write prescription for antibiotics and delay treatment until swelling is
Reduced
569. progenitor cells of platelets...?
Ans. Megakaryocytes are progenitor cells of platlets ..
570. What are neurons of Muscles of mastication ?alpha,beta,gamma
571. what’s the maximun time GDC leaves the dentist physically impaired away from his
profession? - 12 months

572. Which is completely derived from meckles cartilage- incus, malleus, stapeus

573. Regeneration of odontoblast after a pulp pathology is from


A. Regenerate from the left odontoblast
B. Regenerate from undifferentiated mesenchymel cells
C. Regenerate from ectodermal cells
D. Regenerate from the undelying nectrotic tissue

574. In children a disease with enzyme deficiency


A. hypohpospatesia
B. Cyclic neutropenia
C. Juvienile periodontitis

Answer: A.Hypophosphatasia
Hypophosphatasia is a rare, and sometimes fatal metabolic bone disease. Clinical symptoms are
heterogeneous ranging from the rapidly fatal perinatal variant, with profound skeletal
hypomineralization and respiratory compromise to a milder, progressive osteomalacia later in life.
Tissue non-specific alkaline phosphatase (TNSALP) deficiency in osteoblasts and chondrocytes
impairs bone mineralization, leading to rickets or osteomalacia. The pathognomonic finding is
subnormal serum activity of the TNSALP enzyme, which is caused by one of 200 genetic mutations
identified to date in the gene encoding TNSALP.

575. In a flouridated toothpaste with 0.304% monoflourophospate the ampont of flouride ions
A.400ppm
B.1000ppm
c.1500ppm
D.4000ppm
Answer: A. 400ppm
A low fluoride, sorbitol-based toothpaste designed specifically for children is available (Colgate
Junior Toothpaste) and contains 0.304% MFP (400 ppm fluoride) 
 
576. Compared to natural teeth the displacement occurring in the denture supporting mucosa under
functional load
A. Ten times greater
B. Ten times less greater
C. One half

577. Cause of hypoplasia in lateral permanent incisor?


Ans. Caries or trauma to primary tooth

578. Which type of dentin is not formed immediately due to a pulpal trauma
A. primary
B. Secondary
C. Reparative
D. Tertiary
 
Normal prothrombin time & increased partial thromboblastin time seen in
A. Haemophilia A
B. Thrombocytic pupura
C. Leukemia
D. Won willibrand disease

579. Which drug causes abdominal cramps in high doses? - clindamycin


which drug causes diarrhea in high doses? - amoxicillin or ampicillin is better choice

580. 24. According to the Electricity at Work Regulations 1989 electrical equipment must be tested
every?
a. three years
b. one year
c. two years
d. 6 months
 
581. According to the water regulations, all dental equipment must be protected by back-syphonage
and between the water ingress and drainage points there should be a
a. “Type A” gap of 20mm
b. “Type B” gap of 30mm
c. “Type C” gap of 40mm
d. “Type D” gap of 50mm.
 
582. Fire certificates are only required for buildings with 20 people working in them or if more than
ten people work on floors other than ground floors (same applies to dental practices). Fire risk
assessment should be carried every
a. 3 months
b. 6 months
c. 9 months
d. one year
 
583. Most predictable anesthetic technique for lower lateral incisor?
Ans. infiltration 

584. Lingual cavity in post teeth in Parkinson pt which mat is used

Ans. RMGIC

585. Condition in which phagocytic killing of bacteria is difficult?


Ans. Granuloma
586. child had 5mg/kg flouride its potential lethal dose what the first line treatment?
Give child salty drink
Give child sugary drink
Give child injection
Give child glucose

587. Best xray for:


1.Showing successor teeth in young child - OPG
2.caries in 3 years old - Bimolar
3. Bilateral fracture of condyles - OPG
4.proximal caries in dentine - bitewings
5.dentine caries in anterior teeth - direct vision
6.impacted canine not palpable palataly - upper occlusal
7.showing amount of bone on a molar - bitewings
8. Showing relation between impacted 3rd molar and ID nerve - IOPA
9. Salivary calculi in submandibular duct -  lower occlussal

Options: bimolars, bitewings, OPG, lateral oblique, IOPA, upper occlusal, lower occlussal, direct
vision, probing, CT scan, occipitomental, lateral cephAlometrics

588.
1) BP in 70 year old 140/90 mm of Hg
2) Consent - children http://www.dentalprotection.org/adx/aspx/adxGetMedia.aspx?
DocID=3258,3257,158,1,Documents&MediaID=1907&Filename=Consent+P%26A.pdf
3) Root canal sealer causing tissue damage - Paraformaldehyde-containing endodontic filling
materials or sealers (frequently known as Sargenti pastes, N-2, N-2 Universal, RC-2B or RC-2B
White) should not be used for endodontic treatment because those materials are unsafe. Extensive
scientific research has proven unequivocally that paraformaldehyde-containing filling materials and
sealers can cause irreversible damage to tissues near the root canal system including the following:
destruction of connective tissue and bone; intractable pain; paresthesia and dysthesia of the
mandibular and maxillary nerves; and chronic infections of the maxillary sinus.

4)
SELECTED BACTERIAL SPECIES FOUND IN DENTAL PLAQUE
Facultative Anaerobic
Gram-Positive Streptococcus mutans
Streptococcus sanguis
Actinomyces viscosus
Gram-negative Actinobacillus
actinomycetemcomitans
Capnocytophypa species
Eikenella corrodens Porphyromonas gingivalis
Fusobacterium nucleatum
Prevotella intermedia
Bacteroides forsythus
Campylobacter rectus
Spirochetes Treponema denticola
(Other Treponema species)
5) Best Dental Health - Wales
Scotland
North
London
West Midlands
worst South

589. % of UK population with type 1 hypersensitivity?


Ans. 10% scully

590. impresion for gaging reflex is polyether


Ans. t is polyether as it is fast setting

591. What type of virus is hepatitis A.


a. Dna
b. Rna
592. Features of epithelial dysplasia
The changes that occur in epithelial dysplasia include:

 Drop-shaped rete processes


 Basal cell hyperplasia
 Irregular epithelial stratification
 Nuclear hyperchromatism
 Increased nuclear-cytoplasmic ratio
 Increased normal and abnormal mitosis
 Enlarged nucleoli
 Individual cell keratinization
 Loss or reduction of cellular cohesion
 Cellular pleomorphism
 Loss of basal cell polarity
 Anisocytosis
 koilocytosis
Epithelial cell dysplasia is divided into three categories of severity: mild, moderate, and severe.
Epithelial dysplasia becomes microinvasive squamous cell carcinoma once the tumor begins to
invade nearby tissue.
593. 1. Partial pressure of oxygen in the alveoli

Ans. 105

594. Picture of a fractured central incisor with the pink pulp showing and a few drops of blood, how
would you manage it?
a. Non setting calcium hydroxide and composite
b. Non setting calcium hydroxide and gic
c. Setting calcium hydroxide and gic
d. Setting calcium hydroxide and composit

595. A 30 year old man with unknown allergy to latex goes into anaphylactic shock whilst being
treated in the dental surgery.
Which drug and route of administration is of most benefit in this situation?
A. Hydrocortisone - orally
B. Chlorphenamine – intramuscularly
C. Chlorphenamine - orally
D. Epinephrine - intravenously
E. Epinephrine – intramuscularly

596. 1. What is the cause of the white line


a. Amelogenesis imperfecta
b. Dentinogenesis imperfecta
c. Hypomineralization
d. Flourosis
2. How would you manage this white line
a. Composite veneers
b. Porcelain veneers
c. Porcelain crowns

597. Picture of red coloured post in central incisors of cervical level shown.
Identify the post?
What is the ideal length required for post?
What is needed to prevent rotation of post?
.Why is post used?

Ans. it is burnout indirect wrought metal post, length one third of tooth length for CI it is 9 mm
parallel side twist drill to remove dentine and create post space, to distribute stress and provide
retention for the tooth

598. Root filled tooth with no symptoms. When do you take the first x-ray for the follow-up? 6
months, 12 months, 18 months, etc.
2. Best interdental cleaning for perio patients: interdental brushes, dental floss, tooth picks, etc
599. What blood vessels have the most amount of smooth muscles?
a. Arteries
b. Arterioles
c. Capillaries
d. Veinules
e. Veins

Arteries have a great deal more smooth muscle within their walls than veins, thus their greater wall
thickness. This is because they have to carry pumped blood away from the heart to all the organs
and tissues that need the oxygenated blood. The endothelial lining of each is similar. 

600. What cells do you see in parasitic infection? eosinophils


8. What is the best protection for the pulp? Dentine

601. 1.WHICH ONE IS POLISHED THE BEST? MICROFILM,GLASS IONOMER,RESIN


MODIFIED GLASS IONOMER ...SO MANY OPTIONS)
2.WHICH OPTION IS MANDATORY DURING INHALATION SEDATION?OPT:EGG,PULS
OXYMETRY..
3.HOW LONG DEPTH OF NAYARR CORE SHOULD GO THROUGH THE
ROOT.OPT:3MM,6MM,HALF OF THE ROOT..,

601. A 13 year old girl presents with an unerupted permanent canine and a retained primary
canine. You cannot palpate the unerupted canine in the buccal sulcus and you are uncertain as to
whether it is displaced palatally or in the line of the arch.
Which single radiographic view would be most helpful in locating the unerupted tooth?
A. Bitewing
B. DPT
C. Lateral oblique
D. Single periapical
E. Upper anterior occlusal

602. Which vessels involved in thermoregulation and vasodilatation out of capillaries arteries
arterioles

4)cells present in rheumatoid arthritis? - lymphocytes and plasma cells increase in RA

603. An african man with microcytic anaemia,normal ferritin levels, what other investigations
would you do?

Ans. CBC, sickeldex., hb electrophoresis


604. Most important factor for efficacy of hand instruments?

Ans. instrument should be sharp

605. picture of buccal mucosa with a white line at the level of teeth what causes it?
lichen planus
lichenoid reaction
traumatic keratosis

2 first stage in management of rampant caries?


excavation and fluoridation
prevention and stabilisation.

3%of 5yrs olds with dental caries?

4 picture of man with whitish growth on upper lip?


a sq cell papilloma
b sq cell carcinoma
c bcc

5 man with multiple myeloma comes for extraction and comes back after 6 weeks later and his
socket has not healed yet  ?
a multiplemyeloma of mandible
b drug induced osteoradionecrosis
c dry socket
d drug induced osteo sclerosis

6 a man who smokes 2 packs of cigarettes a day and drinks would have increased risk of oral cancer
of?
a 11times
b 13 times
c 47 times

7 a black person with gingiva with melanotic pigmentation and white line across the central incisors
alone  how wud u manage this?
a composite veneers
b porcelain veneers 
c porcelain crowns 

8 a picture of man with inability to open mouth properly for treatment what is the cause?
a sarcoidosis
b scleroderma

606.
1. The ideal rake angle requred for the universal curette in sub gingival currettage?(-20,0,10,20)
2. Rake angle achievable by universal curette in subgingival curette? (options same as above)

Ans. rake angle is not always equal to the cutting angle. by definition, if the leading edge of the
blade is ahead of the perpendicular,( hence acute angled) the rake angle is negative. if it is behind
the perpendicular (obtuse angle) rake angle is positive. and if it is in line with the perpendicular,
rake angle is 0.
in the first ques it asks the ideal rake angle which is -20. (ideal cutting angle is 45)
2nd quest asks upto a max of which angle. ans is 0. (cutting angle can go to a max of 90)

607. Lower anterior missing in a 71 years old lady, has no pathology of significance and good
amount of bone. What kind if prosthesis? - minimum preparation bridge

Upper anterior missing in 22 years old sports man who wears a mouth gard? - conventional bridge

Upper anterior lost in 13 years old - minimum preparation resin bonded bridge

608. Anaemia's most commonly inherited disorder? - Sickle cell anemia

609. Bone with endochondrial ossification only? - squamous part of temporal bone and parts of
mandible

610. Bone with intramembranous ossifi only? Maxilla not given in option:-((

611. Saliva in relation to plasma? Hypo, iso , hrpertonic – hypotonic


612. Cells in granulomatous inflammation? - macrophages, MNG's AND epithelioid cells
613. Maximum occlusal force beared by which pdl fibers? – oblique
614. A patient complains of pain to his dentist. He has caries with premolars and molars. He is
visiting a dentist after 2 years. What part of the history would you ask first.

Social history,
history of presenting complaint,
medical history,
dental history,
family history.
615. Which drug overdose causes hepatotoxicity? – PARACETAMOL
616. What cells do you see in parasitic infection? Basophils, eosinophils,
617. Class III very small. What's the best treatment? Lining, composite filling (no lining), etc
618. What's the best protection for the pulp? Dentine, lining,
619. Analgesic for children? Dose - paracetamol.6-12 yrs 250-500mg, 1to 5 years, 120-250mg
(every 4-6 hours)
620. drug for 8 yrs old child with anaphylixis and dose ? - epinephrine injection 250ug,0.5ml
621. Which lesion is more malignant? Lichen planus, submucous fibrosis, pemphigoid?
622. First line treatment pt 5years old with an abscess on buccal of lower first molar I and D.
623. Most common failure of Maryland bridge? – debonding (retention failure)
624. Pulp stones seen in which syndrome? - Ehlers–Danlos syndrome
625. SUCCESFUL MOLAR ENDODONTIC DEPEND ON EITHER – a. straight line access b.
patency
626. Which of the following drug is mast cell stabilizer?
a. salbutamol
b. adrenaline
c. ipratropium bromide
627. Patient with iatrogenic overproduction of ACTH? - Cushing syndrome
628. Pt with endogenous production of ACTH? - Cushing disease

Exogenous vs. endogenous

Hormones that come from outside the body are called exogenous; hormones that come from within
the body are called endogenous.

The most common cause of Cushing's syndrome is exogenous administration of glucocorticoids


prescribed by a health care practitioner to treat other diseases (called iatrogenic Cushing's
syndrome). This can be an effect of steroid treatment of a variety of disorders such as asthma and
rheumatoid arthritis, or in immunosuppression after an organ transplant. Administration of synthetic
ACTH is also possible, but ACTH is less often prescribed due to cost and lesser utility. Although
rare, Cushing's syndrome can also be due to the use of medroxyprogesterone [8][9]

Endogenous Cushing's syndrome results from some derangement of the body's own system of
secreting cortisol. Normally, ACTH is released from the pituitary gland when necessary to stimulate
the release of cortisol from the adrenal glands.

 In pituitary Cushing's, a benign pituitary adenoma secretes ACTH. This is also known as
Cushing's disease and is responsible for 70% of endogenous Cushing's syndrome.[10]

 In adrenal Cushing's, excess cortisol is produced by adrenal gland tumors, hyperplastic


adrenal glands, or adrenal glands with nodular adrenal hyperplasia.

 Finally, tumors outside the normal pituitary-adrenal system can produce ACTH that affects
the adrenal glands. This final etiology is called ectopic or paraneoplastic Cushing's
syndrome and is seen in diseases like small cell lung cancer.

[edit] Pseudo-cushing's syndrome

Elevated levels of total cortisol can also be due to estrogen found in oral contraceptive pills that
contain a mixture of estrogen and progesterone. Estrogen can cause an increase of cortisol-binding
globulin and thereby cause the total cortisol level to be elevated. However, the total free cortisol,
which is the active hormone in the body, as measured by a 24 hour urine collection for urinary free
cortisol, is normal.[11]

629. Where the bone is lost the most in periodontal disease? Buccal, Lingual, Mesial, Distal,
Approximal
630. LOSS OF TASTE SENSATION IS ATTRIBUTED TO WHICH NERVE - Chorda Tympani
(facial nerve) 
631. WHICH GLAND PRODUCE CORTISOL - Adrenal Cortex ( Adrenal Gland )
632. ACTH IS PRODUCED BY WHICH GLAND - Anterior Pituitary Gland
633. What combines with O2? - hb
634. What is the arterial pressure for 02and C02? - Po2 is 100 Pco2 is 40
635. What cement u use to glue a alumina core ceramic crown? - GIC/RMGIC/COMPOSITE
RESIN CEMENT
636. In which condition will direct immunoflourescence reveal binding of autoantibodies to the
intercellular substance of epithelial cells? - Pemhigus ( IgG ) 
Pemphigus/ benign mucous memb pemphigoid. / erythema multiforme
637. WHAT IS THE FLAVOURED TOPICAL – benzocaine.
638. What’s increased in elderly? Systole, diastole, stroke volume, etc
639. How many of all the cancers in UK are oral in percentage? – 87% in over 50 year age group
How many deaths in UK each year caused by oral cancer?
 In 2007, 5,410 people in the UK were diagnosed with oral cancer .i.e around 3% of all
cancers.

 Oral cancer caused around 1,822 deaths in the UK in 2008.

640. Pulpotomy – which sulphate, etc (calcium hidroxide was not an option!!!)is the best material
to use, especially because of its low toxicity? Formocresol, ferric

Ans. ferric sulphate

641. Root filled tooth with no symptoms. When do you take the first x-ray for the follow-up? 6
months, 12 months, 18 months, etc.
642. Best interdental cleaning for perio patients: interdental brushes, dental floss, tooth picks,
etc

643. Most benign: Kaposi sar / adenolymphoma /  burkitts lymphoma

644. Immunoglobulin in mucosa – IgA

645. Site of oral cancer in a farmer exposed to sun light – lower lip

646. Adenoid cystic carcinoma is the most common type of cancer in the minor salivary

647. In which condition will direct immunoflourescence reveal binding of autoantibodies to the
intercellular substance of epithelial cells?

Pemphigus/ beng mucous memb pemphigoid. / erythema multiforme

648. Everything about intraligamentary anaesthesia: can it affect permanent tooth germ? Does the
needle need to be smaller than periodontal ligament? Is it painful? - YES TO ALL 3

649.
650. Cement for temporary crown: GIC, zinc phosphate, zinc policarboxylate, resin, ZOE
651. Which root filling material comes in 2 pastes? Tubli Seal, etc
652. Which cell release histamine? Mast cells

653. Tumour in salivary gland which spreads along nerve sheaths: Adenoid cystic ca

654. What's the percentage of >25 years with periodontitis in UK? - 64%

655. which oral tissue heals without scar? - Palatal tissue

656. Which salivary gland tumor mostly occurs bilaterally? - wartin tumor benign,
actinic cell carcinoma malignant

657. Which has softer walls? Veins, artery, arteriole, etc..

658. Cell not present in healthy periodontal - Cementoclasts they r present only in pathological
conditions

659. Which LA in patient with cardiac arrythmias? – Lidocaine

660. Which hormone is secreted by the middle lobe of the pituitary gland? MSH

661. In which blood cell. Is the nucleus lost in the early stage? – RBC

662. What is the dose of Amoxicillin oral suspension - 125mg/5ml-kid dose

663. What is the year of when RIDDOR regulation came out? – 1995

664. What is the normal level of Glucose on a BM stick - 4-7m mol

665. What is the altered cast technique?- special technique for mucosa and tooth-born
pd(keneddy's class1&2)? Mucocompressive/mucostatic impression.

666. Most imp function of saliva? - lubrication


667. Disposal of rotary file? - sharps box
668. When do we use a nayyar core? - nayyar core also called amalcore or coronal radicular
amalgam core is used in non vital root filled premolars and molars as a core to give tooth extra
strength before placing a crown
669. Analgesic used in geographic tongue - benzydamine hydrochloride mouthwash
670. Best way to assess caries activity in a root - bitewing radiographs
671.How would you treat a patient with MRSA,TB,Hepatitis?
> options: single use instruments, treat using standard conditions, send them to hospital

672. Pt with epitaxis and desqamative gingivitis....


Which condition?- mucous membrane pemphigoid

673. An upper deciduous molar has a caries exposure and on X ray the corresponding 2nd
permanent premolar is absent. What treatment would you do to the deciduous tooth:
A. Pulpotomy
B. Endodontic treatment
C. Pulp capping

674. Epithelium lining the free gingiva is keratinized or non keratinized?

675. Neutrophils present most in- Bacterial.  /  viral.  / fungi

676. Very small class 3 cavity...- Whats the best treatment? -


> Lining / composite / no lining /.......

677. Child with ulcers on soft palate, fever, malaise? – Herpangina

678. Child with fever, blisters on trunk n mouth...what diagnosis? – Chickenpox

679. Who is responsible for radiation protection? – RPA

680. The periodontal ligament in a teeth without use appear to be:


A. Narrow
B. Wide

681. In class II restoration, all of the following considered to occur as probable causes of
periodontal problems except:

A. Flat ridge
B. Faulty or not proper contour
C. Not properly polished restoration
D. Cervical wall is too deeply apical
E. Overextension of lining in cavity

682. How many pulp horns are presented in a typical mandibular deciduous second molar:
A. 2
B. 3
C. 4
D. 5

683. Which root filling tech use a hand piece? - Thermo mechanical compaction

684.  1st line of treatment for a patient with lower lateral extracted while alveolus heals to cover
the gap? Conventional cantilever, partial denture, etc
685. Gutta percha softens at what temperature - 80-82 degrees
686. Anaesthesia for upper lateral incisor apicectomy . Anesthesia for extraction of canine -
Infraobital n block nd infiltrations along wid depending on options
687. Features of Bilateral fracture body of mandible. - Disturbed occlusion and open bite
Sublingual hematoma
688. which cement is placed after treating tooth with polyacrylic acid? – Gic

689. What is needed to prevent rotation of post? - Ferrule effect


670. What does a single motor neuron to a lingual musculature suppy
1. Muscle spindle
2. Single muscle fiber
3. Multiple muscle fibers
671. Compensated blood loss. - 20%/40%/60%

672. A pt presents with pain in the tongue and on swallowing. What is your diagnosis and how
will be treated? - giant cell arthritis

673. A pt present with sharp pain and wind and cold make it worse.What is your diagnosis and
how to treat it? - Trigeminal neuralgia

674. Who is more appropriate to give consent?


- Stepfather of a child
- Grandmother of a child
- 16 year old sister of an 8 yearold child
- Mother of 4 year old child

675. Crown for a post retained tooth - Metalceramic


676. Impression material for resorbed lower ridge - .Zoe paste
677.cement used for temporary crown - .Zoe cement
678. Medication for chronic sinusitis - amoxicillin / clindamycin+metronidazole

679. To minimize the load on free end saddle partial denture


1.Use teeth with narrow buccal-lingual dimension
2. Use mucco-compressive impression
680. Whats the antagonist for heparin? - protamine sulfate

681. in which condition are all factors deficient ? - disseminated intravascular coagulation

682. What harmone increases blood glucose and potassium? - Cortisol increases blood glucose and
potassium
683.   What harmone increases blood glucose? - Glucagon.
684. Just want to share Diet Analysis
 
Marking diet sheet-
 
3 Day diary record which includes weekdays and weekend
1.       Ring the main meals
2.       Underline all sugar intakes in red.
3.       Identify between meal snacks and note any associations.
 
Dietary Advice –
 
·         After looking at your diet sheet, I’ve noticed that you are taking a lot of added sugar
everyday in tea, candies, fizzy drinks etc.
·         To improve the health of your teeth, you will have to make some changes to your diet.
·         You have to avoid in-between meal sugary snacks and fizzy drinks. If you have to snack,
eat nuts or crisps instead of sugary foods.
·         Eat sugars with the meals instead of dividing them through the day.
·         You can limit sweets to once in the week.
·         Don’t finish your meal with sugars.
·         If you eat a sugary meal eat cheese or chew gums9suger free) afterwards.
·         Eat a lot of fresh fruits and vegetables in the day.
·         If you take fizzy drinks, try to use a straw whenever possible.
·         Do you have any questions?
 
Theory questions-
 
I.      Which study proves relationship between caries and sugar?
 
1.       Vipeholm Study
2.       Turku sugar study
 
II.    What were the findings in the study?
 
1.       Vipeholm study was done to investigate the association between sugar consumption and
dental caries. It concluded that-
 
i.      Consumption of sugary food is associated with a higher caries incidence.
ii.    Increasing the frequency of sugar intake increases caries incidence
iii.  Lowering sugar intake reduces caries incidence.
iv.  Consumption of sticky sugar-containing between meal products can be associated with high
caries risk.
v.    Several dietary factors are associated with caries incidence:
§  Amount of fermentable carbohydrate consumed
§  Sugar concentration of food
§  Physical form of carbohydrate
§  Oral retentiveness
§  Frequency of eating meals and snacks
§  Length of interval between eating
§  Sequence of food consumption
 
2.       Turku Study-A longitudinal study to evaluate the caries incidence as affected by partial
substitution of dietary sucrose (S) with xylitol (X), the effects of S- or X-containing chewing
gums being compared during one year. The results show a profound difference in the caries rate
between the two groups. The findings clearly indicate a therapeutic, caries inhibitory effect of
xylitol.
 
III.  Names of 4 commonest substances to look for and avoid on labels-
 
Refined sugar, Sucrose, Fructose, Glucose

685. What channels does lignocaine block? - sodium channels


686. Anion in bone? – phosphate

687. which orthodontic appliance requires doesnot requires pt compliance? – Fixed

688. GIC bonds better to dentine or enamel?

689. Leison of nerve in cavernous sinus that supplies Musles of eye and divides into two branches
what nerve? - The oculo motor nerve III divides into superior and inferior  branches and also is in
the cavernous sinus.
690. Ulcers not common in which type of viral infection? – measles

691. Which salivary gland tumour occur bilaterally?- Yes, it is Warthin’s but only in 10%

692. WHAT IS THE GOLDEN TIME FOR A TOOTH TO BE OUT OF THE SOCKET? - 30
min

693. What is the equivalent dose of background radiation in hours for 2 bitewing? - 8hrs

694. What is required to increase the efficiency and decrease the fatigue of Universal curette?

Options: sharp, parallel to long axis of tooth surface, lower shank should be parallel to tooth
surface, light weight, small ribbed surface, use a modified pen grasp.)

695. What's the equivalent dose of background radiation in hours for 2 bitewing?

See table in churchill: 


If F speed film: 6.4 hours
If E speed: 8 hrs
Panoramic(rare earth screen) 28hrs
"(calcium tungstate) 56 hrs

696. Which condition their is intracellular killing of bacteria?

Inflammatory bowel disease......monocytes aid in phagocytes

697. Analgesic causing ringing sound? - Aspirin

698. Analgesic given to patient with bleeding disorder? Aspirin

699. which type of hypersensitivity reaction  is seen in tuberculous lesion?


a) type1
b)type2
c)type3
d)type4
700. Which of the following is the most important in the development of  dental caries?
a) time of sugar intake
b) frequency of sugar intake
c) amount of sugar intake
 
701. A 25-year-old patient presents with pain in a non-restored, lower posterior tooth. What
diagnostic tests would be MOST EFFECTIVE in determining cracked tooth syndrome?
1). EPT
2). Fiber optic light
3). Tooth sleuth
4). Radiographs
5). Staining
A. (1) and (5)
B. (1), (2) and (5)
C. (2), (3), and (5)
D. (1), (3), (4) and (5)
E. (4) and (5)
702. Pt with fatigue, night sweats, gingival enlargement and bleeding, - Leukemia
703. Bur to prepare the rest for dentures - round bur
704. Restoration for pt with xerostomia - conventional GIC
705. Restoration for pt with Parkinson – RMGIC
706. Drug causing fixed ulcer? – nicorandil
707. LA for patients with cardiac arrhythmias? - prilocine
708. Which of the clotting factors are associated with the following conditions:
a)chirstmas disease - factor 9
b) von villebrant disease – vonwillbrand factor
c)hemophilia A - factor 8
d)vit K deficiency - factors: 2, 7, 9 and 10
709. BLOOD SUPLY INTERFERE IN SALIVATION, HOW? - sypmathetic decrease salivation,
and parasympathetic increase it
710. WHAT IS THE %OF TOOTH EROSION IN 6 YEAR OLD? - 46% , 15 YEAR OLD 34%IN
UK
711. DISEASE WD PROGRESSIVE CARTILAGENOUS DESTRUCTION? – Osteoarthritis
712. MAN WITH PAIN IN PHALANGES   ? RHEUMATOID ARTHRITIS
713. Development of permanent tooth germ by which week? - 20th week
714. Best way to give dental health education?
1. Dentist advising patient on dental chair
2. Parent teaching their children
3. Through leaflets
4. Dental health education in school days
715. If there is lesion of right hypoglossal nerve then tongue deviates to which side? - TO THE
RIGHT SIDE (Bells palsy due to facial n palsy angle of mouth is drawn to unaffected( opposit)
side )
716. Most potent vasoconstrictor of skin? - non epinephrine
717. 62 YEAR OLD IS DENIED A JOB BECAUSE OF HIS AGE-WHAT ACT? – equalityact
718. DENTAL NJURSE WANTS TO TAKE RADIOGRAPH – IRMER
719. NURSE DENIED LEAVE – EMPLOYMENT act
720. HOW WOULD YOU TREAT A SPILLAGE THAN 30 ML BLOOD? - 10 000 ppm sodium
hypochlorite
721. If amalgam gets contaminated with moisture, the most uncommon result is:**
A. Blister formation
B. Post operative pain
C. Secondary caries
D. Lower compressive strength
722.
Concerning local anaesthetics which one??
1. has the longest duration of action? bupivacaine
2.has shortest duration of action? .mepivacaine
3.is commonly used for topical anaesthetics? lignocaine 5%, benzocaine, tetracaine
4.penetrates bone the most? .articaine
5.causes methaemoglobinaemia? prilocaine
723. BP OF A HEALTHY 70 years man?

What increases in healthy old person? Systolic or diastolic


724. Slowest voltage gated channel? – calcium
725. A patient whose hands fell warm and moist is MOST likely to be suffering from:**
A. Anxiety
B. Congestive cardiac failure
C. Thyrotoxicosis
726. What type of fracture ressults if patient elbowed on body of mandible? - opposite condyle
727. A MAN HAS DIET CONTROLLED TYPE 2 DIABETES.HE CONSUMES 30 GLASSES
OF ALCOHOL AND SMOKES 40 CIGARETTES PER DAY.HE IS OTHERWISE
HEALTHY.WHAT MAY BE THE REASON OF DRY MOUTH? – dehydration
728.
729. Cells found in herpetic stomatitis and lichen planus? -  T lymphocytes 
730. what protien has I,T and C subcomponents in muscle fibers? – Troponin
731. Which cells r seen in TB and RA? - In TB langhan's cells and epitheloid cells, in RA
lymphocytes

732. Which hepatitis doesn’t become chronic? - hep A


733. Sharp shooting pain radiating to shoulders? - Angina
734. Angioneurotic edema is characterized by which esterase inhibitor? - C1
735. Multiple lumps on neck, they are sensitive, and there skin is pigmented. Daignosis? - von
Recklinghausen’s i.e. neurofibromatosis

736. At which angle to the external surface of proximal cavity walls in a class II preparation for
amalgam should be finished
A. An acute angle
B. An obtuse angle
C. A right angle
D. An angle of 45°

737. If the sealant of bonding agent is not placed on part of enamel that has been etched by an acid
solution; you would expect:
A. Arrest of enamel carries by organic sulphides
B. The enamel is to return to normal within 7 days
C. Continued enamel declassification in the etched area
D. Slight attrition of the opposing tooth

738. A major difference between light cured and chemical cured composite is that during setting or
in function the light cures material tends to:
A. Seal the margins better and completely
B. Exhibit less wear on time
C. Undergo greater colour change
D. Shrink rapidly
E. Posses greater fracture toughness
q
739. Which component of RPD provides indirect retention? - Occlusal and cingulum rest they are
indirect retainers

740. Which of the following may be caused by newly placed restoration which interferes with the
occlusion
A. Apical abscess
B. Pulpal necrosis
C. Apical Periodontitis

741. Electrical pulp testing is least useful in /or does not detect in some papers/ ,

A. Traumatised teeth
B. Just erupted teeth
C. Multi-rooted teeth
D. Capped teeth
E. Necrotic pulp

742. why is handpiece inclined lingually during cavity preparation of lower first premolar
A.to avoid fracture of tooth
B.to provide better retention
C.to avoid cutting the marginal ridge
D.to avoid undermining the lingual cusp
E.for better access

743. after the age of 6 years, the greatest increase in the size of the mandible occurs:
A. At the symphysis
B. Between canines
C. Distal to the first molar

744. The effects of tooth removal in healthy individuals can show as,
A. Loss of contacts
B. Slight tilting
C. Pocket formation
D. TMJ problem
E. All of the above

745. FLUORIDE SUPPLEMENT FOR A 4 YEAR CHILD,WHEN WATER FLUORIDE LEVEL


IS LESS THAN 0.1?
Toothpaste 1350-1500ppm pea-size
Fluoride varnish 2.2%F twice yearly

746. Which immunoglobin will increase in gingival inflammation? – Igg


747. what is the percentage of plaque in the uk? - plaque...66%
what is the percentage of calculus in the uk? - calculus...69%
748. The use of physical barriers in Guided Tissue Regeneration is to do what?
A.
Retard apical migration of epithelium.
B.
Exclude gingival connective tissue from the healing wound
C.
Both
D.
Neither

749. Which of the following inflammatory cells in periodontitis transform into plasma cells upon
secondary exposure to an antigen?
a. Neutrophils
b. T-Lymphocytes
c. B-Lymphocytes
d. Macrophages
e.Mast Cells

750. Which of these muscles may affect the borders of mandibular complete denture,
A. Mentalis
B. Lateral pterygoid
C. Orbicularis oris
D. Levator angulioris
E. Temporal

751. For dental caries to progress in dentine, ?


A. The dentine must contain soluble collagen
B. Enamel must contain glycoproteins
C. Diet must contain simple carbohydrate
D. Diet must contain polysaccharides
E. Pulp must contain complement

752. Which is more retentive form for anterior bridge


A. ¾ partial veneer crown
B. Class V inlay
C. Pinlay Veneer
D. Class III inlay with pins

753. When primary molars are prepared for stainless steel crowns should the depth for reduction of
the proximal surface be similar to the depth of the buccal and lingual surfaces?
A. Yes; reduction of all wall is similar for best retention
B. No, proximal reduction is greater to allow the crown to pass the contact area
C. No, the buccal surfaces has the greatest reduction to remove the cervical bulge
D. Yes, all undercuts are uniformly removed so that the steel crown can be seated
E. No, because of lateral constriction, the lingual surface needs greatest reduction

754. Upper anterior absent in a 22 year old sports person who wears a mouth guard, what kind of
prosthesis would you provides? - Although using mouth guard , dental implant is the first choice.

755. Upper anterior lost in a 13 year old, what kind of prosthesis would you provide? - A lost cental
in childhood requires space maintainer as a removable aplliance as mostly all upper teeth have been
erupted at 13 yrs old.

756. Von Willebrand's disease is the most common bleeding disorder. pg-146 Scully.

757. Black stain of amalgam tattoo is caused by  Mercury or silver?

758. 1 most common reason of failure of proximal amalgam filling? - isthmus fracture
759. What is more likely to happen to an interproximal composite filling rather than amalgam it's
feature or undercontouring? - polymerization shrinkage
760. Largest possible cavosurface angle? - 110 degree

761. What cement you use to glue alumina core ceramic crown it's panavia or resin modified gic? -
Resin modified gic
762. Pigmented fissure, which is the best way of caries diagnosis? - investigate the area with round
bur

763. Whom to contact if there is 20s of extra radiation dose? - radiation protection advisor

764. Regeneration of odontoblast after a pulp pathology is from


A.Regenerate from the left odontoblast
B.Regenerate from undifferentiated mesenchymel cells
C.Regenerate from ectodermal cells
D.Regenerate from the undelying nectrotic tissue

765. In children a disease with enzyme deficiency


A.hypohpospatesia
B.Cyclic neutropenia
C.Juvienile periodontits

767. In a flouridated toothpaste with 0.304% monoflourophospate the ampont of


flouride ions
A.400ppm
B.1000ppm
c.1500ppm
D.4000ppm

768. Compared to natural teeth the displacement occurring in the denture


supporting mucosa under functional load
A.Ten times more greater
B.Ten times less greater
C..One half

769. Normal prothrombin time & increased partial thromboblastin time seen in
A.Haemophilia A
B.Thrombocytic pupura
C.Leukemia
D.Von willibrand disease

770. When treating a tooth with a non-vital pulp with a fistula presented;
fistula should be treated by
A. Surgical incision
B. Antibiotic coverage
C. The usual root canal procedures for non-vital teeth and no special procedures for fistula

771. The major cause of mentalis muscle hyperactivity is


A. Class II Division I
B. Tongue thrust

772. What is TRUE


A. Boiling point of acrylic > boiling point of water
B. Boiling point of acrylic is similar to that of water
C. Boiling point of acrylic < boiling point of water

773. The presence of antibodies against which antigens of the following indicates immunity against
Hepatitis B
Hbs antigen
Hbe antigen
Hbc antigen

774. What component of the diet is the most important cause of erosion of teeth- cola, alcohol, etc
775. if x ray beam not horizental to film,,which problem will arise? - superimposition /overlapping
of adjacent structures
776. Painful unilateral parotid sweeling which is acute and having no fluctuency..which of the
folllowing is the cause
a.mumps
b.bacterial seladenitis

777. Anterior lower central incisers avulsed long time ago..what would be the treatment
Ans. If extraalveolar period >24hrs,leave n consider for space maintainence

778. There is assault and tooth is palataly displaced and pt come after 6 days what would be the
treatment. - orthodontic repositioning
779. Pt on immunosuppresive drug develops lesions on the half of palate...
What virus is the cause? - varicella zoster virus
780. Trauma to tooth with closed apex. In which one the tooth is more likely to maintain vitality?
concussion, subluxation, intrusive luxation, avulsion, luxation
781. Sign of asthma: wheezing
782. Name of the first formed dentine: mantle?
783. Where does Hertwig's epithelial root sheath come from? - .cervical loop of enamel organ
784. Buffering in saliva: bicarbonate ions
785. Pt with epitaxis and desqamative gingivitis.... Which condition? - mucous membrane
pemphigoid

786. Which answer is correct?


Approximately 75% of teeth have canal aberrations in the:
a.Apical 1 mm of the root
b.Apical 3 mm of the root
c.Middle 1/3 of the root
d.Furcation area
e.None of the above

Answer: b. Apical 3 mm of the root

787. 1)which cells proliferate in later stage of life


odontoblast
cementoblast
rest cells of malaze
Undifferentiated mysenchymal cells

788. x-ray for trauma of chin... - PA mandible. OPG

789. For retention of post which is imp?


 Diameter
 Size
 Luting agent
 Type of post
 Length

790. .rct treated tooth ,post placed which is the best option to restore tooth . Nayar tech used
1. Gold crown
2. Gold inlay
3. Pjc

Ans. 1) In post,the most important factor is the lenght


2) Nayar core usually done on molar with amalgam or composite,gold crown is the best choice
 
791. The effective dose depends on various parameters like voltage, collimator used and speed of
film used: As you will notice below the effective dose varies with the type of film and collimator
used.
two bite-wings-70kv-rectangular collimation- F speed film@ effective dose is 0.0016mSv
two bite-wings-70kv-rectangular collimation- E speed film@ effective dose is 0.002mSv
two bite-wings-70kv-round collimation- D speed film@ effective dose is 0.008mSv.

792. In the construction of a full veneer gold crown, future recession of gingival tissue can be
prevented or at least minimised by,

A. Extension of the crown 1 mm under the gingival crevice

B. Reproduction of normal tooth incline in the gingival one third of the crown

C. Slight over contouring of the tooth in the gingival one fifth of the crown

D. Slight under contouring of the tooth in the gingival one fifth of the crown

Ans.B

793. In planning and construction of a cast metal partial denture the study cast

A. facilitates the construction of custom trays

B. minimizes the need for articulating

C. provides only limited information about inter ridge distance, which is best assessed clinically
D. can be used as a working cast when duplicating facilities are not available

Ans. D

794. Picture - Patient complains about her lower canines (last teeth), she has a nice partial
denture, canines pocketing less then 2mm and 50% bone support, what treatment is the best
– implant-based overdenture, extraction and wait for healing to make a new denture,
extraction and immediate dentures, RCT if Canines treatable and overdenture

795. A 43year old patient has missing upper right first premolar and molar.He hasgood oral
hygiene and requests a fixed replacement for these teeth. The other teeth on the same side are
all moderately restored with MOD amalgam restorations and are vital, except the canine,has
a very large restoration and is root-filled. He has group function.
Radiographs show a large sinus cavity and no peri-apical pathology.

What would be the restoration of choice for replacement of the missing teeth?
A. Implant supported crowns
B. A conventional fixed bridge using the 7 and 5 as abutments
C. Two conventional cantilevered bridges, using the 7 and 3 as abutments
D. A resin-bonded bridge, using the 7 and 5 as abutments
E. A conventional fixed-moveable bridge using the 7 and 5 as abutments

796. A woman has an ulcer for about 3 weeks and she says that it is there since the new lower
denture was placed. What would you do?
Adjust the lower denture and wait for 3 weeks for the ulcer to heal, do incision biopsy immediately,
do excision, biopsy, send her to oncologist.

Answer
If the ulcer is caused due to new denture, adjust the denture and wait. Give ora base or similar for
ulcer healing.
Or
If the ulcer is due to other cause Do incision biopsy if lesion is large (Incision biopsy or excesional
biopsy depends on the size of the lesion)

797. What 5 things would you check in a crown which comes back from the lab before you call
a patient?
Answer

1. Check the lab card prescription and compare the cast and crown if it belongs to the same patient.
2. Check if its done according to your specification e.g. metal crown, ceramic etc.
3. Check for any faults like fracture, fit.

798. Give two reasons why the crown would not fit the patient if it seats the model perfectly

Answer
1. Undercuts
2. Migration of the abutments

799. You try but you cannot manage to seat the crown. What would you do- 3 things
before/when you are sending it to the lab

Answer
1. Correct the undercuts and occlusion
2. Re record the impression
3. Instructions to the lab technician
800. Hepatitis

Hepatitis A Virus

The hepatitis A virus, or HAV, is relatively hardy. In good conditions, it can survive
outside the body for months. HAV can survive certain acids and some heat. For a
period of time and under certain conditions, HAV can survive in sea water, dried
feces and live oysters.

Hepatitis B Virus

The hepatitis B virus, or HBV, can still be infectious for up to a week outside the
body.

How long does the hepatitis C virus survive outside the body?

The hepatitis C virus can survive outside the body at room temperature, on environmental surfaces,
for at least 16 hours but no longer than 4 days.

Hepatitis D Virus

This virus needs help from the hepatitis B virus to be infectious, so it only infects someone with
hepatitis B.

Hepatitis E Virus

This virus is spread similar to the hepatitis A virus and causes acute disease similar to the others.

801. non synovial joint


Cartilaginous joints are connected entirely by cartilage (fibrocartilage or hyaline).Cartilaginous
joints allow more movement between bones than a fibrous joint but less than the highly mobile
synovial joint. An example would be the joint between the manubrium and the sternum.
Cartilaginous joints also forms the growth regions of immature long bones and the intervertebral
discs of the spinal column.
cartilaginous joints (synchondroses and symphyses)

Fibrous joints are connected by dense connective tissue, consisting mainly of collagen.
Types
These joints are also called "fixed" or "immoveable" joints, because they do not move. These joints
have no joint cavity and are connected via fibrous connective tissue. The skull bones are connected
by fibrous joints.
* Sutures are found between bones of the skull. In fetal skulls the sutures are wide to allow slight
movement during birth. They later become rigid (synarthrodial).
* Syndesmoses are found between long bones of the body, such as the radius and ulna in forearm
and the fibula and tibia in leg. Unlike other fibrous joints, syndesmoses are moveable
(amphiarthrodial), albeit not to such degree as synovial joints.
* Gomphosis is a joint between the root of a tooth and the sockets in the maxilla or mandible

A Synovial joint, also known as a diarthrosis, is the most common and most movable type of joint
in the body of a mammal. As with most other joints, synovial joints achieve movement at the point
of contact of the articulating bones.

examples for Synovial joint.


the carpals of the wrist, acromioclavicular joint

elbow (between the humerus and the ulna

http://www.teachpe.com/anatomy/joints.php

802.
 

Cranial Nerves
.
There are 12 pairs of cranial nerves. These nerves arise from the brain and brain stem,
carrying motor and or sensory information.

 
 
 
 

 
 

Cranial nerve
I: Olfactory
nerve
The olfactory nerve is
composed of axons from
the olfactory receptors in
the nasal sensory
epithelium. It carries
olfactory information
(sense of smell) to the
olfactory bulb of the
brain. This is a pure
sensory nerve fiber.
 

Cranial nerve II: Optic nerve


The optic nerve is composed of axons of the ganglion cells in
the eye. It carries visual information to the brain. This is a pure
sensory nerve fiber. This nerve travels posteromedially from the
eye, exiting the orbit at the optic canal in the lesser wing of the
sphenoid bone. The optic nerves join each other in the middle
cranial fossa to form the optic chiasm.

Cranial nerve III: Oculomotor nerve


The oculomotor nerve is composed of motor axons coming from
the oculomotor nucleus and the edinger-westphal nucleus in the
rostral midbrain located at the superior colliculus level. This is a
pure motor nerve. It provides somatic motor innervation to four
of the extrinsic eye muscles: the superior rectus, inferior rectus,
medial rectus, and the inferior oblique muscles. It also
innervates the muscles of the upper eyelid and the intrinsic eye
muscles (the pupillary eye muscle.) Together, CN III, CN IV and CN VI control the six
muscles of the eye.

Cranial nerve IV: Trochlear nerve


The trochlear nerve provides somatic motor innervation to the superior oblique eye muscle.
This cranial nerve originates at the trochlear nucleus located in the tegmentum of the
midbrain at the inferior colliculus level and exits the posterior side of the brainstem. It is also
a pure motor nerve fiber.

Cranial nerve V: Trigeminal nerve


The trigeminal is the largest cranial nerve . It provides sensory
information from the face, forehead, nasal cavity, tongue, gums
and teeth (touch, and temperature) and provides somatic motor
innervation to the muscles of mastication or “chewing”.
This cranial nerve has 3 branches: the ophthalmic, maxillary and mandibular branches.

It is composed of both sensory and motor axons. The sensory fibers are located in the
trigeminal ganglion and the motor fibers project from nuclei in the pons.

Cranial nerve VI: Abducens nerve


The abducens nerve carries somatic motor innervation to one of the extrinsic eye muscles,
the lateral rectus muscle. It is another pure motor nerve fiber and originates from the
abducens nucleus located in the caudal pons at the facial colliculus level.

Cranial nerve VII: Facial nerve


The facial nerve carries somatic motor innervation to the many
muscles for facial expression. It carries sensory information form
the face (deep pressure sensation) and taste information from the
anterior two thirds of the tongue. It arises at the pons in the
brainstem and it emerges through openings in the temporal bone
and stylomastoid foramen and has many branches. It is
composed of both sensory and motor axons.

Cranial nerve VIII: Vestibulocochlear


nerve
The vestibulocochlear nerve innervates the hair cell receptors of
the inner ear. It carries vestibular information to the brain from
the semicircular canals, utricle, and saccule providing the sense
of balance. It also carries information from the cochlea providing
the sense of hearing. This cranial nerve branches into the
Vestibular branch (balance) and the cochlear branch (hearing).
The cochlear fibers originate from the spiral ganglion. It is pure sensory nerve fiber.

Cranial nerve IX: Glossopharyngeal


nerve
The glossopharyngeal nerve innervates the pharynx (upper part
of the throat), the soft palate and the posterior one-third of the
tongue. It carries sensory information (touch, temperature, and
pressure) from the pharynx and soft palate. It carries taste
sensation from the taste buds on the posterior one third of the
tongue. It provides somatic motor innervation to the throat
muscles involved in swallowing. It provides visceral motor
innervation to the salivary glands. This cranial nerve also
supplies the carotid sinus and reflex control to the heart . It is
composed of both sensory and motor axons and originates from the nucleus ambiguous in
the reticular formation of the medulla.

Cranial nerve X: Vagus nerve


The vagus nerve consists of many rootlets that come off of the
brainstem just behind the glossopharyngeal nerve. The branchial
motor component originates from the nucleus ambiguous in the
reticular formation of the medulla. The visceral component
originates from the dorsal motor nucleus of the vagus located in
the floor of the fourth ventricle in the rostral medulla and in the
central grey matt er of the caudal medulla. It is the longest
cranial nerve innervating many structures in the throat, including
the muscles of the vocal cords, thorax and abdominal cavity. It provides sensory information
(touch, temperature and pressure) from the external auditory meatus (ear canal) and a portion
of the external ear. It carries taste sensation from taste buds in the pharynx. It also provides
sensory information from the esophagus, respiratory tract, and abdominal viscera (stomach,
intestines, liver, etc.). It provides visceral motor innervation to the heart, stomach, intestines,
and gallbladder. It is part of the ANS, the parasympathetic branch. It is composed of both
sensory and motor axons. Other parasympathetic ganglia include CN III , CN VII and CN IX
.
 

Cranial nerve XI: Spinal Accessory


nerve
The spinal accessory nerve has two branches. The cranial branch
provides somatic motor innervation to some of the muscles in
the throat involved in swallowing. This cranial branch is
accessory to CN X, originating in the caudal nucleus ambiguous,
with the fibers of the cranial root traveling the same extracranial
path as the branchial motor component of the vagus nerve. The
spinal branch provides somatic motor innervation to the
trapezius muscles, providing muscle movement for the upper shoulders head and neck. It is
pure motor nerve fiber.

Cranial nerve XII: Hypoglossal nerve


The hypoglossal nerve provides somatic motor innervation to the muscles of the tongue.
This pure motor nerve originates from the hypoglossal nucleus located in the tegmentum of
the medulla.

Here are a few pneumonic devices to help you remember the cranial nerves and which
ones carry sensory, motor or both!
Mnemonic
Mnemonic 1 Mnemonic 2
Cranial Nerve 3*(Sensory
(nerves) (alternate)
vs. Motor)
I. Olfactory On Oh Some*
II. Ocular Old Oh Say
III. Oculomotor Olympus' Oh Marry*
IV. Trochlear Towering To Money
V. Trigeminal Tops Touch But*
VI. Abducens A And My
VII. Facial Fin Feel Brother
VIII. Vestibulocochlear/Acoustic And A Says
IX. Glossopharyngeal German Good Big
X. Vagus Viewed Vein Business
XI. Accessory (Spinal Accessory) Some Ah Makes
XII. Hypoglossal Hops Heaven Money
*S: sensory *M: motor *B: both

803.  You can either calculate your BMI using Imperial measurements (the kind used in America)
or metric measurements.

IMPERIAL METHOD
For this method, your weight is in pounds, and your height is in inches.
1 foot=12 inches

BMI=weight in pounds*703/height in inches (squared)

For example, you weigh 155 pounds and are 70 inches tall.

BMI=155*703/70 (squared)
BMI=108965/4900
BMI=22.2377 or 22.2

METRIC METHOD
For this method, your weight is in kilograms, and your height is in meters.
100 cm=1 meter

BMI=weight in kilograms/height in meters (squared)

For example, you weigh 65 kilograms at 170 centimeters, or 1.70 meters.

BMI=65/1.7 (squared)
BMI=65/2.89
BMI=.34602 or 34.6

804. Wikipedia: hypersensitivity


Hypersensitivity
Classification & external
resources
ICD-10 T78.4
ICD-9 995.3
DiseasesDB 28827
MeSH D006967
Hypersensitivity refers to undesirable (damaging, discomfort-producing and sometimes fatal)
reactions produced by the normal immune system. Hypersensitivity reactions require a pre-
sensitized (immune) state of the host. The four-group classification was expounded by P. H. G. Gell
and Robin Coombs in 1963.[1]

Type 1 - immediate (or atopic, or anaphylactic)


Main article: Allergy

Type 1 hypersensitivity is an allergic reaction provoked by reexposure to a specific type of antigen


referred to as an allergen.[2] Exposure may be by ingestion, inhalation, injection, or direct contact.
The difference between a normal immune response and a type I hypersensitive response is that
plasma cells secrete IgE. This class of antibodies binds to Fc receptors on the surface of tissue mast
cells and blood basophils. Mast cells and basophils coated by IgE are "sensitized." Later exposure to
the same allergen, cross-links the bound IgE on sensitized cells resulting in degranulation and the
secretion of pharmacologically active mediators such as histamine, leukotriene, and prostaglandin
that act on the surrounding tissues. The principal effects of these products are vasodilation and
smooth-muscle contraction.

The reaction may be either local or systemic. Symptoms vary from mild irritation to sudden death
from anaphylactic shock. Treatment usually involves epinephrine, antihistamines, and
corticosteroids.

Some examples:

 Allergic asthma
 Allergic conjunctivitis
 Allergic rhinitis ("hay fever")
 Anaphylaxis
 Angioedema
 Atopic dermatitis (eczema)
 Urticaria (hives)
 Eosinophilia
 Penicillin
 cephalosporin

Type 2 - antibody-dependent
In type 2 hypersensitivity, the antibodies produced by the immune response bind to antigens on the
patient's own cell surfaces. The antigens recognized in this way may either be intrinsic ("self"
antigen, innately part of the patient's cells) or extrinsic (absorbed onto the cells during exposure to
some foreign antigen, possibly as part of infection with a pathogen). These cells are recognised by
macrophages or dendritic cells which act as antigen presenting cells, this causes a B cell response
where antibodies are produced against the foreign antigen. An example here is the reaction to
penicillin where the drug can bind to red blood cells causing them to be recognised as different, B
cell proliferation will take place and antibodies to the drug are produced. IgG and IgM antibodies
bind to these antigens to form complexes that activate the classical pathway of complement
activation for eliminating cells presenting foreign antigens (which are usually, but not in this case,
pathogens). That is, mediators of acute inflammation are generated at the site and membrane attack
complexes cause cell lysis and death. The reaction takes hours to a day.

Another form of type 2 hypersensitivity is called Antibody Dependent Cell Mediated Cytotoxicity
(ADCC). Here, cells exhibiting the foreign antigen are tagged with antibodies (IgG or IgM). These
tagged cells are then recognised by Natural Killer (NK) cells and macrophages (recognised via IgG
bound to the cell surface receptor, CD16 (FcγRIII)), which in turn kill these tagged cells.

Some examples:

 Autoimmune hemolytic anemia


 Goodpasture's syndrome
 Erythroblastosis Fetalis
 Pemphigus
 Pernicious anemia (if autoimmune)
 Immune thrombocytopenia
 Transfusion reactions
 Hashimoto's thyroiditis
 Graves' disease (see type V below)
 Myasthenia gravis (see type V below)
 Rheumatic fever
 Hemolytic disease of the newborn

Type 3 - immune complex


In type 3 hypersensitivity, soluble immune complexes (aggregations of antigens and IgG and IgM
antibodies) form in the blood and are deposited in various tissues (typically the skin, kidney and
joints) where they may trigger an immune response according to the classical pathway of
complement activation (see above). There are two stages relating to the development of the
complexes, firstly the complex forms when IgG and IgM antibodies are bound to an antigen, after
this, the complexes can form larger ones which can be cleared by the body. It is at the first stage of
this formation where clearance is not possible and the antigen-antibody complex will spread and
deposit as stated above. The reaction takes hours to days to develop.

Some clinical examples:

 Immune complex glomerulonephritis


 Rheumatoid arthritis
 Serum sickness
 Subacute bacterial endocarditis
 Symptoms of malaria
 Systemic lupus erythematosus
 Arthus reaction
 Farmer's Lung (Arthus-type reaction)

Type 4 - cell-mediated (Delayed-Type Hypersensitivity, DTH)


See also: Cell mediated immunity

Type 4 hypersensitivity is often called delayed type as the reaction takes two to three days to
develop. Unlike the other types, it is not antibody mediated but rather is a type of cell-mediated
response.

CD8+ cytotoxic T cells and CD4+ helper T cells recognise antigen in a complex with either type 1
or 2 major histocompatibility complex. The antigen-presenting cells in this case are macrophages
which secrete IL-12, which stimulates the proliferation of further CD4+ T cells. CD4+ T cells
secrete IL-2 and interferon gamma, further inducing the release of other Type 1 cytokines, thus
mediating the immune response. Activated CD8+ T cells destroy target cells on contact while
activated macrophages produce hydrolytic enzymes and, on presentation with certain intracellular
pathogens, transform into multinucleated giant cells.

Some clinical examples:

 Contact dermatitis (poison ivy rash, for example)


 Temporal arteritis
 Symptoms of leprosy
 Symptoms of tuberculosis
 Transplant rejection
 Coeliac disease

Metals

Type 5 - stimulatory
This is an additional type that is sometimes (often in Britain) used as a distinction from Type 2.[3]

Instead of binding to cell surface components, the antibodies recognize and bind to the cell surface
receptors, which either prevents the intended ligand binding with the receptor or mimics the effects
of the ligand, thus impairing cell signalling.

Some clinical examples:

 Graves' disease
 Myasthenia gravis

Typ Alternative names Often mentioned Mediators


e disorders
I Allergy (immediate)  Atopy IgE
 Anaphylaxis
 Asthma

II Cytotoxic, antibody-dependent  Autoimmune  IgM or


hemolytic IgG
anemia  (Complem
 Thrombocytop ent)
enia
 Erythroblastos
is fetalis
 Goodpasture's
syndrome
 Graves'
disease *see
type V
explanation
below
 Myasthenia
Gravis *see
type V
explanation
below

III Immune complex disease  Serum  IgG


sickness  (Complement)
 Arthus
reaction
 Systemic
lupus
erythematosus
(SLE)

IV http://en.wikipedia.org/wiki/  Contact T-cells


Type_IV_hypersensitivityDelayed-type dermatitis
hypersensitivity[2] [3](DTH), cell-mediated immune  Mantoux test
memory response, antibody-independen  Chronic
transplant
rejection
 Multiple
sclerosis [4]

V http://en.wikipedia.org/wiki/  Graves'  IgM or


Type_V_hypersensitivityAutoimmune disease, receptor disease IgG
mediated (see below)  Myasthenia  (Complem
Gravis ent)

805. Inlays and Onlays

Inlays and Onlays are indirect dental restorations that reinforce an existing tooth that is too
damaged to support a filling, but not damaged enough to require a crown. An inlay is
placed on the chewing surface between the cusps of the tooth. It is an intracoronal
preparation. Whereas an onlay is an indirect restoration which is partly intracoronal and
partly extra coronal that covers all cusps of posterior teeth. The main advantage of inlays
and onlays is that the tooth structure is saved and it is more economical as compared to
the crowns. The dental office time and the numbers of visits required to dentist office are
also less. The composite inlays take less time and are more economical than the ceramic
inlays and onlays but the ceramic inlays and onlays are more durable as they are stronger
than the composite inlays. Both the inlays and onlays are esthetic restorations which can
replace the metallic restorations.  

Various materials can be used for preparing inlays and onlays. These can be
 Cast metal
 Gold
 Composite
 Porcelain 
GOLD INLAYS
 
A gold inlay is indicated when there is extensive caries affecting the tooth that a
satisfactory restoration cannot be fabricated using silver amalgam. Specific indications are:
 
1.    When the cavity width does not exceed 1/3rd the intercuspal distance.
2.    In case of extensive proximal caries involving the buccal and lingual line angles of the
tooth.
3.    In situations where the proximal margins are extending subgingivally. This is because well
polished gold alloys are compatible with the periodontium.
4.    Whenever there is need to establish ideal contact, contours and embrasures. The
mesiodistal dimension of cast gold inlays can be extended to establish good contact.
5.    In a grossly carious tooth where one or more but not all cusps need coverage.
6.    In patients with good oral hygiene and low caries incidence.
7.   When there are other gold castings present in the mouth.

 
Contraindications for Gold Inlays
 
1. Gold inlays are used as an abutment for a fixed or a removable prosthesis. In these
situations the gold inlays are not strong enough retainer.
2. They are contraindicated in post endodontic restorations as they can wedge and fracture
the remaining tooth structure.
3. In young permanent tooth gold inlays are avoided as there are increased chances of
iatrogenic pulp exposure.
4. in patients with high plaque and caries incidence. Gold inlays should be avoided as there
may be a greater tendency for recurrent caries.
5. When the adjacent or opposing teeth have dissimilar metallic restorations cast gold inlays
should be avoided in order to prevent galvanism.
6. When cost is major factor for the patient, gold inlays are not indicated.
7. Gold inlays are not preferred in case of grossly destroyed teeth with weak cusps.
  
TOOTH COLOURED INLAYS AND ONLAYS      
 
Tooth colored inlays and onlays have certain advantage over direct resin composite and
bonded ceramic restorations. These advantages are:
 
1. Highly esthetic.
2. Can be fabricated intraorally or on a cast.
3. Highly successful in larger restorations.
4. Have many materials and techniques.
 

 
Various tooth colored materials are given as under
 
Composite Resin Inlays and Onlays
 
Compared with direct composite resin restoration
 
1. Contours and contacts can be developed outside of the mouth. If contact is
inadequate, it can be easily corrected prior to cementation.
2. Polymerization shrinkage should be less because they are polymerized before
cementation.
3. Less micro leakage.
4. Greater strength and hardness.
5. Less post-operative sensitivity. 
Compared with ceramic
 
1. Less abrasive to opposing tooth structure.
2. Repairable.
3. Cheaper.
 Advantages of composite inlays:
1. High esthetics.
2. Better control of the contact areas.
3. Excellent marginal adaptation.
4. Reduced or no laboratory fee if done in office.
5. Ready reparability of material intraorally.
6. Compensation for complete polymerization shrinkage by curing the material outside
the mouth.
7. Increased composite resin strength because of the heat curing process.
 
 
Disadvantage of composite inlays
 
1. Wear faster.
2. Less stable.
3. Higher cost time.
4. Difficult to modify extrinsic color chair side.
5. More tooth reduction to create path of insertion.
 
Indication of composite inlay
 
1. Replacement of large amalgam restorations.
2. Replacement of direct resin composite restorations in premolar and molar.
3. Large caries.
4. Esthetics. 
Contraindication of composite inlay
 
1. Heavy occlusal forces.
2. Inability to maintain dry operative field.
3. Deep subgingival preparations. 
Porcelain Inlay and Onlay
 
Advantages:
1. Highly esthetic.
2. Acceptable marginal fit.
3. Conservation of tooth structure.
4. Less occlusal wear.
5. Highly technique sensitive.
6. Low thermal conductivity.
7. Low coefficient of thermal expansion.
8. Biocompatibility. 

 
Disadvantages:
1. High cost.
2. Need for special and laboratory equipment.
3. Fabrication and cementation processes are highly technique sensitive.
4. Ceramic inlays are brittle and can fracture during try in or cementation.
5. The increased hardness of ceramics can wear the opposing teeth. 

Indications
1 .High esthetic demands.
2. Replace moderate to large existing restoration.
3. Fractured tooth/restoration.
4. Moderate to large primary caries.
5. Patients with good oral hygiene.
6. When there is no excessive attrition.
7. Where access and isolation are easy.
8. When there is no excessive undercuts in the preparation. 

Contraindications:
1. Unable to isolate the field.
2. Parafunctional habits like clenching, bruxism, excessive wear.
3. Patients with poor oral hygiene.
5. In case with minimal tooth loss.
6. When there is inadequate enamel left for bonding.
7. When there are marked undercuts in the cavity preparation.
Inlay

Sometimes, a tooth is treatment planned to be restored with an intracoronal restoration, but the
decay or fracture is so extensive that a direct restoration, such as amalgam or composite, would
compromise the structural integrity of the restored tooth by possibly undermining the remaining
tooth structure or providing substandard opposition to occlusal (i.e. biting) forces. In such
situations, an indirect gold or porcelain inlay restoration may be indicated. The following
documents the indirect (out of the mouth) fabrication of a gold inlay. When an inlay is used, the
tooth-to-restoration margin may be finished and polished to such a super-fine line of contact that
recurrent decay will be all but impossible. It is for this reason that some dentists recommend inlays
as the restoration of choice for pretty much any and all filling situations. While these restorations
might be ten times the price of direct restorations, the superiority of an inlay as a restoration in
terms of resistance to occlusal forces, protection against recurrent decay, precision of fabrication,
marginal integrity, proper contouring for gingival (tissue) health, ease of cleansing and many other
aspects of restorative quality offers an excellent alternative to the direct restoration. For this reason,
some patients request inlay restorations so they can benefit from its wide range of advantages even
when an amalgam or composite will suffice. The only true disadvantage of an inlay is the higher
cost.

Onlays
Additionally, when decay or fracture incorporate areas of a tooth that make amalgam or composite
restorations essentially inadequate, such as cuspal fracture or remaining tooth structure that
undermines perimeter walls of a tooth, an "onlay" might be indicated. Similar to an inlay, an onlay
is an indirect restoration which incorporates a cusp or cusps by covering or onlaying the missing
cusps. All of the benefits of an inlay are present in the onlay restoration. The onlay allows for
conservation of tooth structure when the only other alternative is to totally eliminate cusps and
perimeter walls for restoration with a crown. Just as inlays, onlays are fabricated outside of the
mouth and are typically made out of gold or porcelain. Gold restorations have been around for many
years and have an excellent track record. In recent years, newer types of porcelains have been
developed that seem to rival the longevity of the gold. Either way, if the onlay or inlay is made in a
dental laboratory, a temporary is fabricated while the restoration is custom made for the patient. A
return visit is then required to deliver the final prosthesis. Inlays and onlays may also be fabricated
out of porcelain and delivered the same day utilizing techniques and technologies relating to
CAD/CAM Dentistry

806. Inferior alveolar nerve anaesthesia

Inferior alveolar nerve anaesthesia is a technique for dental anaesthesia, used to cause numbness
to the areas of the face innervated by the inferior alveolar nerve; namely, the lower lip and the teeth
and gingivae of the mandible. This procedure attempts to anaesthetise the inferior alveolar
nerve prior to it entering themandibular foramen.

The inferior alveolar nerve is also known as the infra-alveolar nerve and the inferior dental nerve
and is thus sometimes abbreviated as IAN or IDN. The inferior alveolar nerve is a branch of
the mandibular nerve, which is itself the third branch of the trigeminal nerve.

Area of Numbness

Administration of anaesthesia near the mandibular foramen causes blockage of the inferior alveolar
nerve and the nearby lingual nerve (supplying the tongue). This is why the numbing of the lower
jaw during dental procedures causes patients to lose sensation in:

 their teeth (inferior alveolar nerve block)


 their lower lip and chin (mental nerve block)
 and their tongue (lingual nerve block).
Administration of anaesthetic solution more superior to the mandibular foramen (usually in the
vicinity of the ascending ramus of the mandible), will cause a more profound anaesthesia on that
particular side of the mandible, also involving the buccal nerve, which branches off the inferior
alveolar nerve superior to the mandibular foramen and supplies sensation to the buccal side of the
mandible. This is often referred to as the V3 block or the Gow-Gates block.[1]
Injection techniques

There are a number of techniques that are commonly used to achieve inferior alveolar nerve
anaesthesia. The most commonly used techniques involve an attempted block of an entire portion of
the inferior alveolar nerve:

 Standard mandibular nerve block - The nerve is approached from the opposite side of the
mouth by angling the syringe from the premolars on the opposite side. After piercing the
mucosa and the buccinator muscle between the palatoglossal & palatopharangeal folds until
hitting bone (the ascening ramus), the syringe is drawn backwards slightly and brought parallel
to the width of the ramus, so that the needle lies lateral to the medial pterygoid at the mandibular
foramen.
 Gow-Gates technique - Invented by Australian dentist George A.E. Gow-Gates in the mid-
1970's, the needle is directed at the neck of the condyle just under the insertion of the lateral
pterygoid muscle.[2]
 Vazirani-Akinosi technique - a closed-mouth injection technique, the syringe is "advanced
parallel to the maxillary occlusal plane at the level of themaxillary mucogingival junction."[3]
With the advent of mandibular infiltration with articaine, commonly known in the United States by
the brand name Septocaine, certain dental and oral surgical procedures can now be performed
without blocking an entire portion of the inferior alveolar nerve.

Dangers

If needle is positioned too posteriorly, anaesthetic may be put into parotid


gland (dangerous systemic effects), or paralyse Cranial Nerve VII (7), resulting in Bells Palsy-like
symptoms. Also if the needle is placed too medially the medial pterygoid muscle can be injected,
resulting in trismus.

The sphenomandibular ligament is most often damaged in an inferior alveolar nerve block

807. Autonomic Innervation of the Heart and Vasculature


The heart is innervated by vagal and sympathetic fibers. The right vagus nerve primarily innervates
the SA node, whereas the left vagus innervates the AV node; however, there can be significant
overlap in the anatomical distribution. Atrial muscle is also innervated by vagal efferents, whereas
the ventricular myocardium is only sparsely innervated by vagal efferents. Sympathetic efferent
nerves are present throughout the atria (especially in the SA node) and ventricles, including
the conduction system of the heart.

Cardiac function is altered by neural activation. Sympathetic stimulation increases heart


rate (positive chronotropy),inotropy and conduction velocity (positive dromotropy), whereas
parasympathetic stimulation of the heart has opposite effects.  Sympathetic and parasympathetic
effects on heart function are mediated by beta-adrenoceptors and muscarinic receptors, respectively.

Sympathetic adrenergic nerves travel along arteries and nerves and are found in the adventitia (outer
wall of a blood vessel). Varicosities, which are small enlargements along the nerve fibers, are the
site of neurotransmitter release. Capillaries receive no innervation. Activation of vascular
sympathetic nerves causes vasoconstriction of arteries and veins mediated by alpha-adrenoceptors.

Parasympathetic fibers are found associated with blood vessels in certain organs such as salivary
glands, gastrointestinal glands, and in genital erectile tissue. The release of acetylcholine (ACh)
from these parasympathetic nerves has a direct vasodilatory action (coupled to nitric
oxide formation and guanylyl cyclase activation). ACh release can stimulate the release
of kallikrein from glandular tissue that acts upon kininogen to form kinins (e.g., bradykinin). Kinins
cause increased capillary permeability and venous constriction, along with arterial vasodilation in
specific organs.

Source: http://www.cvphysiology.com/Blood%20Pressure/BP008.htm

808. Skull
Anterior view
Supra-orbital Supra-orbital nerve and vessels
foramen
Infra-orbital foramen Infra-orbital nerve and vessels
Mental foramen Mental nerve and vessels
Lateral view
Zygomaticofacial Zygomaticofacial nerve
foramen
Superior view
Parietal foramen Emissary veins
Inferior view
Incisive foramina Nasopalatine nerve; sphenopalatine vessels
Greater palatine Greater palatine nerve and vessels
foramen
Lesser palatine Lesser palatine nerves and vessels
foramina
Pterygoid canal Pterygoid nerve and vessels
Foramen ovale Mandibular nerve [V3]
Foramen spinosum Middle meningeal artery
Foramen lacerum Filled with cartilage
Carotid canal Internal carotid artery and nerve plexus
Foramen magnum Continuation of brain and spinal cord; vertebral arteries and nerve plexuses; anterior
spinal artery; posterior spinal arteries; roots of accessory nerve [XI]; meninges
Condylar canal Emissary veins
Hypoglossal canal Hypoglossal nerve [XII] and vessels
Jugular foramen Internal jugular vein; inferior petrosal sinus
Stylomastoid Facial nerve [VII]
foramen
809. Cranial nerves

Components
Exit from
Nerve Afferent Efferent skull Function
Olfactory nerve SA Cribriform Smell
[I] plate of
ethmoid bone
Optic nerve [II] SA Optic canal Vision
Oculomotor nerve GSE, Superior GSE-innervates levator palpebrae superioris,
[III] GVE orbital fissure superior rectus, inferior rectus, medial rectus, and
inferior oblique muscles
GVE-innervates sphincter pupillae for pupillary
constriction; ciliary muscles for accommodation of
the lens for near vision
Trochlear nerve GSE Superior Innervates superior oblique muscle
[IV] orbital fissure
Trigeminal nerve GSA BE Superior GSA-sensory from: ophthalmic division [V1]-eyes,
[V] orbital conjunctiva, orbital contents, nasal cavity, frontal
fissure- sinus, ethmoid sinus, upper eyelid, dorsum of nose,
ophthalmic anterior part of scalp; maxillary nerve [V2]-dura in
division [V1] anterior and middle cranial fossae, nasopharynx,
Foramen palate, nasal cavity, upper teeth, maxillary sinus,
rotundum- skin covering the side of the nose, lower eyelid,
maxillary cheek, upper lip; mandibular division [V3]-skin of
nerve [V2] lower face, cheek, lower lip, ear, external acoustic
Foramen meatus, temporal fossa, anterior two-thirds of
ovale- tongue, lower teeth, mastoid air cells, mucous
mandibular membranes of cheek, mandible, dura in middle
division [V3] cranial fossa BE-innervates temporalis, masseter,
medial and lateral pterygoids, tensor tympani, tensor
veli palatini, anterior belly of digastric, and
mylohyoid muscles
Abducent nerve GSE Superior Innervates lateral rectus muscle
[VI] orbital fissure
Facial nerve [VII] GSA, GVE, Internal GSA-sensory from external acoustic meatus, skin
SA BE acoustic posterior to ear
meatus SA-taste from anterior two-thirds of tongue
GVE-innervates lacrimal gland, submandibular and
sublingual salivary glands, and mucous membranes
of nasal cavity, hard and soft palates
BE-innervates muscles of face (muscles of facial
expression) and scalp derived from the second
pharyngeal arch, and stapedius, posterior belly of
digastric, stylohyoid muscles
Vestibulocochlear SA Internal Vestibular division-balance
nerve [VIII] acoustic Cochlear division-hearing
meatus
Glossopharyngeal GVA, GVE, Jugular GVA-sensory from carotid body and sinus, posterior
nerve [IX]* SA BE foramen one-third of tongue, palatine tonsils, upper pharynx,
and mucosa of middle ear and pharyngotympanic
tube
SA-taste from posterior one-third of tongue
GVE-innervates parotid salivary gland
BE-innervates stylopharyngeus muscle
Vagus nerve [X]* GSA, GVE, Jugular GSA-sensory from skin posterior to ear and external
GVA, BE foramen acoustic meatus, and dura in posterior cranial fossa
SA GVA-sensory from aortic body chemoreceptors and
aortic arch baroreceptors, mucous membranes of
pharynx, larynx, esophagus, bronchi, lungs, heart,
and abdominal viscera of the foregut and midgut
SA-taste from the epiglottis
GVE-innervates smooth muscle and glands in the
pharynx, larynx, thoracic viscera, and abdominal
viscera of the foregut and midgut
BE-innervates one tongue muscle (palatoglossus),
muscles of soft palate (except tensor veli palatini),
pharynx (except stylopharyngeus), and larynx
Accessory nerve BE Jugular Innervates sternocleidomastoid and trapezius
[XI] foramen muscles
Hypoglossal nerve GSE Hypoglossal Innervates hyoglossus, genioglossus, and
[XII] canal styloglossus muscles and all intrinsic muscles of the
tongue

In human embryology, six pharyngeal arches are designated, but the fifth pharyngeal arch never
develops. Each of the pharyngeal arches that does develop is associated with a developing cranial
nerve or one of its branches. These cranial nerves carry efferent fibers that innervate the
musculature derived from the pharyngeal arch.
Innervation of the musculature derived from the five pharyngeal arches that do develop is as
follows:

 first arch-trigeminal nerve [V3];


 second arch-facial nerve [VII];
 third arch-glossopharyngeal nerve [IX];
 fourth arch-superior laryngeal branch of the vagus nerve [X];
 sixth arch-recurrent laryngeal branch of the vagus nerve [X].
810.
Table 8-7. Muscles of the
face
Muscle Origin Insertion Innervation Function
Orbital group
Orbicularis oculi
-Palpebral part Medial palpebral ligament Lateral palpebral Facial nerve Closes the eyelids
raphe [VII] gently
-Orbital part Nasal part of frontal bone; Fibers form an Facial nerve Closes the eyelids
frontal process of maxilla; uninterrupted ellipse [VII] forcefully
medial palpebral ligament around orbit
Corrugator Medial end of the Skin of the medial Facial nerve Draws the eyebrows
supercilii superciliary arch half of eye-brow [VII] medially and
downward
Nasal group
Nasalis
-Transverse Maxilla just lateral to nose Aponeurosis across Facial nerve Compresses nasal
part dorsum of nose with [VII] aperture
muscle fibers from
the other side
-Alar part Maxilla over lateral incisor Alar cartilage of Facial nerve Draws cartilage
nose [VII] downward and
laterally opening
nostril
Procerus Nasal bone and upper part Skin of lower Facial nerve Draws down medial
of lateral nasal cartilage forehead between [VII] angle of eyebrows
eyebrows producing transverse
wrinkles over bridge
of nose
Depressor septi Maxilla above medial Mobile part of the Facial nerve Pulls nose inferiorly
incisor nasal septum [VII]
Oral group
Depressor Oblique line of mandible Skin at the corner of Facial nerve Draws corner of
anguli oris below canine, premolar mouth and blending [VII] mouth down and
and first molar teeth with orbicularis oris laterally
Depressor labii Anterior part of oblique Lower lip at Facial nerve Draws lower lip
inferioris line of mandible midline; blends with [VII] downward and
muscle from laterally
opposite side
Mentalis Mandible inferior to incisor Skin of chin Facial nerve Raises and protrudes
teeth [VII] lower lip as it
wrinkles skin on chin
Risorius Fascia over masseter Skin at the corner of Facial nerve Retracts corner of
muscle the mouth [VII] mouth
Zygomaticus Posterior part of lateral Skin at the corner of Facial nerve Draws the corner of
major surface of zygomatic bone the mouth [VII] the mouth upward
and laterally
Zygomaticus Anterior part of lateral Upper lip just Facial nerve Draws the upper lip
minor surface of zygomatic bone medial to corner of [VII] upward
mouth
Levator labii Infra-orbital margin of Skin of upper lateral Facial nerve Raises upper lip;
superioris maxilla half of upper lip [VII] helps form nasolabial
furrow
Levator labii Frontal process of maxilla Alar cartilage of Facial nerve Raises upper lip and
superioris nose and upper lip [VII] opens nostril
alaeque nasi
Levator anguli Maxilla below infra-orbital Skin at the corner of Facial nerve Raises corner of
oris foramen mouth [VII] mouth; helps form
nasolabial furrow
Orbicularis oris From muscles in area; Forms ellipse Facial nerve Closes lips;
maxilla and mandible in around mouth [VII] protrudes lips
midline
Buccinator Posterior parts of maxilla Blends with Facial nerve Presses the cheek
and mandible; orbicularis oris and [VII] against teeth;
pterygomandibular raphe into lips compresses
distended cheeks
Other muscles or groups
Anterior Anterior part of temporal Into helix of ear Facial nerve Draws ear upward
auricular fascia [VII] and forward
Superior Epicranial aponeurosis on Upper part of Facial nerve Elevates ear
auricular side of head auricle [VII]
Posterior Mastoid process of Convexity of concha Facial nerve Draws ear upward
auricular temporal bone of ear [VII] and backward
Occipitofrontalis
-Frontal belly Skin of eyebrows Into galea Facial nerve Wrinkles forehead;
aponeurotica [VII] raises eyebrows
-Occipital belly Lateral part of superior Into galea Facial nerve Draws scalp
nuchal line of occipital aponeurotica [VII] backward
bone and mastoid process
of temporal bone

zygomaticus major and zygomaticus minor help produce a smile-


levator labii superioris deepens the furrow between the nose and the corner of the mouth during
sadness
levator anguli oris is more deeply placed and covered by the other two levators and the
zygomaticus muscles, arises from the maxilla, just inferior to the infra-orbital foramen, and inserts
into the skin at the corner of the mouth-it elevates the corner of the mouth and may help deepen the
furrow between the nose and the corner of the mouth during sadness.
811. Cranial nerve lesions
Cranial nerve Clinical findings Example of lesion
Olfactory nerve [I] Loss of smell (anosmia) Injury to the cribriform plate; congenital
absence
Optic nerve [II] Blindness/visual field abnormalities, Direct trauma to the orbit; disruption of
loss of pupillary constriction the optic pathway
Oculomotor nerve Dilated pupil, ptosis, loss of normal Pressure from an aneurysm arising from
[III] pupillary reflex, eye moves down the posterior communicating, posterior
inferiorly and laterally (down and out) cerebral, or superior cerebellar artery;
pressure from a herniating cerebral
uncus (false localizing sign); cavernous
sinus mass or thrombosis
Trochlear nerve Inability to look inferiorly when the Along the course of the nerve around
[IV] eye is adducted (down and in) the brain-stem; orbital fracture
Trigeminal nerve Loss of sensation and pain in the Typically, in the region of the
[V] region supplied by the three divisions trigeminal ganglion, though local
of the nerve over the face; loss of masses around the foramina through
motor function of the muscles of which the divisions pass can produce
mastication on the side of the lesion symptoms
Abducent nerve Inability of lateral eye movement Brain lesion or cavernous sinus lesion
[VI] extending onto the orbit
Facial nerve [VII] Paralysis of facial muscles below the Damage to the branches within the
eye; paralysis of facial muscles parotid gland
Abnormal taste sensation from the Injury to temporal bone; viral
anterior two-thirds of the tongue and inflammation of nerve
dry conjunctivae
Paralysis of contralateral facial Brainstem injury
muscles below the eye
Vestibulocochlear Progressive unilateral hearing loss Tumor at the cerebellopontine angle
nerve [VIII] and tinnitus (ringing in the ear)
Glossopharyngeal Loss of taste to the posterior one-third Brianstem lesion; penetrating neck
nerve [IX] of the tongue and sensation of the soft injury
palate

Vagus nerve [X] Soft palate deviation with deviation of Brainstem lesion; penetrating neck
the uvula to the normal side; vocal injury
cord paralysis
Accessory nerve Paralysis of sternocleidomastoid and Penetrating injury to the posterior
[XI] trapezius muscles triangle of the neck
Hypoglossal nerve Atrophy of ipsilateral muscles of the Penetrating injury to the neck and skull
[XII] tongue and deviation towards the base pathology
affected side; speech disturbance
812. Eye
Innervation
Innervation of the eyelids includes both sensory and motor components.
The sensory nerves are all branches of the trigeminal nerve [V]. Palpebral branches arise from:

 the supra-orbital, supratrochlear, infratrochlear, and lacrimal branches of the ophthalmic


nerve [V1];
 the infra-orbital branch of the maxillary nerve [V2]

Motor innervation is from:

 the facial nerve [VII], which innervates the palpebral part of the orbicularis oculi;
 the oculomotor nerve [III], which innervates the levator palpebrae superioris;
 sympathetic fibers, which innervate the superior tarsal muscle.

Loss of innervation of the orbicularis oculi by the facial nerve [VII] causes an inability to close the
eyelids tightly and the lower eyelid droops away, resulting in a spillage of tears.
Loss of innervation of the levator palpebrae superioris by the oculomotor nerve causes an inability
to open the superior eyelid voluntarily, producing a complete ptosis.
Loss of innervation of the superior tarsal muscle by sympathetic fibers causes a constant partial
ptosis.

Table 8-8. Extrinsic (extra-ocular)


muscles
Muscle Origin Insertion Innervation Function
Levator Lesser wing of Anterior surface of Oculomotor Elevation of
palpebrae sphenoid anterior to tarsal plate; a few fibers nerve [III]- upper eyelid
superioris optic canal to skin and superior superior branch
conjunctival fornix
Superior Superior part of Anterior half of eyeball Oculomotor Elevation,
rectus common tendinous superiorly nerve [III]- adduction, medial
ring superior branch rotation of
eyeball
Inferior Inferior part of Anterior half of eyeball Oculomotor Depression,
rectus common tendinous inferiorly nerve [III]- adduction, lateral
ring inferior branch rotation of
eyeball
Medial Medial part of Anterior half of eyeball Oculomotor Adduction of
rectus common tendinous medially nerve [III]- eyeball
ring inferior branch
Lateral Lateral part of Anterior half of eyeball Abducent nerve Abduction of
rectus common tendinous laterally [VI] eyeball
ring
Superior Body of sphenoid, Outer posterior Trochlear nerve Depression,
oblique superior and medial to quadrant of eyeball [IV] abduction, medial
optic canal rotation of
eyeball
Inferior Medial floor of orbit Outer posterior Oculomotor Elevation,
oblique posterior to rim; quadrant of eyeball nerve [III]- abduction, lateral
maxilla lateral to inferior branch rotation of
nasolacrimal groove eyeball

813. Sphenomandibular ligament


Ans: The irregularly shaped spine of sphenoid, which is the attachment site for the cranial end of
the sphenomandibular ligament.
Immediately anterosuperior to the mandibular foramen is a triangular elevation (the lingula) for
attachment of the mandibular end of the sphenomandibular ligament.

 the stylomandibular ligament passes from the styloid process of the temporal bone to the
posterior margin and angle of the mandible

814. Muscles of mastication attachments


ANS:
Ramus: Most of the lateral surface provides attachment for the masseter muscle.
The coronoid process extends superiorly from the junction of the anterior and superior borders of
the ramus. It is a flat triangular process that provides attachment for temporalis muscle
The condylar process extends superiorly from the posterior and superior borders of the ramus. It
consists of:

 the head of mandible, which is expanded medially and participates in forming the
temporomandibular joint;
 the neck of mandible, which bears a shallow depression (the pterygoid fovea) on its
anterior surface for attachment of the lateral pterygoid muscle.

The medial surface of the ramus of mandible is roughened for attachment of the medial pterygoid
muscle.

815. Movements of the mandible

A chewing or grinding motion occurs when the movements at the temporomandibular joint on one
side are coordinated with a reciprocal set of movements at the joint on the other side. Movements of
the mandible include depression, elevation, protrusion, and retraction (Fig. 8.130):

 depression is generated by the digastric, geniohyoid, and mylohyoid muscles on both sides,
is normally assisted by gravity and, because it involves forward movement of the head of the
mandible onto the articular tubercle, the lateral pterygoid muscles are also involved;
 elevation is a very powerful movement generated by the temporalis, masseter, and medial
pterygoid muscles and also involves movement of the head of the mandible into the
mandibular fossa;
 protraction is mainly achieved by the lateral pterygoid muscle, with some assistance by the
medial pterygoid
 retraction is carried out by the geniohyoid and digastric muscles, and by the posterior and
deep fibers of the temporalis and masseter muscles, respectively

Except for the geniohyoid muscle, which is innervated by the C1 spinal nerve, all muscles that
move the temporomandibular joints are innervated by the mandibular nerve [V3] by branches that
originate in the infratemporal fossa

816.

Muscles of mastication
Muscle Origin Insertion Innervation Function
Masseter Zygomatic arch and Lateral surface of ramus of Masseteric nerve Elevation of
maxillary process of the Mandible from the anterior mandible
zygomatic bone trunk of the
mandibular nerve
[V3]
Temporalis Bone of temporal fossa Coronoid process of Deep temporal Elevation and
and temporal fascia mandible and anterior nerves from the retraction of
margin of ramus of anterior trunk of mandible
mandible almost to last the mandibular
molar tooth nerve [V3]
Medial Deep head-medial Medial surface of mandible Nerve to medial Elevation and
Pterygoid surface of lateral plate near Angle pterygoid from the 'side-to-side'
of pterygoid process mandibular nerve movements of
and pyramidal process [V3] the mandible
of palatine bone;
superficial head-
tuberosity and
pyramidal process of
maxilla
Lateral Upper head-roof of Capsule of Nerve to lateral Protrusion and
pterygoid infratemporal fossa; temporomandibular joint in pterygoid directly 'side-to-side'
lower head-lateral the region of attachment to from the anterior movements of
surface of lateral plate the articular disc and to the trunk of the the mandible
of the pterygoid process pterygoid fovea on the mandibular nerve
neck of mandible [V3] or from the
buccal branch

817. Temporal fossa

Contents: The major structure in the temporal fossa are the temporalis muscle.
Also passing through the fossa is the zygomaticotemporal branches of the maxillary nerve [V2],
which enter the region through foramina on the temporal fossa surface of the zygomatic bone.

818. Infratemporal fossa

Contents: Major contents of the infratemporal fossa include the sphenomandibular ligament,
medial and lateral pterygoid muscles, the maxillary artery, the mandibular nerve [V3], branches of
the facial nerve [VII], and the glossopharyngeal nerve [IX], and the pterygoid plexus of veins.

819. Lingual nerve injury

 A lingual nerve injury proximal to where the chorda tympani joins it in the infratemporal
fossa will produce loss of general sensation from the anterior two-thirds of the tongue, oral
mucosa, gingivae, the lower lip, and the chin.
 If a lingual nerve lesion is distal to the site where it is joined by the chorda tympani,
secretion from the salivary glands below the oral fissure and taste from the anterior two-
thirds of the tongue will also be lost.

820. Pterygopalatine fossa

Contents

The maxillary nerve [V2] and terminal part of the maxillary artery enter and branch within the
pterygopalatine fossa. In addition, the nerve of the pterygoid canal enters the fossa carrying:

 preganglionic parasympathetic fibers from the greater petrosal branch of the facial nerve
[VII];
 postganglionic sympathetic fibers from the deep petrosal branch of the carotid plexus.

The maxillary nerve [V2] is purely sensory. It originates from the trigeminal ganglion in the cranial
cavity, exits the middle cranial fossa, and enters the pterygopalatine fossa through the foramen
rotundum (Fig. 8.146). It passes anteriorly through the fossa and exits as the infra-orbital nerve
through the inferior orbital fissure.

While passing through the pterygopalatine fossa, the maxillary nerve [V 2] gives rise to the
zygomatic nerve, the posterior superior alveolar nerve, and two ganglionic branches. The two
ganglionic branches orizginate from its inferior surface and pass through the pterygopalatine
ganglion.

Postganglionic parasympathetic fibers, arising in the pterygopalatine ganglion, join the general
sensory branches of the maxillary nerve [V2] in the pterygopalatine ganglion, as do postganglionic
sympathetic fibers from the carotid plexus, and the three types of fibers leave the ganglion as
orbital, palatine, nasal, and pharyngeal branches

821. Tongue

Muscles of the tongue


Muscle Origin Insertion Innervation Function
Intrinsic
Superior Submucosal Muscle fibers pass Hypoglossal Shortens tongue;
longitudinal (just connective tissue at forward and nerve [XII] curls apex and sides
deep to surface of the back of the obliquely to of tongue
tongue) tongue and from the submucosal
median septum of connective tissue and
the tongue mucosa on margins
of tongue
Inferior Root of tongue Apex of tongue Hypoglossal Shortens tongue;
longitudinal (some fibers from nerve [XII] uncurls apex and
(between hyoid) turns it down-ward
genioglossus and
hyoglossus
muscles)
Transverse Median septum of Submucosal Hypoglossal Narrows and
the tongue connective tissue on nerve [XII] elongates tongue
lateral margins of
tongue
Vertical Submucosal Connective tissue in Hypoglossal Flattens and widens
connective tissue on more ventral regions nerve [XII] tongue
dorsum of tongue of tongue
Extrinsic
Genioglossus Superior mental Body of hyoid; Hypoglossal Protrudes tongue;
tubercles entire length of nerve [XII] depresses center of
tongue tongue
Hyoglossus Greater horn and Lateral surface of Hypoglossal Depresses tongue
adjacent part of body tongue nerve [XII]
of hyoid bone
Styloglossus Styloid process Lateral surface of Hypoglossal Elevates and
(anterolateral tongue nerve [XII] retracts tongue
surface)
Palatoglossus Inferior surface of Lateral margin of Vagus nerve [X] Depresses palate;
palatine aponeurosis tongue (via pharyngeal moves
branch to palatoglossal fold
pharyngeal toward midline;
plexus) elevates back of the
tongue
The glossopharyngeal nerve [IX] leaves the skull through the jugular foramen and descends along
the posterior surface of the stylopharyngeus muscle. In addition to taste and general sensation on the
posterior one-third of the tongue, branches creep anteriorly to the terminal sulcus of tongue to carry
taste (SA) and general sensation from the vallate papillae.

822. Which is the most suitable examination for the diagnosis of crestal bone loss?

a. DPT
b. periapicals
c. true occlusals
d. horizontal bitewings
 Ans: D

823. In metal porcelain crown why is there a butt join?


a. porcelain is brittle
b. metal is brittle
c. porcelain is thin
d. metal is thin
Ans:A      

824. The use of latex gloves does has the following effect when a polyvinyl siloxane
impression is taken
a.       retards the set of the impression material
b.      enhances the set of the impression material
c.       results in porosities in the impression material
d.      latex gloves stick to the polyvinyl siloxane impression material
Ans:A, as sulphur present in latex inhibit polymerisation of pvs.so it retards

825.  Attached gingiva is the thinnest in the mouth in the following region of the
mouth?
a.       Maxillary anterior buccal region
b.      Maxillary molar buccal region
c.       Mandibular anterior buccal region
d.      Mandibular anterior lingual region
e.       Mandibular molar lingual region
Ans: E, attached gingiva is generally greatest in anteror region and narrower in posterior
with 1.8 mm in maxilla and 1.9 in mandible
826. Which fracture affects facial nerve – maxillary fractures (le fort 1,2,3) affect facial nerve causing
paraesthesia of  upper lip and cheeks

827. man on NSAIDs has blood crusted lips – blood crusted lips are the sign of erythema
multiformae triggered by a variety of drugs. nsaids is one of them

828. Basic Periodontal Examination (BPE) – if only one tooth is present in one of the
sextants score of that tooth is
a. included in the score of the adjacent sextant
b. not included anywhere
c. included in the score of the opposite arch same sextant
d. included in the scores of the remaining two sextants of the same arch
Ans: A

829. Relation of apex of 3rd molar to id canal,which radiograph will show it?
Ans: opg

830. Radiograph for mental nerve?


Ans. periapical

831. If it is missing incisor-the most common a supernumerary tooth

MISSING : congenitally missing (usually accompanied by othe missing teeth in mouth), avulsed,
extracted
PRESENT BUT UNERUPTED : ectopic position of tooth germ,dilaceration due to trauma, scar
tissue,supernumerary teeth, crowding,pathology eg cyst, odontogenic tumor etc.
But the most common cause is SUPERNUMERARY TOOTH with an incidence of 1-3% in
premaxilla 

832. characteristic for best retention of post 


a. length
b. taper
c. superficial texture
Ans : A

833. Radiograph for condylar fracture? opt-pa jaw,opg


Ans: Reverse townes

834. 1.     A patient complains of pain to his dentist. He has caries with premolars and molars.
He is visiting a dentist after 2 years. What part of the history would you ask first.
 
Social history, history of presenting complaint, medical history, dental history, family history.
 
2.     Same scenario. No treatment should be undertaken without taking which part of the
 case history
Ans; dental history

835. Which antibiotic causes a rash if given in infectious mononeucleosis?Amoxicillin


 Which antibiotic causes diarrhoea and abdominal cramps at a high dose?erthromycin
 Which antibiotic is prescribed for gram positive streptococcal infection? amoxicillin
 
Which antibiotic causes pseudomembranous colitis? Clindamycin
Which antibiotic is used in case of MRSA? according to scully its vancomycin in combination with
orther drugs .according to BNF clindamycine a lone is oneof the options

836. Patient with history of myocardial infarction who underwent angioplasty 3 days ago and now has
acute pulpitis. How would you manage it?
> > Refer to the hospital
> > Prescribe antibiotics
> > Extirpate the pulp.
> > Do rct
> > 
> > 2) the percentage of oxygen in the blood of the veins. 
> > 47mmhg
> > 3)A patient comes to you with a 10 year old amalgam filling that seems to be raised from the
occlusal surface of the tooth. What could be the reason for this? (Creep, Hygroscopic expansion
> > Creep
> > 4)Which bone is formed entirely intra-membraneously?
 Ans: perital, greater wing of sphenoid, lacerimal and Maxilla

837. RCT was carried out in a molar which had a MOD amalgam restoration. What would be your
choice of restoration.
Full gold crown, cast inlay, gold inlay, repeat the amalgam, leave it as it is.

838.  What would be the choice of restoration in a young bulimic female?


 
Metal ceramic crowns, ceramic onlay, metallic onlay.
 
839. which drug stabilises Mast cells?
cromolyn ,nedocromyl, Salbutamol
840. what percentage of people in uk have type 1 hypersensitivity?
i think its 10 % but not sure as its different in different books....but overall sensitivity is 30-40 %

841.What would be the choice of restoration for a root filled canine?


Cast post and core, fiber reinforced post and ceramic core ?
842. What is creep amalgam?
creep : slow plastic deformation occurs due to application of static or dynamic force over time.
843. mast cell stablizers
salbutamol ....cos the medical emergency document says salbutamol stabilises mast cells and
prevents release of histamine and chemical mediators.

Mast cell stabilizers are cromone medications used to prevent or control certain allergic disorders.


They block a calcium channel essential for mast celldegranulation, stabilizing the cell and thereby
preventing the release of histamine[1] and related mediators. One suspected pharmacodynamic
mechanism is the blocking of IgE-regulated calcium channels. Without intracellular calcium, the
histamine vesicles cannot fuse to the cell membrane and degranulate.
As inhalers they are used to treat asthma, as nasal sprays to treat hay fever (allergic rhinitis) and as
eye drops for allergic conjunctivitis. Finally in oral form they are used to treat the rare condition
of mastocytosis.

844. Which of the following needs explicit consent?


a. to share info with insurance company
b. to share info with other doctors in practice
c. to share info with therapist treating the patient
d. to share info with patient's GMP

845. When the enamel of a tooth is exposed to preparation containing a high fluoride
concentration;the major reaction is:
1- sodium flouride
2- calcium flouride
3- stannous fluoride
4- fluorapatite

846. PERIODENTAL LIGAMENT IS THINNER IN?


 
NEAR THE CROWN
 
INTHE MIDDLE
 
NEAR THE APEX

847. PRINSIBLE ACTION OF SALIVA IS:


 
1-LUBRICATION
 
2-ANTIBACTERIAL
 
3-BUFFERING
 
4-FOOD TASTING
 
5-FOOD DIGESTION
Excrection, saturation, dilution
848. Anemia’s most commonly inherited disorder???
Sickle cell anemia
849. Bone with intramembranous ossification only? Maxilla not given in option
Mandibular growth is principally attributed to intramembranous ossification, augmented by focal
endochondral ossification at condylar head, angle and coronoid process.

850. Bone with endochondral ossification only?


ethmoid, body of sphenoid, basiocciput and the petrous temporal bones.
851. which la can be given for pregnant ladies?
 lidocaine

852. Unit of alcohol in 1 pint of beer?


Ans: 2.3 Units
Formulae

The number of UK units of alcohol in a drink can be determined by multiplying the volume of the
drink (in millilitres) by its percentage ABV, and dividing by 1000. Thus, one pint (568 ml) of beer
at 4% ABV contains:

The formula uses the quantity in millilitres divided by 1000; this has the result of there being
exactly one unit per percentage point per litre of any alcoholic beverage.

You can also think of 4%  as being   Hence .04 X 568 mL gives the amount of alcohol in mL,
which you then divide by 10 to show the number of units.
When the volume of an alcoholic drinks is shown in centilitres, determining the number of units in a
drink is as simple as multiplying volume by percentage (converted into a fraction of 1). Thus 75
centilitres of wine (the contents of a standard wine bottle) at 12 % ABV contain:

853. bud stage of upper 1st molar begins at ?


Ans: 8th week i.u (primary molar)
854. Which dental tissue is 45 percent mineralised and resorts slower than bone?
Options? Enamel, dentin, cementum

856. levels of IgA. Is more in saliva, serum, breast milk,tears... Can any one help me to arrange
them 
Which constitutes more Iga? 
857. Iatrogenic overproduction of ACTH causes?
Cushing’s syndrome 

858. Endogenous overproduction of ACTH causes? Options ,cushing syndrome,Cushing disease


Cushing’s syndrome 

859. which nerve is affected if the p can not gaze laterally to the left
a-right abducens  b-left abducens  c-left trochlear  d-right trochlear.
 
2-which nerve is affected if the tongue deviates to the right when protruded
 
a- right hypoglossal b- left hypoglossal   c-right glossopharyngeal  d- left glossopharyngeal
 
3- Which side of the face do fractures occur most commonly?
a- right      b- left
 
4-Developmental of parotid gland begins in utro at what month?
 4to 6 weeks
Parotid gland primordia (anlage) arises in weeks 5-6 from ectoderm, followed by submandibular
gland primordia in week 6 from endoderm, then sublingual gland primordia in weeks 7-8 from
endoderm
5- 30 y man with a lip lesion he had come bach from a holiday and his partner had the same lesion
earlier on
a- herpes simplex  b-TB c- erythema multiform

860. which cement has the greatest wear resistance?

zinc phosphate

861. which is the toxic root canal sealant?


AH 26 -resin based sealer

TOXIC ROOT CANAL SEALANTS--AH26 TOXIC---A REVIEW OF THE LITERATURE


REVEALS THAT THIS REPRESENTS THE LARGEST PUBLISHED CASE SERIES OF
ENDODONTICALY RELATED INJURIES TO THE INFERIOR ALVEOLAR
NERVE.STUDIES HAVE SHOWN THAT ALL RC SEALANTS ARE NEUROTOXIC TO
SOME DEGREE.THE MOST NEUROTOXIC APPEAR TO BE THOSE CONTAINING
PARAFORMALDEHYDE OR ONE OF ITS ANALOGUE INCLIDING SARGENTI PASTE(n2)
OR ENDOMETHASONE(SEPTODENT-AVAILABLE IN CANADA N EUROPE)OTHER
SEALANTS CONTAIN ANALOGS OF FORMALDEHYDE PARTICULARLY BEFORE
THEY HAVE SET(EXAMPLE-AH26-DENTSPLY MAILEFER,TULSA,OKLA)

862. Man with candidiasis, generalised enlarged lymph nodes is seen in?
HIV infection
863. Which drug causes fixed ulcer
Nicodril

864. Atrial pressure of oxygen and carbon di oxide


Partial pressure of Oxygen is 75 - 100mm of Hg

    partial pressure of  CO2 is 40 mm of Hg (vander shrman)


Oxygen tension
Arterial blood oxygen tension (normal)
PaO2 — Partial pressure of oxygen at sea level (765 mmHg) in arterial blood is between 75 mmHg and 100
mmHg

Venous blood oxygen tension (normal)


PvO2 — Oxygen tension in venous blood at sea level is between 30 mmHg and 40 mmHg

Carbon dioxide tension


Carbon dioxide is a by-product of food metabolism and in high amounts has toxic effects
including: dyspnea, acidosis and altered consciousness[8].

Arterial blood carbon dioxide tension


PaCO2 — Partial pressure of carbon dioxide at sea level (765 mmHg) in arterial blood is between 35 mmHg
and 45 mmHg

Venous blood carbon dioxide tension


PvCO2 — Partial pressure of carbon dioxide at sea level in venous blood is between 40 mmHg and 50 mmHg

865. Which immunoglobulin is present after a bacterial and viral infections

IgG is present after bacterial n viral infections


IgM in early infections

866. Trauma to tooth with closed apex,


in which one tooth is more likely to maintain vitality - Concussion

And which one is more likely to loose vitality? - Avulsion 

Concussion

Intrusive luxation

Luxation

Avulsion 
Subluxation

867. which of following conditions would warrant hospital admission for dental surgery?
a.haemophillia
b. h/o pertussis in childhood
c.Hb-12gms
d. urine analysis showing acidic ph
e.all of above

2.when a CD wearer says s n sh tip of tongue touches 


a. hard palate
b.soft palate
c.max.canines
d.max.premolars
e.max.molars.

3.why should the lingual embrassure b/w upper3 n upper 4 be enlarged during mouth preparation
for maxillary partial denture??
a.to prevent denture slip mesially
b.to prevent denture slip distally
c.to provide adequate retention
d.to provide adequate space for reciprocating arm
e.none

868. 1)cells present in acute periodontitis - neutrophils.

2)cells present in chronic periodontitis - lymphocytes.

3) What all can be seen during the process of healing


A clot forms and inflammatory cells debride injured tissue during the inflammatory phase.
Epithelialization, fibroplasia, and angiogenesis occur during the proliferative phase. Meanwhile,
granulation tissue forms and the wound begins to contract. Finally, during the maturation phase,
collagen forms tight cross-links to other collagen and with protein molecules, increasing the tensile
strength of the scar. 

4) Malignant melanoma prevalance in oral cavity


Over 70 per cent of cases involve the posterior maxillary alveolar ridge and hard palate

5) which is the most benign lymphoma


 Burkitt's lymphoma

869. PEPPER POT SKULL APPEARANCE CAN BE SEEN IN WHICH


MEDICAL CONDITION?
In primary hyperparathyroidism, extensive resorption bone in the skull in combination with cystic
areas of osteopenia are termed pepper pot skull.

870. What's the maximum time the GDC leaves the dentist physically impaired away from his profession ?
36 months (decided by gdc health committee) reference: GDC website

871. Which one of the following drugs in hospital cannot be used in pt with anaphylactic shock?

1-oxygen 
2-Hartman's solution 
3-adrenaline
4-predenisolone
5-chlorpheniramine maleate
Answer : hartmann's solution because its side effects include edema and breathing difficulties.
 oxygen and adrenaline are the first line treatment and prednisolone and chlorpheniramine are
among the second line treatment for anaphylaaxis

872. Which periodontal ligament is thinnest and thickest along the tooth surfaces?
Periodontal ligament is hour glass shaped, meaning is thinner in the middle third and wider in apical
and coronal thirds.

873. Fluoridated Toothpaste used: 


 
Risks Adult >16 years 6 -16 years 3-6 years 0-3 years

High 5000 ppm 2800ppm 1500ppm 1000 ppm


Medium 2800 ppm 1500 ppm 1500 ppm 500 ppm
Low 1500 ppm 1500 ppm 1000-1500 ppm 500 ppm
Toxicity
Safely tolerated dose (STD) Dose below which symptoms of toxicity are unlikely = 1 mg/kg body
weight
Potentially lethal dose (PLD) Lowest dose associated with a fatality. Patient should be hospitalized
= 5 mg/kg body weight
Certainly lethal dose (CLD) Survival unlikely = 32 - 64 mg/kg body weight
Fluoride concentration in various products
Standard adult fluoride toothpaste 1000 ppm F (parts per million fluoride) = 1 mg F/ml
Daily fluoride mouthrinse 0.05% NaF = 0.023% F=0.23mg F/ml
APF gel 1.23% F = 12.3 mg/ml
Fluoride varnish 5% NaF = 2.26% F = 22.6 mg/ml
To reach the 5 mg F/kg threshold (requiring hospitalization) a 5 yr-old (about 19 kg) would have to
ingest 95 (1 mg F) tablets, 95 ml of toothpaste, or 7.6 ml of 1.23% of APF gel.
Antidotes: <5 mg F/kg body weight large volume of milk. >5 mg F/kg body weight refer to hospital
quickly for gastric lavage. If any delay give IV calcium gluconate and an emetic.
Water fluoridation in a concentration of 1 ppm (1 mg F per litre) gives a caries reduction of 50%.
Yet despite the proven benefits only 10% of the UK population receive fluoridated water.
Topical fluoride
Overall, caries reductions of 20 - 40% are reported
Rinsing solutions Mouthrinses are C/I in children <7 yrs. The concentration prescribed depends
upon the frequency of use: 0.2% fortnightly/weekly or 0.05% daily. Daily use is the most beneficial.
Caries reductions of the order of 16 - 50% have been reported with rinsing alone. The most widely
used solution is sodium fluoride.

Toothpastes aid tooth cleaning and polishing, but, most importantly, act as a vehicle for fluoride
delivery

 Fluoride. Most toothpastes contain sodium monofluoro-phosphate &/or sodium fluoride, in


concentrations of 1000 - 1450 ppm (i.e. 1-1.45 mg per 1 cm of paste). Caries reductions of
15% (in fluoridated areas) to 30% (in non-fluoridated areas) are reported. Low-dose
formulations for children <7 yrs containing <500 ppm are available, to reduce risk of
mottling.
 Anticalculus agents, e.g. sodium pyrophosphate, can reduce calculus formation by 50%.
 De-sensitizing agents, e.g. 10% strontium or potassium chloride, or 1.4% formaldehyde.
 Antibacterial agents, e.g. triclosan

Advised prescribing regime for fluoride tablets

Age of child < 0.3 ppm 0.3–0.7 ppm > 0.7 ppm

6 months – 3rd birthday 250 μg daily (need to Not advised Not advised
halve 500 μg tablet )

(0.25mg)
3–6th birthday 500 μg daily 250 μg daily (need to Not advised
halve 500 μg tablet )
(0.50mg)
(0.25mg)
6 and over 1 mg daily 500 μg daily Not advised

(0.50mg)

what`s the concentration of flouride in 2.26% NaF reparation.? - 22600ppm or 22.6 mg/ml

2.26% NaF actually contains 5% NaF and 2.26% Fl ion.


The formula is %NaF X .45 = %F ion
%SnF2 x 0.25 = %F ion
%F ion X 10000= ppm F
ppm=mg/L
So 5%NaF multiplied by 0.45 is 2.25% which then multiplied by 10000 will give the result
22500(which is close enough since Im sure figures have been rounded off in the original
question)

Fissure sealants
The accepted figures for composite resin-based sealant retention are > 85% after 1 yr and > 50%
after 5 yrs.
Sugar
The term, sugar, is commonly used to refer to the mono- and disaccharide members of the
carbohydrate family. Monosaccharides include glucose (dextrose or corn sugar), fructose (fruit
sugar), galactose, and mannose. Disaccharides include lactose (in milk), maltose, and sucrose (cane
or beet sugar). Polysaccharides (starch) are composed of chains of glucose molecules and are not
readily broken down by the oral flora.
In reducing cariogenicity:

 Sucrose, glucose, fructose, maltose (honey).


 Galactose, lactose.
 Complex carbohydrate (e.g. starch in rice, bread, potatoes).

874. Which following antifungal medication intercat with warfarin and enhance its effect
a-miconazol
b-fluconazol
c-amphoticinb
d-ketokonazol

875. Which test is the most reliable test to indicate the presence of active hepatitis
Answer is HbsAg
Hbs(surface)antigen - Hepatitis B surface antigen is a marker of infectivity. Its presence indicates
either acute or chronic HBV infection.
Hbc(core)antigen - It may be used in prevaccination testing to determine previous exposure to
HBV infection.
Hbe antigen - Hepatitis B “e” antigen is a marker of a high degree of HBV infectivity, and it
correlates with a high level of HBV replication. It is primarily used to help determine the clinical
management of patients with chronic HBV infection.
Antibody to HBs antigen - Antibody to hepatitis B surface antigen is a marker of immunity. Its
presence indicates an immune response to HBV infection, an immune response to vaccination, or
the presence of passively acquired antibody. (It is also known as HBsAb, but this abbreviation is
best avoided since it is often confused with abbreviations such as HBsAg.)
876. Edentulous population in UK :It was 13% in 1998(given in pink book, 5th edition)
BUT MOST RECENT STATISTICS SAYS:
Edentulous population: 6%
 Population with visible plaque: 66%
 Population with calculus : 68%
Acc to  ADULT DENTAL HEALTH SURVEY,UK,2009, published in 2011 

877. The ph when tooth starts deminarelising? – 5.5


878. etching
porcelain requires to b etched with hydrofluoric acid n glass ceramics with ammonium bifluoride---
master dent vol 2 chapter3
879. Which has the most tendencies to become malignant?

Erythroplakia

Speckled /nodular leukoplakia 

Erosive lichen planus

Oral sub mucous fibrosis


Ans:  Erythroplakia>speckled leukoplakia>leukoplakia>oral sub mucous fibrosis(10%)>lichen planus (1%).

880. what is the fusion time for anterior , posterior and lateral fontanelle (6,12,18,24
months)
ans: Ans is anterior at 18-24 mths posterior at 2mths

881. Would you plz give me example of

Paracrine gland
Endocrine gland
Autocrine gland
Exocrine gland
Ans: Exocrine gl. - sweat gl., mammary gl, sebaceous gl., salivary gl.

Endocrine gl. - thyroid (produce thyroxine), adreanl gland, ... actually most of the hormones are
produced this way

If I understood correctly, paracrine and autocrine can be relevent to the type of signalling.
According to this: 
Paracrine  - Somatostatin and histamine are paracrine agents, hence pancreas is an example for this.
Autocrine - monocytes (for cytokine interleukin-1)

882. The treatment for cracked tooth syndrom is:


1:Remove any associated restoration and investigate the extent of the fracture line.
2:Adjust the occlusion  to reduce loading of offending cusp.
3:Placement of adhesive restorations,such as GIC,composit resin and bonded amulgam may be
effective in preventive propagation of the fracture in the short term. However,they are often
ineffective,by themselves,ihe longterm.
4:Placement of a full venner crown or adhesive metal onlay to splint the remaining tooth structure is
more effective.As an emergency measure,the tooth may be modestly reduced,to allow placement of
a temporary aluminium (or similar)crown form.Orthodontic ban d or copper rings are (less
satisfactory)alternatives. 
5:Definitive restoration should be delayed untile the pulpal condition has been determined.If RCT is
required,this can be undertaken through the temporary crown. (all these applied to the vital tooth or
the pulp not involved)

883. Which of the following decrease with age in the dental pulp?
A.Number of collagen fibers
B.Number of reticulin fibers
C.The size of the pulp
D.Calcifications within the pulp
E. A & B
F. B & C
G. A & C
H. C & D
Explanation: as pulp ages there is decrease in reticulin fibres(pulp bcome more fibrous and less
cellular) size of pulp also decreases coz of continued deposition of dentin.there is increase in
number of collagen fibres and calcification within pulp called pulpstones. 
 
884. Which of the following are true for porcelain Veneering alloys?
a) High noble metal is 45% noble
b) Gold (Au), Palladium (Pd) and Platinum (Pt) are noble metals
c) Pd-Ag is a base metal alloy
d) Cobalt-Cromium (Co-Cr) is a noble metal alloy
e) Noble metals alloys are 25% noble and have no gold requirement
1)
a, c, d
2)
c, d, e
3)
a, b
4) b, e

885. Which of the following medications has NOT been associated with gingival enlargement?
A. Erythromycin.
B. Oral contraceptives.
C. Valproic acid.
D. Fluconazole.
E. Verapamil.
http://books.google.co.uk/books?
id=VTeMNWAKqUcC&pg=PA330&lpg=PA330&dq=erythromycin+and+gingival+enlargement&
source=bl&ots=mwccziX9UQ&sig=_Lr4QL34x1EwyqtbZpVyHLy6Ibg&hl=en&ei=lnK6ToGjG4
TH8gPov9nGBw&sa=X&oi=book_result&ct=result&resnum=10&sqi=2&ved=0CG4Q6AEwCQ#v
=onepage&q=erythromycin%20and%20gingival%20enlargement&f=false
Gum hypertrophy may be caused by various drugs including:
 phenytoin
 nifedipine
 cyclosporin
 oral contraceptive with high oestrogen concentration - now uncommon
The drugs mainly associated with GO are:

• Phenytoin, a drug used for the management of epilepsy, 


and other anti-convulsants such as sodium valproate, 
phenobarbital, vigabatrin.

• Ciclosporin, an immuno-suppressant drug used to 


reduce organ transplant rejection;

• Calcium-channel blockers (nifedipine, verapamil, 
diltiazem, oxodipine, amlodipine), a group of anti-
hypertensive drugs.

Other drugs, such as antibiotics (erythromycin) and 


hormones, have been also associated with this side effect.

886. X-ray with defect x-ray placed on the wrong side (there were foil impressions)
Ans: If u ever open an x ray film packet, u will see a lead foil which has its typical surface
irregularity. This foil is present on the side from where u open the xray packet ie. Opposite to the
plain white part of the xray packet. ideally, the white part of the packet is hit by the xray after
hitting the tooth. If the film is place the other way, the xrays after passing through the tooth,will hit
the film with the lead foil thereby giving the foil impression or design of the foil on the developed
film. This appearance is also called as "tyre track" appearance.

887. Which of the following techniques could lead to the following complications: hematoma,
trismus, temporary paralysis of cranial nerves III, IV, VI, diplopia, and paralysis of the eye?
1. Posterior superior Alveolar (PSA)
2. Maxillary Nerve Block (V2)
3. Gow-Gates mandibular nerve block
4. Akinosi technique
5. Inferior alveolar nerve block -
888. what medication can we prescribed to a patient in a lichenoid reaction to an amalgum
restoration who has a sore patch. - benzydamine
889.
1, Which cement is more susceptible to fail if is contaminated with saliva? – gic
 
2. Common reason for abscence of upper central incisor ?
 
3).According to…2000, there is color code separation.To which color correspond the clinical areas and
decontamination ?
a.Blue
b.red
c.orange
d.yellow

4)A patient is in status epilepticus,which medications will you give him?


a. Adrenaline
b.Diazepam
c.Midazolam (midazolam 10mg single dose )

890. Which of the following are disadvantages of using systemic antibiotics?


a. Development of superinfections with opportunistic bacteria.
b. Potential drug interactions.
c. Increased cost of periodontal therapy.
d. Reliance on patient taking antibiotics as they are actually prescribed.
e. All the above.

891. which one of the following radiograghic  feachures would NOT suggest that the patient would
be at high reisk of damage to thier inferior dental nurve during the removal of mand 3th molar
 
1- loss of tramline of IDcanal
2-deviation of the tramlines of the IDcanal
3-narrowing of the tramline of the ID canal
4-radiolucint band across the tooth
5- Widening of the tramline of ID canal.

892. 1).How many colonies should contain a dental water unit?


a.>20 colonies/ml
b.>200 colonies/ml
c.>2000 colonies/ml
d.>20000 colonies/ml
 Less than <200 clonies/ml ( European standar)

2)A mum is concern by the oral hygien of her child who is 3 yrs old.They live in area with
appropriate fluoridation,which advice will you give her?
a. Do not swallow the tooth paste
b. Spit the tooth paste and do not rince
 
 

3)What is the least important factor for a post?


a. The length
b. The diameter
 

4)you want to do a review on water fluoridation,which method will be the most appropriate
a.Cohort study
b.randomized controlled study
c.systemic reviews 
d.cross-sectonial surveys

893. please , match the following files with their use 


 
 
1. H files                                  a. engage and remove soft tissue from the canal
2. Broaches                              b. useful for penetrating and enlarging root canals
3. K files                                   c. screwing-in forces due to sharp edges
Ans:
1c
2a
3b
894. diamond bur removes tooth hard tissue by grinding while carbide do it by cutting for that
reason the latter is used in crown and bridges preparation

895. Ore question child with red & gingival enlargement & fever & lymphadenopathy 
Which virus?

Herpes 1
Herpes 2
HIV 
Cytomegalovirus 
Coxakivirus 
896. Ore question about antibiotic prophylaxis for patient allergic to penicillin and cants swallow 

Ans: Azithromycin if unable to swallow capsule( churchill page 72) or i.v clindamycin 

897. 3)Heart rate in a healthy young male individual during long period of exercise - 90-100
-
4)Male suffered trauma and lost almost 1 lt of blood. What is the reference blood volume per
kilogram: 70ml, 120ml, 150ml, 220m – 70

898. Ore question about white line at the level of teeth ,what is the cause?

Lichen planus

Lichenoid reaction

Traumatic keratosis 

899. howmany ml of lignocaine 2% 1/80000 can injected to 125 kilogram person? 24 ,26 ,27 ,30
 
ince the maximum dose allowed is 4.4mg/kg of lidocain 1:80 000

1. 7 x 125= 875 mg ( but remember you must never exceed the maximum dose that is 500 mg, no
matter what the weight of the patient) so you will use 500 mg in the second equation
if the patient weight was 70 then its 7mg x 70kg  =  490 mg since its below 500 maximum dose you
will us it in the second equation.

In  a 20% lidocain means there is 1ml= 20 mg that`s because:


20% lidocain means 2gm per 100 ml of solution that`s equal to
                                2000 mg per 100 ml  ( remove the zeros)
                                20 mg per 1 ml

2. then do a cross multiplication if:


1ml= 20mg then
x ml = 500 mg

that equals to= 25 


from the available options 24 is the closest you can`t give 26 ml or u will be exceeding the
maximum dose.

so there are some points u need to memorize

2% plain lidocaine maximum dose is 4.4mg/kg     maximum is 300 mg


2%  lidocaine with 1:80 000 epinephrine 7mg/kg     maximum is 500 mg
3% prilocaine  6 mg/kg maximum is 400 mg
4% prilocaine  6   mg/kg maximum is 400 mg
4% Articaine  7 mg/kg maximum is 440 mg

900. Sudden unilateral onset of facial swelling and pain over parotid area, which is not yet fluctuant

Options
a) Bacterial saliadenitis
b) Salivary duct infection
c) Saliorrhea
d) Viral sialadinitis 
I think answer should be salivary duct infection which is unilateral.
others like viral sialadinitis is most commonly caused by mumps which is bilateral and similarly
bacterial one is caused mostly by s.aureus and is bilateral as well.
Saliorrhea which is hypersalivation is associated with oral lesions and heavy metal poisioning but
never read it to be unilateral.

901. the calculation is done in this way : 


daily mouth wash contain  neutral sodium fluoride which is in 4.5%  so  for 0.5%   (it will be like
4.5 /1000 x 0.05 = 0.000225)  - 225ppm 

on the other hand if you want to know the ppm for 20% mouthwash then you will do  ( 4.5/1000 x
0.20) =  900ppm

902.
1- what part of nervous system is responsible for fight or flight?
Sympathetic
2- What part of nervous system increase salivary secretion?
Parasympathetic

3- What part of nervous system increase heart rate?


Sympathetic

903. 1- What blood vessels have the most amount of smooth muscles?
 Arteries
2- what blood vessels are the capacitance and hold most of blood volume? 
Veins serve to return blood from organs to the heart. Veins are also called "capacitance vessels"
because most of the blood volume (60%) is contained within veins.

904. In which condition is intra articular bleeding is minimal


.Vonwillebrand disease 

905. osmf survival rate in 5 yrs & 10 yrs


osmf 10-15 yrs survival rate: 10%
5 yr : 8% 

905.
UK Guideline Daily Amounts

Calories Sugar Fat sarurates


Women 2000 90g 75g 20
Men 2500 120g 95g 30
Children (5-10) 1800 85g 70g 20

Salt Intake
Women 6
Men 6
Children (5-10) 4

906. fluorides
Fluoride conversion chart
APF (10)(%)(1000) ppm
10,00
1.0%
0
12,30
1.23%
0
NaF (4.5)(%)(1000) ppm
0.05% 225
0.20% 900
0.44% 1,980
1.0% 4,500
1.1% 4,950
2.0% 9,000
22,50
5.0%
0
SnF2 (2.4)(%)
ppm
(1000)
0.40% 960
0.63% 1,512

907. what does increase with age?

Systole - In old age arteries get stiffer and systole increases.so the bp increases

Diastole

Stroke volume 
908 ELISA test used commonly for the diagnosis of which disease
Because the ELISA can be performed to evaluate either the presence of antigen or the presence of
antibody in a sample, it is a useful tool for determining serum antibody concentrations (such as with
the HIV test[8] or West Nile Virus). It has also found applications in the food industry in detecting
potential food allergens such as milk, peanuts, walnuts, almonds, and eggs.[9] ELISA can also be
used in toxicology as a rapid presumptive screen for certain classes of drugs.

ELISA (Enzyme-Linked Immunosorbent Assay) plate.


The ELISA was the first screening test widely used for HIV because of its high sensitivity. In an
ELISA, a person's serum is diluted 400-fold and applied to a plate to which HIV antigens are
attached. If antibodies to HIV are present in the serum, they may bind to these HIV antigens. The
plate is then washed to remove all other components of the serum. A specially prepared "secondary
antibody" — an antibody that binds to other antibodies — is then applied to the plate, followed by
another wash. This secondary antibody is chemically linked in advance to an enzyme.
Thus, the plate will contain enzyme in proportion to the amount of secondary antibody bound to the
plate. A substrate for the enzyme is applied, and catalysis by the enzyme leads to a change in color
or fluorescence. ELISA results are reported as a number; the most controversial aspect of this test is
determining the "cut-off" point between a positive and a negative result.
A cut-off point may be determined by comparing it with a known standard. If an ELISA test is used
for drug screening at workplace, a cut-off concentration, 50 ng/mL, for example, is established, and
a sample that contains the standard concentration of analyte will be prepared. Unknowns that
generate a signal that is stronger than the known sample are "positive." Those that generate weaker
signal are "negative."
Doctor Dennis E Bidwell and Alister Voller created the ELISA test to detect various kind of
diseases, such as Malaria, Chagas' disease, and Johne disease. [10] ELISA tests also are used as in in
vitro diagnostics in medical laboratories. The other uses of ELISA include:

 detection of mycobacterial antibodies in tuberculosis.


 detection of rotavirus in feces.
 detection of hepatitis B markers in the serum.
 detection of enterotoxin of E. coli in feces.

909. What’s more likely to cause sickle cell crisis?


http://en.wikipedia.org/wiki/Sickle-cell_disease

2 Cement for temporary crown: GIC, zinc phosphate, zinc policarboxylate, resin, ZOE

 3 What are lymph node seperators and it's clinical significant?

 4 what combines irreversible to Hb (haemoglobin)?

910. What’s the percentage of tooth erosion in 6 years-old and 15 years-old in uk?
6yrs-27%.  15yrs-54% erosion
911. Percentage of people indicated to hospital by infection?
5%
10%
15%
20%

912. What’s the percentage of >25 years with periodontitis in UK? - 64% 
913. 1) Patient had trauma – Nasal – midface area and has watery discharge and blood discharge
from nose
a) Orbital fracture
b) Zygomatic fracture
c) Lefort-1
d) Lefort-11

2) Patient with only mid-root fracture of front tooth, what is appropriate splinting time?
a) 1 week
b) 2 weeks
c) 4 weeks
d) 3 months

3) Patient had trauma with minimal mobility of front teeth, no symptoms or pain or anything, what
is the best approach
a) Splinting one week
b) Splinting two weeks
c) Splinting 4 weeks
d) Splinting 6-8 weeks
e) Soft diet and review

914. A 90-year old gentleman presented to clinic who is edentulous and has dentures upper and
lower full 15-years old. Denture bit uncomfortable, tooth structures little bit worn out, freeway
space 2-4mm, polished surfaces satisfactory and occlusal wear minimal. How would you proceed?
Copy dentures
Hard reline
Soft reline
Construction of new dentures

915. Treatment of basilar artery aneurism ?


The basilar artery is the most important artery in the back of the brain. The basilar artery can
develop a bulge that pushes outward, putting stress on the artery wall and causing it to burst. When
basilar artery aneurysms burst, they account for an estimated 27 percent of ischemic strokes,
according to neurologists affiliated with St. Louis University.

Symptoms
 Most people with basilar artery aneurysm rupture experience mini strokes (transient ischemic
attacks) in the days and weeks before the aneurysm. The most common symptoms of basilar artery
aneurysm include weakness on one side of the body or face; problems with speech and articulation;
dizziness, nausea and vomiting; headaches and visual problems; loss of consciousness.
Risk Factors
 People who develop basilar artery aneurysms often have the following risk factors: high blood
pressure (hypertension), diabetes, heart disease, vascular disease, cigarette smoking and high
cholesterol levels.
Diagnosis
 A person may be diagnosed with basilar artery aneurysm after undergoing blood tests to check for
blood clotting problems and heart disease. Doctors also use imaging tests, including computed
tomography (CT) scanning and magnetic resonance imaging and angiography to check a patient's
brain for bleeding, tumors and aneurysm and rule out other conditions that may be causing the
patient's symptoms. In addition, doctors may use transcranial Doppler, echocardiography, and
electrocardiography to make the diagnosis of basilar artery aneurysm.
Medications
 If a basilar artery aneurysm is found before it ruptures, doctors may prescribe medications to lower
blood pressure and relax blood vessels and prevent the aneurysm from breaking through the artery
wall.
Basilar artery aneurysm is usually treated with intravenous (in the vein) or intra-arterial (in the
artery) thrombolysis, which uses drugs to break down clots in the body. In the United States,
doctors administer medications called tissue plasminogen activators (tPA) to people with basilar
artery aneurysm rupture. The success of these medications is dependent on how quickly a person
sought treatment. TPA medicines may not be useful if given more than 12 hours after symptoms
appear.
Surgical Treatment
 The National Heart, Lung and Blood Institute also notes that people with basilar artery aneurysm
may need surgery to strengthen the artery wall and reduce the risk of basilar artery aneurysm
rupture.
People who have basilar artery aneurysm rupture are admitted to a stroke unit for immediate and
long-term care. Doctors' first priority in treating patients with basilar artery aneurysm involves
maintaining blood flow to the brain and other areas of the body. Patients may undergo a surgical
procedure known as clipping, which involves placing a small clip near the aneurysm. This takes the
aneurysm out of the brain's circulation and prevents excessive bleeding.
Survival and Complications
 Basilar artery aneurysm rupture causes death in more than 70 percent of the people who experience
it and occurs twice as often in men as in women, eMedicine.com reports. People who survive
basilar artery aneurysm rupture have a 20 percent chance of having another stroke later. They often
need to make changes to their diet, exercise habits and lifestyle once they are released from the
hospital and post-stroke rehabilitation program. They may need neurological and psychological
therapy to cope with physical and emotional changes they experience after aneurysm.
916. Which dental tissue is 45 percent mineralised and resorts slower than bone? Options?
Enamel,dentin ,cementum

917. A 3 YEAR OLD BABY YOU WANT TO CHECK PERMANENT GERMS.WHICH


RADIOGRAPH?
 
ANS- OPG
918. WHO IS RESPONSIBLE FOR JUSTIFICATION,QUALITY ASSURANCE AND
CONTROL X-RAY
 
ANS--IRMER PRACTITIONER OR LEGAL PERSON
919.
 Mand Incisors - >40% 2 canals but separate foramina in only 1%
        Maxillary 4 – 74% >1 canal >1 foramina
        Maxillary 5- 75% 1 canal and 1 foramina

Alcoholics in UK – Men – 72%   Women – 57%

94 % OF COMBINED POPULATION OF ENGLAND, WALES AND NORTHERN IRELAND


WERE DENTATE

EDENTATE IS 6% England 12% Scotland

17 % HAS HEALTHY PERIODONTAL GUMS.

10% HAS EXCELLENT ORAL HEALTH

31%HAS TOOTH DECAY.

45% OF ADULTS HAS PERIODONTAL POCKETING EXCEEDING 4MM.

84%OF DENTATE ADULTS HAS ATLEAST ONE FILLED TOOTH.

ADULTS AGED UNDER 45 WERE LESS LIKELY TO HAVE ANY FILLINGS.

37% OF DENTATE ADULTS HAD ARTIFICIAL CROWNS.

66%OF DENTATE ADULTS HAD VISIBLE PLAQUE.

CANCER survival in 65 year old:  Stage 1->85%    Stage 4-> 10%


                               stage1 males: 90% survival 1st year     stage 4 :   45%
                                                          `65% survival 5 year                       16%
                                                         <55% survival 10 years                   12%
Submucosal fibrosis (due to betel quid chewing) 10% chances of malignancy in 10 – 15 years

Tooth Wear (Attrition) 16-24yr old – 36%       65 yr and over – 89%

Percentage of population having type 1 hypersensitivity – 20 – 30%

Oral Cancer in UK – 2%

Squamous Cell Carcinoma – 90% of total oral cancers

0.2 – 8% in Europe and USA     Men- 58%    

CHILDREN DENTAL HEALTH SURVEY

Caries FREE 5 YEAR OLD IN ENGLAND AND WALES IS 55%

33% active caries – 2006/2007     

38% OF 12 YEAR OLD IN UK AND WALES HAD CARIES EXPERIENCE IN PERMANENT


DENTISTION.
54%OF 9 YEAR OLD NEED ORTHODONTIC EXPERIENCE.

33% boys and and 19% girls have experienced trauma at 12 years age

 5 year old trauma 30-40% at toddler stage

More than 50% <4years trauma to primary tooth affects underlying developing successor

Tooth erosion: 6 year old 52% 15 year old 27%

 Oral Cancers in uk: 2% which are 2000. Half die i.e. 1000

Percentage of fluoridated water in uk: 10%

Percentage of people with allergy in uk 10%

% of bacterial load decrease by cleaning – upto 80%

Gingivitis : 17-24yr olds -  77%  25+ - 87%

Periodontitis: 17-24 yr olds – 3%  25+ - 64%

Percentage of plaque – 72%


Percentage of calculus – 73%

%age of fissure sealants that fail and get carious     >85% - after one year             >50% - after 5
years

920. As for the hereditary enamel defects: 


AMELOGENSIS IMPERFECTA, it’s either

1. Hypoplasia: according to pink book 9 u will find it in the peads section)  its Autosomanal
dominant.
   However in Cowson the book illustrates that it has different patterns of inheritance ( autosomal
dominant, recessive and X linked, X linked dominant  types.

2. HYPOCALCIFICATION:  Autosomal dominant OR recessive in ( pink and cowson)

3. HYPOMATURATION:  Autosomal dominant

921. You get 39 weeks paid, first 6 weeks you get 90% of you salary, further 33 weeks you get
£128.78 or 90% of your weekly earning, whichever is lower.
 
You are also entitled to holidays while on maternity leave at full pay. They cannot deduct bank
holidays from your holiday entitlement while you're on maternity leave. So you get 28 days paid
holiday at the end of maternity leave.
 
For more information check the goverment website:
 
http://www.direct.gov.uk/en/MoneyTaxAndBenefits/BenefitsTaxCreditsAndOtherSupport/
Expectingorbringingupchildren/DG_10018741
 
922. Techniques for behaviour management
Tell, show, do Self-explanatory, but use language the child will understand.
Desensitization Used for child with pre-existing fears or phobias. Involves helping patient to relax
in dental environment, then constructing a hierarchy of fearful stimuli for that patient. These are
introduced to the child gradually, with progression on to the next stimulus only when the child is
able to cope with previous situation.
Modelling Useful for children with little previous dental experience who are apprehensive.
Encourage child to watch other children of similar age or siblings receiving dental Rx happily.
Behaviour shaping The aim of this is to guide and modify the child's responses, selectively re-
inforcing appropriate behaviour, whilst discouraging/ignoring inappropriate behaviour.
Reinforcement This is the strengthening of patterns of behaviour, usually by rewarding good
behaviour with approval and praise. If a child protests and is uncooperative during Rx, do not
immediately abandon session and return them to the consolation of their parent, as this could
inadvertently reinforce the undesirable behaviour. It is better to try and ensure that some phase of
the Rx is completed, e.g. placing a dressing.
923. Severe early childhood caries
Aetiology Frequent ingestion of sugar &/or reduced salivary flow.
Nursing bottle or bottle mouth caries Associated with frequent consumption of a sugar-
containing drink, especially from a feeding-bottle. Also attributed to prolonged on-demand breast-
feeding, especially at night, due to the lactose in breast milk.1 Characteristically, starts with the
maxillary 1° incisors, but in more severe cases the first primary molars are also involved. The
mandibular incisors are relatively protected by the tongue and saliva.
Rampant caries A term often used to describe extensive, rapidly progressing caries affecting many
teeth in the primary &/or permanent dentition.
Severe early childhood caries may also be caused by the prolonged and frequent intake of sugar-
based medications; however, both pharmaceutical companies and doctors are more aware of the
problem and the number of alternative sugar-free preparations is increasing. See p. 126 for list.
Management

 Removal of aetiological factors (education, artificial saliva).


 Fluoride rinses for older age groups (daily 0.05%).
 1°dentition may need to extract teeth of poor prognosis and concentrate on prevention for
permanent dentition.
 2°dentition need assessment of long-term prognosis for teeth. Final Rx plan should be drawn
up in consultation with orthodontist.

Radiation caries Radiation for head and neck cancer may result in fibrosis of salivary glands and
salivary flow. Patients often resort to sucking sweets to alleviate their dry mouth, which exacerbates
the problem.
924. Thalasemia is an autosomal dominant disorder (scully 131-2)
925.
1.    A 50-year-old man with a prosthetic heart valve requiring a tooth extraction. There is no other
relevant medical history.     No antibiotic                                                   
2.    The same patient who requires a further dental procedure three weeks after the extraction. No
antibiotic                                                                                                                                               
3.    A 55-year-old man who is allergic to penicillin and who is unable to swallow
capsules.       F.                                                                                       
                             
4.    A 5-year-old child attends in the dental emergency clinic at the weekend, with swelling of the
right side of the face resulting from an abscessed tooth. There is no significant medical
history.       A.              
5.    A 23-year-old woman presents complaining of ‘sore’, red, bleeding gums and
halitosis.     metronidazole                                                                                    
6.    A pyrexic 30-year-old man presents with an exacerbation of an apical infection two days after
initiation of root canal therapy. Amox 500mg  
Options list:
 
A.   Amoxicillin 250mg qds. for 5 days
B.   Amoxicillin 500mg bd. for 7 days
C.   Amoxicillin 1g intravenously
D.   Amoxicillin 3g single oral dose
E.    Azithromycin 500mg single oral dose
F.    Clindamycin 600mg single oral dose
G.   Erythromycin 250mg qds for 5 days
H.   Metronidazole 200mg tds for 7 days
I.      Metronidazole 200mg qds for 7 days
J.    Penicillin V 125mg qds. for 5 days
K.   Penicillin V 250mg qds. for 5 days
L.    Tetracydine 250mg qds. for 5 days
 

926.  muscles of snoring


2) muscled of swallowing
Eating and swallowing are complex neuromuscular activities consisting essentially of three phases,
an oral, pharyngeal and esophageal phase. Each phase is controlled by a different neurological
mechanism. The oral phase, which is entirely voluntary, is mainly controlled by the medial temporal
lobes and limbic system of the cerebral cortex with contributions from the motor cortex and other
cortical areas. The pharyngeal swallow is started by the oral phase and subsequently is co-ordinated
by the swallowing center in the medulla oblongata and pons. The reflex is initiated by touch
receptors in the pharynx as a bolus of food is pushed to the back of the mouth by the tongue.
Swallowing is a complex mechanism using both skeletal muscle (tongue) and smooth muscles of
the pharynx and esophagus. The autonomic nervous system (ANS) coordinates this process in the
pharyngeal and esophageal phasesllowing

927. What's the origin of primary and secondary palate ?


primary palate-frontonasal and medial nasal processes  
secondary palate-fronto nasal and maxillary process

928. when a pt suffers LMN lesion on diagnosis: the tongue deviates to the unaffected side, while in
the hypoglossal lesion it deviated towards affected side.
To test the function of the nerve, a person is asked to poke out his/her tongue. If there is a loss of
function on one side (unilateral paralysis), the tongue will point toward the affected side, due to
unopposed action of the genioglossus muscle (which pulls the tongue forward) on the side of the
tongue that is usually innervated. This is the result of a lower motor neuronlesion (the damaged
neuron directly innervates the skeletal muscle), and can lead to fasciculations and atrophy of the
tongue.[3]
The strength of the tongue can be tested by getting the person to poke the inside of his/her cheek,
and feeling how strongly he/she can push a finger pushed against the cheek - a more elegant way of
testing than directly touching the tongue.
The tongue can also be looked at for signs of lower motor neuron disease, such
as fasciculation and atrophy.
Paralysis/paresis of one side of the tongue results in ipsilateral curvature of the tongue (apex toward
the impaired side of the mouth); i.e., the tongue will move toward the affected side.
Cranial Nerve XII is innervated by the contralateral cortex, so a purely upper motor neuron (cortex)
lesion will cause the tongue to deviate away from the side of the cortical lesion. Additionally, the
fasciculations and atrophy seen in lower motor neuron lesions are not present.[3]
Weakness of the tongue is displayed as a slurring of speech. The tongue may feel "thick", "heavy",
or "clumsy." Lingual sounds (i.e., l's, t's, d's, n's, r's, etc.) are slurred and this is obvious in
conversation

929. Fusion time for anterior fontanelle?


6 months
12 months
24 months 
Ans. The range is 9- 18 months. So best answer would be (average)12 months 

930. Most appropriate situation to use hand gel to clean hands?

1 After eating and drinking 

2 after seeing patient 

3 before seeing patient

931. Which ion is most important for nerve endings?

HCO3

Mg+2

CO3
The ion necessary for action potential as asked in first question is HCO3. it maintains intercellular nerve
potential negative with respect to extracellular.

in your second question, the Na+ ion moves inward and causes the action potential.
 It’s Na and K also in addition to these at terminal end they have voltage gated Ca channels. 
932. 1- which cells are present in nerve ending? Neurons with lots of axons

2- Which cells are important for nerve ending?

933. A boy 14 years old came to ur clinic with avulsed tooth not accompanied by parents, most
appropriate in this scenario ?
1- carry out treatment 

2- ask the boy to place tooth in storage media

3- contact parents

934. Which cells are defective in diabetes mellitus?

Neutrophil 

Monocytes

Lymphocyte 

Acidophils 

935. Parotid gland nerve supply

Although the facial nerve passes through the parotid gland it does not receive innervation from it.
Instead, the parotid gland is innervated by parasympathetic fibers (of inf. salivatory nucleus) of
the glossopharyngeal nerve arising from the otic ganglion, & sympathetic fibers originating from
the middle meningeal plexus. 
Vascularization
Branches of the external carotid artery traverse the glandular tissue and supply the parotid gland
with oxygenated blood. The main branch to supply the gland is the transverse facial artery, whereas
numerous local veins drain the organ. These veins drain into tributaries of external and internal
jugular veins.
The maxillary vein and superficial temporal vein meet to form the retromandibular vein within the
parotid gland, but are not responsible for draining it.
Lymphatics comprise pre-auricular lymph nodes.

936. The Index of Orthodontic Treatment Need


Grade 1 (None)
1 Extremely minor malocclusions including displacements less than 1 mm.
Grade 2 (Little)

 2a Increased overjet 3.6-6 mm with competent lips.


 2b Reverse overjet 0.1-1 mm.
 2c Anterior or posterior crossbite with up to 1 mm discrepancy between retruded contact
position and intercuspal position.
 2d Displacement of teeth 1.1-2 mm.
 2e Anterior or posterior openbite 1.1-2 mm.
 2f Increased overbite 3.5 mm or more, without gingival contact.
 2g Pre-normal or post-normal occlusions with no other anomalies. Includes up to half a unit
discrepancy.

Grade 3 (Moderate)

 3a Increased overjet 3.6-6 mm with incompetent lips.


 3b Reverse overjet 1.1-3.5 mm.
 3c Anterior or posterior crossbites with 1.1-2 mm discrepancy.
 3d Displacement of teeth 2.1-4 mm.
 3e Lateral or anterior openbite 2.1-4 mm.
 3f Increased and complete overbite without gingival trauma.

Grade 4 (Great)

 4a Increased overjet 6.1-9 mm.


 4b Reversed overjet greater than 3.5 mm with no masticatory or speech difficulties.
 4c Anterior or posterior crossbites with greater than 2 mm discrepancy between retruded
contact position and intercuspal position.
 4d Severe displacement of teeth, greater than 4 mm.
 4e Extreme lateral or anterior openbites, greater than 4 mm.
 4f Increased and complete overbite with gingival or palatal trauma.
 4h Less extensive hypodontia requiring pre-restorative orthodontic space closure to obviate
the need for a prosthesis.
 4l Posterior lingual crossbite with no functional occlusal contact in one or both buccal
segments.
 4m Reverse overjet 1.1-3.5 mm with recorded masticatory and speech difficulties.
 4t Partially erupted teeth, tipped and impacted against adjacent teeth.
 4x Supplemental teeth.

Grade 5 (Very great)

 5a Increased overjet greater than 9 mm.


 5h Extensive hypodontia with restorative implications (more than one tooth missing in any
quadrant) requiring pre-restorative orthodontics.
 5i Impeded eruption of teeth (with the exception of third molars) due to crowding,
displacement, the presence of supernumerary teeth, retained deciduous teeth, and any
pathological cause.
 5m Reverse overjet greater than 3.5 mm with reported masticatory and speech difficulties.
 5p Defects of cleft lip and palate.
 5s Submerged deciduous teeth.

Cephalometrics
Most commonly used cephalometric points:
S = Sella: mid-point of sella turcica.
N = Nasion: most anterior point on fronto-nasal suture.
Or = Orbitale: most inferior anterior point on margin of orbit (take average of two
images).
Po = Porion: uppermost outermost point on bony external auditory meatus.
ANS = Anterior nasal spine.
PNS = Posterior nasal spine.
Go = Gonion: most posterior inferior point on angle of mandible.
Me = Menton: lowermost point on the mandibular symphysis.
A = A point: position of deepest concavity on anterior profile of maxilla.
B = B point: position of deepest concavity on anterior profile of mandibular symphysis.
Frankfort plane = Po-Or.
Maxillary plane = PNS-ANS.
Mandibular plane = Go-Me.
SNA = 81° (±3)
SNB = 79° (±3)
ANB = 3° (±2)
1-Max = 109° (±6)
= 93° (±6) or 120 minus MMPA
-Mand
MMPA = 27° (±4)
Facial proportion = 55% (±2)
Inter-incisal angle= 133° (±10)

937. Cleft lip and palate


Prevalence CLP varies with racial group and geographically. Occurs in 1:750 Caucasian births, but
prevalence M > F. If unilateral L > R. Family history in 40% of cases.
Isolated cleft palate occurs in 1:2000 births. F > M. Family history in 20%.

938. Removable appliance components (Ortho)


Active
Springs, elastics and screws

Fixed appliance components


Bands – used on molar teeth
Bonds( brackets) – attached to enamel with composite
Arch wires - Flexible nickel titanium (NiTi) archwires are used in the intial stages of Rx and more
rigid stainless steel wires for the planned tooth movements. Tungsten molybdenum, and cobalt
chromium alloys are also popular.
Auxillaries Elastic rings or wire ligatures are used to tie the archwire to the brackets. Forces can be
applied to the teeth by auxiliary springs or elastics

939. Force range for tooth movement


Tipping – 30 – 60g
Bodyly – 100-150g
Rotation/extrusion – 50-75g
Intrusion – 15-25g
Optimal tooth movement is 1mm in 4 week period.

940. Classification of functional appliances


Tooth borne passive – Andersen activator ( Overbite reduction)
Tooth borne active – bionator ( expansion)
Tissue borne – frankel appliance (Arch expension, managing hyper active mentalis muscle
(abnormal soft tissue activity))

941. What is th Ph of Venous blood(6.8 ,7.0 ,7.4)?


arterial blood- 7.4

Blood pH:
The bloodstream is the most critically buffered system of the entire body, far more sensitive than
any other. Arterial and venous blood must maintain a slightly alkaline pH: arterial blood pH = 7.41
and venous blood pH = 7.36. Because the normal pH of arterial blood is 7.41, a person is considered
to have acidosis when the pH of blood falls below this value and to have alkalosis when the pH rises
above 7.41.

Figure 3. Range of Arterial pH Values

ACIDOSIS ALKALOSIS
NEUTRAL
pH = 1 to pH = 7.42 to
pH = 7.41
7.40 14.0

 http://www.chemcraft.net/acidph2.html

942. Which is completely derived from Meckles cartridge 

Incus

Malleus 
Stapes

Malleus and incus from meckel’s cartilage and stapes from reichet's cartilage.

943. Commonest microorganism causing food poisioning in UK?

Campylobacter

http://www.nhs.uk/Conditions/Food-poisoning/Pages/Causes.aspx

944. In which condition are all clotting factors deficient ?

Disseminated intravascular coagulation

945. Cause of pain 2 years after RCT.


1. INTRA RADICULAR causes,
    *Necrotic material being left in the root canal
    *Contamination of an initially sterile root canal during treatment
    *Persistent infection of a root canal after treatment
    *Loss of coronal seal and reinfection of a disinfected and sealed canal system
    *Bacteria left in accessory or lateral canals
 
2. Extra radicular causes,
   *Persistent periradicular infection
   *Radicular cysts
   *Vertical root fractures

 946. Hiatus Hernia tooth surface loss

we should offer dietary advice but in addition management of dental erosion presented in a chronic
condition 
reference the guidelines on dental erosion ( royal college of surgeons of England)
The document mentioned the causes of erosion and Hiatus hernia was one of the causes.
THE TREATMENT PROPOSED:

1. children:
a. no complain: no ttt
b. sensitivity: cover the eroded surface with composite
c. affecting the appearance:  composite on the anterior teeth and stainless steel crown on posterior
teeth

2. Adults:
Asses the intercuspal distance:
If sufficient
 USE COMPOSITE TO REDUCE THE SENSITIVITY AND IMPROVE THE APPEARANCE.
The eroded labial and palatal surface can be restored with veneers or dentin bonded crowns.
If its not suffiecent ( due to a generalized erosion)
USE DAHL APPLIANCE
Periodontology
947. Microorganisms
Streptococcus mutans group Several species are recognized within this group, including S.
mutans and S. sobrinus. Aerobic. Synthesizes dextrans. Colony density rises to >50% in presence of
high dietary sucrose. Able to produce acid from most sugars. Most important organisms in the
aetiology of caries.
Streptococcus oralis group includes S. sanguis, S. mitis, and S. oralis. Account for up to 50% of
streptococci in plaque. Heavily implicated in 50% of cases of infective endocarditis.
Streptococcus salivarius group Accounts for about half the streptococci in saliva. Inconsistent
producer of dextran.
S. intermedius, S. angiosus, S. constellatus (formerly S. milleri group) Common isolates from
abscesses in the mouth and at distant sites.
Lactobacillus Secondary colonizer in caries. Very acidogenic. Often found in dentine caries.
Porphyromonas gingivalis Obligate anaerobe associated with chronic periodontitis and aggressive
periodontitis.
Prevotella intermedia Found in chronic periodontitis, localized aggressive periodontitis, (juvenile
periodontitis), necrotizing periodontal disease, and areas of severe gingival inflammation without
attachment loss.
Prevotella nigrescens New, possibly more virulent.
Fusobacterium Obligate anaerobes. Originally thought to be principal pathogens in necrotizing
periodontal disease. Remain a significant periodontal pathogen.
Borrelia vincenti (refringens) Large oral spirochaete; probably only a co-pathogen.
Actinobacillus actinomycetemcomitans Microaerophilic, capnophilic, Gram -ve rod. Particular
pathogen in juvenile periodontitis and rapidly progressive periodontitis.
Actinomyces israelii Filamentous organism; major cause of actinomycosis. A persistent rare
infection which occurs predominantly in the mouth and jaws and the female reproductive tract.
Implicated in root caries.
Candida albicans Yeast-like fungus, famous as an opportunistic oral pathogen; probably carried as
a commensal by most people.
Spirochaetes Obligate anaerobes implicated in periodontal disease; present in most adult mouths.
Borrelia, Treponema, and Leptospira belong to this family.

948. Plaque

Cocci predominate in plaque for the first 2 days, following which rods and filamentous organisms
become involved. This is associated with increase in numbers of leucocytes at the gingival margin.
Between 6 and 10 days, if no cleaning has taken place, vibrios and spirochaetes appear in plaque
and this is associated with clinical gingivitis.

949. Plaque is the principal aetiological factor in virtually all forms of periodontal disease.
Actinobacillus actinomycetemcomitans a capnophilic organism thought to be involved in the
aetiology of LAP. is also active against neutrophils.
950. Debris or Oral Hygiene Index This can be modified for personal use by using disclosing
agents.

 0 No debris or stain.
 1 Soft debris covering not more than 1/3 of the tooth surface.
 2 Soft debris covering more than 1/3 but less than 2/3.
 3 Soft debris covering over 2/3 of tooth surface.

951. Pathogenesis of gingivitis and periodontitis

Initial lesion: polymorphs

Early: lymphocytes and polymorphs

Established: lymphocytes ans plasma cells

Advanced: plasma cells predominate

952. Basic Periodontal Examination (BPE) Also known as Community Periodontal Index of
Treatment Needs (CPITN).

 0 = No disease,
 1 = Gingival bleeding but no pockets, no calculus, no overhanging restoration. Rx: OHI.
 2 = No pockets >3 mm, subgingival calculus present or subgingival retention site, e.g.
overhang. Rx: OHI, scaling, and correction of any iatrogenic problems.
 3 = Deepest pocket 4 or 5 mm. Rx: OHI, scaling, and root planning.
 4 = One or more tooth in sextant has a pocket >6 mm. Rx: scaling and root planing, &/or
flap as required.
 * = Furcation or total loss of attachment of 7 mm or more. Rx: full periodontal examination
of the sextant regardless of CPITN score.

953. Diagnosis Need to distinguish from apical abscess.


Apical abscess

 Non-vital
 TTP vertically
 May be mobile
 Loss of lamina dura on radiograph

Periodontal abscess

 Usually vital
 Pain on lateral movements
 Usually mobile
 Loss of alveolar crest on radiograph

954. Mobility Index:2


Grade 1 = Mobility <1 mm buccolingually
Grade 2 = Mobility 1-2 mm buccolingually
Grade 3 = Mobility of >2 mm buccolingually &/or vertical mobility

955. Furcation involvement

Classification
1st degree: horizontal loss of support not exceeding 1/3 tooth width. Requires scaling and root
planing, possibly with furcation plasty.
2nd degree: horizontal loss of support exceeding 1/3 but not encompassing the total width of the
furcation area. May require furcation plasty, &/or tunnel preparation, &/or root resection, &/or
extraction. GTR.
3rd degree: horizontal through-and-through destruction in the furcation area. May require tunnel
preparation, &/or root resection, &/or extraction.

Restorative
956. Assessment of retention
Greatest→→↠→→Poorest

Maxilla 6 7 4 5 3 1 2
Mandibl 6 7 5 4 3 2 1
e

Assessment of support
Greatest→→↠→→Poorest

Maxilla 6 7 3 4 5 1 2
Mandibl 6 7 3 5 4 2 1
e

957. Bridge failures


Most common reasons

 Loss of retention.
 Mechanical failure, e.g. # of casting.
 Problems with abutment teeth, e.g. secondary caries, periodontal disease, loss of vitality.

958. Root canal therapy


Average working lengths (in mm):
1 2 3 4/56 7
Maxilla 2120 2519 19 18.5
Mandible1919. 2420 19. 18.5
5 5
Remember:
Maxillary
4 74% have >1 canal with> 1 foramina.
5 75% have 1 canal with 1 foramina.
6,7 Assume these teeth have 4 canals (2 MB; 1P; 1DB) until second MB canal cannot be found.
Mandibular
1, 2 > 40% have 2 canals, but separate foramina are seen in only 1%.
4, 5 May have 2 canals, but these usually rejoin to give 1 foramina.
6, 7 Generally have 3 canals (MB; ML; D), but 1/3 have 4 canals (2 in D root).
Some endodontists are using a technique involving maintenance of apical patency, but at present
there is little research evidence to support this approach.

959. Evidence based practice.

Stregnth of evidence of research designs

Systematic reviews and Meta analysis

RCT

Chort studies

Case control studies

Cross sectional survey

Case reports

Confidence interval: provides a measure of the precsion or uncertinity of study results for making
inferences about the population of patients.

970. 67.The light emitted by the polymerization lamp has to be checked from time to
time. The meter used for this only measures light in the range of:
A. 100-199 nm
B. 200-299 nm
C. 300-399 nm
D. 400-499 nm
Halogen light unit for polymerization of visible-light-curing materials.

 spectral range 400 - 520 nm (blue light) - spectral intensity is well coordinated with spectral
absorption of all commercially available composites
 75 W halogen lamp guarantees high curing depth even under extreme conditions
 built-in light meter for to control the output light intesity
 built-in switch bottons allow accurate time exposure in the range of 20 or 40 seconds, or
manual mode with acoustic beep every 10 seconds
 standard package includes 8 mm 55 degree bend dental probe (universal), the probes can be
rotated through 360 degree enabliong precise application of blue light
 Ergonomic design of the handpiece is comfortable to handle and allows precise
manipulation

971. Patient with xerostomia, will benefit much more from


1. Scaling polishing
2. Flouride rinse
 
972.
1- who checks if there is written protocol put up in the clinic for radiation protection?
Employer(Legal person)
2- who justifies and optimises radiographs?
Practitioner
3- Whom to contact if there is 20s of extra radiation dose?

4- Radiation dose of 2 bite wings compared to natural?


Radiation dose of 2 bite wings = radiaton dose of a panoramic = 1-4 days of background radiation
http://www.radman.co.uk/resources/dental-radiography-and-x-ray.aspx

973. Which is least likely to be unilocular?

A ameloblastoma

B odontogenic keratocyst

C aneurysmal bone cyst

D myxoma 

E osteoporosis 

974. Question 1: In a class II.2 malocclusion, which bridge design would be contraindicated for a
missing lateral upper incisor?
A.Cantilever bridge
B.Maryland bridge

Question 2: Which is the best cantilever bridge design for missing maxillary canine? Abutment on
A.Both premolars
B.Lateral and central incisor
C.Lateral incisor
D.First premolar

975 . 1.How old patient most likely to have trauma that damages tooth germ of permanent incisor?

Opt-1, 2 year??
 Ans: its 2 years
The reference is master 2 chap 8 page 192  second ed

2. 30 year old man, front tooth missing, with sound abutment, what is the best treatment between
fixed fixed bridge and implant?
 Ans: implant 

976. A 9 years-old child who has sustained a fracture of a maxillary permanent central incisor in
which 2 mm of the pulp is exposed, presents for treatment 30 minutes after injury. Which of the
following should be considered?

A.Remove 1-2 mm of the pulp tissue surface, place calcium hydroxide and fill with resin

B.Remove 1-2 mm of the pulp tissue surface and cover with ledermix

C.Place calcium hydroxide directly on the exposed pulp

D.Pulpotomy using formocresol

E.Pulpectomy and immediate root filling

In a flouridated toothpaste with 0.304% sodium fluoride the amount of flouride ions is

A.400 ppm

B.1000 ppm

C.1500 ppm

D.4000 ppm

A low fluoride, sorbitol-based toothpaste designed specifically for children is available (Colgate
Junior Toothpaste) and contains 0.304% MFP (400 ppm fluoride). Data from several independent
studies indicate that, although a dose-response relationship does exist for fluoride levels in
toothpaste and caries, use of a 400 ppm fluoride-containing paste by children under 7 years of age
instead of the standard 1000 ppm fluoride paste should not increase their caries risk.
http://www.australianprescriber.com/magazine/17/2/49/51/

2.Reversible pulpitis is characterized by


A.Pain lasts longer on hot or cold stimulus than normally

B.Patient can’t localize pain

C.Will have periapical involvement in radiograph

Reversible pulpitis
Symptoms Fleeting sensitivity/pain to hot, cold or sweet with immediate onset. Pain is usually sharp
and may be difficult to locate. Quickly subsides after removal of the stimulus.
Signs Exaggerated response to pulp testing. Carious cavity/leaking restoration.
Rx Remove any caries present and place a sedative dressing (e.g. ZOE) or permanent restoration
with suitable pulp protection.

3.Irreversible pulpitis is characterized by

A.There is often a history of spontanous pain

B.Sudden throbbing pain

C.Pain can’t be localised when it reaches the periapical area

D.There is pain which lingers for a short duration after romoval of stimulus

Irreversible pulpitis
Symptoms Spontaneous pain which may last several hours, be worse at night, and is often pulsatile
in nature. Pain is elicited by hot and cold at first, but in later stages heat is more significant and cold
may actually ease symptoms. A characteristic feature is that the pain remains after the removal of
the stimulus. Localization of pain may be difficult initially, but as the infiammation spreads to the
periapical tissues the tooth will become more sensitive to pressure.
Signs Application of heat (e.g. warm GP) elicits pain. Affected tooth may give no or a reduced
response to electric pulp tester. In later stages may become TTP.

Rx Extirpation of the pulp and RCT is the treatment of choice (assuming the tooth is to be saved). If
time is short or if anaesthesia proves elusive then removal of the coronal pulp and a Ledermix
dressing can often control the symptoms until the remaining pulp can be extirpated under LA at the
next appointment.

4.2.2 Mg of NaF contains how many mg of fluoride ions? 

A. 0.5mg
B. 1.0 mg
C. 1.5mg
D. 10mg 

Oral Rinse:
Sodium fluoride oral rinse is acidulated phosphate sodium fluoride and is an oral rinse/supplement.
Each teaspoonful (5 ml) contains 1.0 mg fluoride ion (F-) from 2.2 mg sodium fluoride (NaF), in a
0.1 Molar phosphate solution at pH 4, for use as a dental caries preventive in children. Cherry, cool
mint, bubble gum, grape - sugar and saccharin free. Cinnamon - contains saccharin, but is sugar
free.

Dental Rinse:

Sodium fluoride dental rinse provides 0.2% sodium fluoride in a mint-flavored, neutral aqueous
solution containing 6% alcohol. For weekly use as caries preventive.

Brush-On Gel:

Self-topical neutral fluoride containing 1.1% sodium fluoride for use as a dental caries preventive in
children and adults. This prescription product is not a dentifrice.

Gel-Drops:

Sodium fluoride (acidulated) gel-drops contain 0.5% fluoride ion (F-) from 1.1% sodium fluoride
(NaF) in a lime-flavored aqueous solution containing 0.1 Molar Phosphate at pH 4.5. For daily self-
topical use as a dental caries preventive. This form of this drug (neutral) also contains 0.5% fluoride
ion (F-) from 1.1% NaF, but with no acid phosphate, nor artificial flavor or color, at neutral pH.

Drops/Tablets:

Each ml contains 0.5 mg fluoride ion (F-) from 1.1 mg sodium fluoride (NaF). For use as a dental
caries preventive in children. Sugar free. Saccharin-free.

Sodium fluoride lozenge-type chewable tablets for use as a dental caries preventive in children.
Sugar free. Saccharin-free. Erythrosine (FD&C red #3) Free. Each 0.25 mg F tablet (quarter-
strength) contains 0.25 mg F from 0.55 mg NaF. Each 0.5 mg F tablet (half-strength) contains 0.5
mg F from 1.1 mg NaF. Each 1.0 mg F tablet (full-strength) contains 1.0 mg F from 2.2 mg NaF.
Each SF 0.25 mg F tablet (SF for Special Formula: no artificial color or flavor) contains 0.25 mg F
from 0.55 mg NaF.

http://www.orgyn.com/resources/genrx/D002244.asp

977. sensitivity is according to definition number of true positives that are correctly identified by a
test and specificity is the number of true negatives identified by a test....so its better for a test to be
sensitive rather than just specific...sensitivity of a test is the probability that it will surely give us a
positive result in the end when we compare it to a gold standard.For eg if drinking sugary drinks is
proved to give you caries then a test in which we give children who don't brush their teeth sugary
drinks is a test with high sensitivity...meaning it has a high likelihood of giving us a positive
result....something they call true positive(the patient has caries and the test is positive)

On the other hand if you do a test in which you give children sugary drinks but the children are
brushing their teeth and have a good oral hygiene...then there is a high likelihood that the children
wont get caries...this means that this test has high specificity meaning the test is producing a true
negative result(the patient does not have caries and the test is negative) when tested on a sample
which was initially normal.

So I think that would mean sensitivity is inversely proportional to specificity.

So sensitivity is the prob that a test says that a person has the disease and when we do the
test/experiment we find out it is fact that the person has a disease.
Specificity is when the test says the person doesn't have the disease and when we do the
test/experiment it is a fact that the person doesn't have the disease.

978. Which dental hard tissue is mineralised first?

Enamel 

Dentine

Cementum

Bone

979. WHAT THE treatment for -  Denture stomatitis and resisted angular


cheilitis?
FOR DENTURE STOMATITIS:
start with:
1. fluconazole 50 mg onece daily  for 7 days
2. miconazol oromucosal gel 2% 24mg/ml
3. Amphotrecin 10mg  use 4 times a day for 10 days
4. Nystatine suspension 100,000 UNITS 

1 ml after food 4 times a day

RESISTANCE CASE OF ANGULAR CHELITIS


USE:
Miconazole 2% plus hydrocortisone 1% cream or oinments use 30gm apply twice daily to
the corner of the mouth for a maximum of 7 days
Unresponsive cases can be treated with hydrocortisone and miconazole cream or ointment.
Continue treatment until clinical resolution is achieved. A lack of clinical response might
indicate
predisposing factors such as a concurrent haematinic deficiency or diabetes. Refer such
cases to
a specialist or the patient’s general medical practitioner.
980. WHAT    ARE THE SEPARATORS OF LYMPHNODES ?
Lymphnodes seprators are investing layer of fascia,they prevent the spread of infection in
surrounding tissues.

981. What is the maximum dose of salbutamol?


If you asking about emergency application of sul for an acute episode of asthma  during a 
dental session the answer is 10 activation of the sul inhaler

 SALBUTAMOL
By mouth (but use by inhalation preferred), 4 mg (elderly and sensitive patients initially 2
mg) 3–4 times daily; max. single dose 8 mg 

By inhalation of nebulised solution, adult and child over 5 years 2.5–5 mg, repeated up to 4
times daily or more frequently in severe cases; child under 5 years 2.5 mg, repeated up to 4
times daily or more frequently in severe cases
Please refer BNF

982. In which year is GDC going to conduct revalidation?


a. 2010
b. 2011
c. 2012
d. 2013
e. 2014
http://www.gdc-uk.org/Dentalprofessionals/Revalidation/Pages/Revalidation-Q-and-As.aspx

983. Which one is not a developmental line of teeth.

Von Ebner

Neonatal line

Perikymata

answer is Perikymata.

incremental line of retzius(development line of enamel) that do not complete circle but radiate out
to the tooth surface form a groove on tooth surface  are called incremantal lines of
PICRRILL.between these 2 grooves is a convexity known as Perikymata.

Neonatal line ..its accentuated inc line of retzius.it demarcates prenatal enamel from postnatal
enamel.
von ebner is incremental(developmental )line of  dentine.

984. you are examining a patient whom u suspect as a cracked tooth. which of the following signs n
symptoms and diagnostic test help confirm ur diagnosis

a)pt has pain wen they bite on something


b)pt has pain wen they release their bite
c)tooth is tender to percussion
d)appying orthodontic band to tooth will result in reduction of the pain
e)transillumunation shows that the light travels thru the tooth
Ans: B,D

985. Which antibiotic is used to manage a super infected herpetic lip lesion.

Which analgesic cause ringing in the ear after overdose.

Antibiotic for a patient allergic to penicillin who can not swallow pills.

ANS:

1. cefuroxime(2nd generation cephalosporins)normally given in super infections.


2. asprin as it causes tinnitus
3. It depends upon choices but macrolide azithromycin is normally given. (oral azithromycin
suspension (>/=10 years: 500 mg; <5 years of age: 200 mg; >/=5 to <10 years of age: 300 mg)
given 1 h before the procedure can be used as an alternative.)

986. Which LA is given  for patient with Cardiac Arrythmias?

Ans: prilocaine

987. Periodontal probing force is 0.25N.


N stands for Newton.

988. JVP is best described as

1- pulse pressure
2- 10 mmHg less than ventricular pressure 
3- 10 mmHg more than ventricular pressure
4- 20 mmHg less than ventricular pressure
5- 20 mmHg more than ventricular pressure

989.
1- which is more stretchable? 
Veins are the most elastic..n hence most stretchable.
2- Which is carrying least of blood?

Veins
Venues
Arteries
Arterioles
Capillaries 

990.
1)best radiograph to asses bone level for Implants or xray for Implants in mandible ?
2) radiograph for  caries in 3year old? or for primary molars?
3) xray for calculi in slivary duct?

1-PA.
2-Bitewing.
3-Lower true occlusal.
 
991.
1- what is RPI in partial denture? 

2- patient has a nice partial denture with a canine(last teeth) pocketing less than 2mm and 50% bone
support, what treatement is the best:
implant
based overdenture
extraction and wait healing to make a new denture
extraction and make an immediate denture
RCT if canine treatable and overdenture
Ans;
1. Rest seat ( mesialy located)  -proximal plate- I bar clasp
2. Based overdentue
Explnation:
Its an overdenture
I choose it since the ideal indication is when the canine are the only teeth remaining in the
arch.
since you reduce the tooth length  to a dome shape, you decrease the crown and root ratio
thus reducing the stresses on the teeth.
It indicated ideally when the canines are the only remaining teeth in  the arch.
According to the conservative dental approach instead of extracting the teeth , they can be
preserved to:

1. Increase retention
2. offer proprioceptive stimulation
3. transmit the forces to the bone aid in the remodeling and decrease the rate of bone
resorption
4. increase the masticatory force

I choose it since its more conservative and the tooth only had a 2 mm pocket depth, plus
they are canines ( corner stone of the arch with long roots)
check pink book overdenture

992. AB for chronic sinusitis

Its amoxicillin 250mg for 7 days according to scottish dental prescribing pdf.
We can also prescribe Doxycycline 100mg.

993. Which drugs are c/I in lactating mothers


Antihistamines, benzoadepines, aspirin, metronidazole, tetracyclines, caramezipine

994. CVS and exercise


to understand the changes during exercise, we need to know what is required by the body in such a
situation..
Imagine yourself doing a workout and the physical changes will be apparent:
1. increased heart rate.
2. increased flushing/warmth/sweating.
3. breathlessness
the other changes in the body would be to increase the flow of blood to the areas where it is needed
the most: the skeletal muscles and to some extent, the skin. 
Now, To increase this flow, we need vasodilation but since the total amount of blood in the body is
limited, we also need vasoconstriction in areas that do not need blood at that time:
viscera/kidney/etc.. 

Peripheral resistance is a measure of the resistance to the flow of blood in any vessel. so obviously,
vasodilation would decrease PR and vasoconstriction would inc it.
when exercising, the amt of vasodilation in the working muscles is much greater than the amt of
vasoconst in the non working areas leading to a "NET DECREASE" in TOTAL peripheral
resistance. 

Now, cardiac output is the amount of blood pumped by the heart each minute, which should
increase to be able to supply the increased demands of the body during exercise. cardiac output is a
function of heart rate and stroke volume C.O=H.R * S.V , 
We know that the heart rate increases during exercise.
Stroke volume is simply the amount of blood pumped by the heart in each stroke i.e after each
systole.
To increase the stroke volume we need 
1. increased volume of blood in the heart to pump out and
2. increased effectiveness of the heart to pump out this blood i.e increased contraction. 
To increase the volume of blood in the heart, the end-diastolic volume should increase, which is a
result of an increased venous return. 
and an increase in the heart's contraction is caused by increased sympathetic stimulation to the
heart(beta 1 receptor action) and to some extent by the frank-starling mechanism. 
So, we have a NET INCREASE in cardiac output. 

All these changes will occur in moderate to extensive exercising, however exercising beyond a limit
can cause a decrease in the cardiac output because increased heart rate also decreases the amount of
time available for filling, which would decrease the stroke volume and hence ultimately the cardiac
output.. 

995. Origin of perio ligament?


it's fibroblast from dental follicle.

996. Wht is th concentration of lignocaine in topical anaesthetic?


10% spray and 4% gel for oral use
5% lidocain in topical,    EMLA is 2.5 % lidocain and 2.5% prilocain

997. 1) where more blood vessels seen in which part of pulp ? 


 1) Close to dentine
 2) Centre of pulp chamber
 3) Above furcation area

2) Cells that give rise to the permenant germ?


3) What is the function of lateral dental lamina? why is it formed?
4) Cells which connect enamel organ to the oral epithelium?
5) Best way of comparing milk teeth and permenant?

Ans:
1. IS CENTRAL PART
2. DENTAL LAMINA
3. GIVE RISE TO THE PERMANENT SUCCESSOR
4. DENTAL LAMINA THEN IT DEGENERATE TO GIVE EPTHELIAL REST OF
SEARS 
 LATERAL LAMINA DOES NOT FORM TEETH...
           
Lateral lamina can be seen in the cap and bell stages  .....when the tooth bud grows it drags
a part of dental lamina along with it....and it is just an extension of the dental lamina which
connects It to the tooth bud NO FUNCTIONAL SIGNIFICANCE....
                

998. 1-WHICH BUR WOULD YOU USE TO CUT ENAMEL DENTINAL JUNCTION

2-PICTURE OF RED LESION ON BORDER OF TONGUE IN A PT WHO HAD HEAVY


AMALGAM FILLINGS ON THAT SIDE.

WHAT IS THE DIAGNOSIS(OPT:SCC,TRAUMATIC ULCER ETC)

3-WHICH ONE IS POLISHED THE BEST?MICROFILLED,GLASS IONOMER,RESIN


MODIFIED GLASS IONOMER

4-WHAT IS THE MOST USEFUL FILLING FOR MOLAR


5-MIN.TOOTH REDUCTION IN GOLD CHAMFER
Ans: Ans:
1. ROUND
2. TRAUMATIC ULCER
3. MICRIFILLED
4. AMALGAM
5. 0.5mm

999.
1. 3 year-old child, baby teeth to be extracted.Which ragiograph you select to check permanent
germs. 

Panoramic, periapical, bimolar, occlusal, lateral skull view, etc

2. Trauma to tooth with closed apex. In which one the tooth is more likely to maintain vitality?
Concussion, subluxation, intrusive luxation, avulsion, luxation

3. What's the most effective method of diet advise for a child. 

a.tell the parents not to give sweets for the child b. dentist tells the child which are good and bad
foods c. nurse talk to the child d.send the child to a dietician etc

1000. Origin of the lower lip?


fusion of  ectomesenchyme of mandibular processes.

1001. Which odontogenic pathology would contain clear straw coloured fluid? Options
are 1. Odontomes, 2. Solitary bone cyst, 3. Ameloblastoma, 4. Dentigerous cyst,
5. Keratocyst. 
1002.  What is the annual dose limit of radiation for non classified worker?
Ans. 6mSV 
1003. Most abundant immunoglobulin in saliva during infections?
According to wikipedia its IGA, easy way to remember is most of the body excreted fluid are IGA
eg: tears, blood

1004. 1. What would you do when a patient comes with an asymptomatic root canal treated tooth
with periapical radioluscency treated by some other dentist?
2. Who is responsible when the dentist is sanctioned
3. What's the max. Time the gdc leaves the dentist physically impaired away from, his profession?
4. Best solution used for hand scrubbing
5. Urgent referrals-time
6. Surgical removal-which consent?
7. A new dental nurse wants to take radiograph~~~~~~~~~~~~~~~~~~~~~~~
8. A female nurse denied leave
9. enquring about pts oral hygiene care- open or closed questions, some questions given so you
select one
10. What happens if the dentist does not pay his annual retention fees on time?
11. Dental nurse that had needle stick injury, can she go back to work.?
Ans:
1. Since its asymptomatic, jus inform the pt, but do not attempt re rct..if in future pt has pain
then go for re rct.. 
3. 12 months 

5. 2 weeks

6. Written consent

10. Erase from register 

1005.
1-HOW OFTEN SHOULD YOU REVIEW SMOKING STATUS?3,6,12 MONTHS
2-IF YOU STIMULATE THE SALIVARY FLOW IN ONE SIDE OF THE MOUTH-WHERE
DOES THE SALIVA APPEAR?BILATERAL, IPSILATERAL,CONTRALATERAL ETC
3- WAT R THE ELECTRIC PULP TEST READINGS??
4-Epulis is a characteristic of which condition?

Ans:
1. 12months
2. Ipsilateral
3. 80 non vital (0 - 40 NORMAL ,40 - 80 PULPITIS OVER 80 DEAD PULP )
4. Pregnancy epulis

1006.
1.WHAT`S MOST COMMON DISEASE IN ASIA?

2.WHO IS RESPONSIBLE IF THE DENTIST IS SANCTIONED?

3.MOST DIFFICULT MECHANISM OF PHAGOCYTIC KILLING?

4.DENTIST SUSPECTING HE HAS HIV SHOULD HE STOP WORKING, REVIEW GP.

1007.
Whay is the method of disposal of following instrument????
1)metal matrix band - sharps
2)disposable matrix band - sharps
3)suction tube - clinical waste.
4)aspirator - clinical waste(orange bag)
5)stock tray - disinfection,wash & autoclave 
6)disposable tray- single use so discard into clinical waste. 

1008.

1- which ig will increase in periodontitis ?


2- which ig will increase in gingival inflammation?
3- eosinophils increase in which hypersensitivity ?

1009. Cellls present in rhumatoid arithritis?

 the answer is B lymphocytes. Rheumatoid factor seen in Rheumatoid arthritis are immunoglobulins. They
are synthesised by plasma cells derived from B lymphocytes. Cytokines from T cells cause B cells to
differentiate to plasma cells and release more antibodies.

1010.

1.  min persentage of leukocytes for extraction

2. Ig present in mumps

3. antibiotic for child allergic to penicillin and cant swallow

4. pre dominant fibers in pdl


Ans:

Q1. 4 x 10 (9). 
Q2. IgM.  Early (IgM) and late (IgG) phases of the immune response.
Q3. Erythromycin oral suspension  

Q4. Type1 collagen fibers (80%) and type lll (20%).

1011. Poor technique of matrix band cause


marginal overconstuction,underconstruction,open contact point

1012.

1. Which is the most benign lymphoma?


HODGKIN'S LYMPHOMA

2. Which of the following salivary gland tumours is most likely to occur bilaterally?
a) pleomorphic adenoma
b)mucoepidermoid carcinoma
c)adenoid cystic carcinoma
d)acinic cell carcinoma

3.which of these is most benign?


a)kaposis sarcoma
b)adenolymphoma
c)burkitts lymphoma

1013. which used for root filled canine


1. cast post and core
2.fiber reinforced post
3. ceramic core

1014. INR for healthy young patient(0,1,2,3,4)

1015. Best cement for temporary crown? -


ZOE

1016.

1)survival rate of tumours in stage 1,2,3,4


2)first stage in the management of rampant caries
3)oral thrush is associated with which disease
4)common reason for missing centrals incisors
5) cyst in which white paste is seen?

Ans: STAGE 1 WHICH IS T1N0M0 IS 85% (5 year survival)


STAGE 4 reduced to 15%  (5 year survival)
STAGE 2 (NOT SURE) 65%
RAMPANT CARIES - DIETARY ADVISE
ORAL THRASH - HIV
MISSING CENTRAL INCISORS - SUPERNUMERARY

1017.

1)what component in amalgum causes tooth blackening 


2)what component in amalgum causes expansion
3)main constituent of gutta percha
4)which composite gives the best polish

Ans:

1. Silver
2. Zinc (silver increses the expansion on setting due to gamma 1 phase (Ag2Hg3).Volume of
gamma 1 is greater than Ag/Hg independently.{phillips dental material book UG}

Zinc causes delayed expansion if amalgm mix is contaminated witjh moisture during manipulation)

3. Main responsible for its properties is Guttapercha is 20%, zoe 66%, wax and resin 3%, 11 metal
sulphate for radioopacity
Although gp is not the major ingredient it serves as a matrix while the ZOE serves as a plasticizer.
4. WHEN THE SIZE OF THE FILLERS DECREASE THE POLISH-ABILITY INCREASE SO
ITS HIGH IN MICRO and Nano FILLED COMPOSITES.

1018. Q best perio patient interdental cleaning is dental floss, interdental brush...etc they mentioned
both in pink book but I think dental floss is the best isn't it?

1019. Glass inomer cement bind better to enamel or dentin

Enamel (its enamel because of the reaction between hydroxyapetite crystals and polyacrelate.)

1020. Cyst in which white paste is seen?

OKC and sabecous cyst.

1021. How to dispose half full cartridges and out of date prescribed medicine ,as they are not
hazardous anymore

NO EMPTY CARTRIDGE - SHARPS 
HALF FULL - SPECIAL WASTE   
1022.
1- what's the colour of sharp box is it yellow with yellow lid or yellow with orange lid?

2- what's the colour of clinical waste bag is it yellow or orange?


Ans:

1. its yellow box with yellow lid for sharps


2.orange bag for soft clinical wastes

1023. 1. Best restoration for peg laterals...


Wld it be composite

1024. Causative microorganism in th uk?(infective endocarditis)

Viridans Streptococcus  GROUP.
S.MUTANS 

1025.

1What's more likely to cause sickle cell crises?(Exercise,...


2when nutrophils present th most?(Becterial,viruses,fungi..etc)
3what cells see in parasitic infection?(Basophils,eosinophils..etc)

1026. WHAT KIND OF BIOPSY FOR DIAGNOSE OF SCC ON TONGUE ? 

Chronic ulcer or squamous cell carcinoma - incisional biopsy of margin, not suitable for general
practice ,urgent referral

1027.  
1)lingual cavity in posterior teething Parkinson's patient ,which material is used?
2) patient is having bulimia nervosa which crown u give?
3)patient has sjogrens syndrome,has class 5 caries which restoration
Ans;
1 AND 3 GIC
1028. WHICH MUSCLE OPENS AUDITORY TUBE ? 
It is Tensor veli palitini (muscle of soft palate)

1029. According to the delivering better oral health doc the percentage of:

1. men and women drinking above the low risk levels is 26%: of them men are 38% and women are
16%
2. the hazardous drinkers 23%; men 32% and women 15%

1030. Bilateral parotid swelling may be a feature of: (MD 1 – page 219)
a. Sarcoidosis
b. Warthin's tumour
c. Primary Sjogren's syndrome
d. HIV (human immunodeficiency virus)
infection
e. Chronic lymphocytic leukaemia (CLL)
Explnation:
a. True. Sarcoidosis is a multisystem chronic granulomatous inflammatory disorder that can
involve the salivary glands and cause swelling and dry mouth. Parotitis, facial palsy, uveitis in
sarcoidosis is known as Heerfordt syndrome.
True. Up to 10% of Warthin's tumour are bilateral cases.
c. True.
d. True. Bilateral lymphoepithelial cysts of the parotid gland, Sjogren-like syndrome and malignant
lymphoma may occur.
e. True. CLL may infiltrate any organ. Bilateral parotid enlargement and dry mouth are rare
manifestations.

1031. Somatic and genetic effects of X-rays


The irradiation of cells can result in somatic effects (i.e. those occurring in the irradiated somatic
cells of an individual) or genetic effects (i.e. those occurring in the germ cells and transmitted to the
offspring of the irradiated individual) because of gonadal exposure. In properly conducted dental
radiography, genetic effects are not usually considered because the gonads should not be irradiated.
Somatic effects can be:
• deterministic: cataract formation, loss of fertility, erythema of the skin, 'radiation sickness'
• stochastic: tumour induction (also occurs in genetic effects).
All deterministic effects have threshold doses below which they do not occur. Above the threshold
dose, the effect is certain to occur. In dental radiography these thresholds should never be reached.
The risk from dental radiography is for stochastic effects.

Protection of patients
In dental radiography, protection of patients is achieved by three main means:
• justification
• dose limitation
• quality assurance
Selection of bitewing radiographs

Selection of periapical radiographs


Periapical radiographs are indicated in the following situations:
1. When dictated by localised symptoms/signs (pain, swelling and tenderness of a tooth).
2. Prior to the extraction of third molars, lone-standing upper molars or where there is reasonable
clinical suspicion that problems may arise. The fashion of routine preextraction radiographs has
arisen in the absence of any scientific evidence of benefit.
3. Prior to preparation of a tooth for a crown or bridge retainer.
4. In endodontics, where basic guidelines suggest radiograph(s) at the following stages:
a. preoperative
b. working length estimation
c. master cone position (precondensation)
d. postcondensation
e. At 1 year after treatment completion
f. At 4 years after treatment completion
5. Dental trauma.
Selection of panoramic radiographs
In terms of image quality, panoramic radiography is inferior to good intra-oral radiographs.
Consequently, for most dental diagnostic uses it is a 'second best' imaging technique. Possible
situations where it may be useful include:
• where a bony lesion or unerupted tooth is of a size or position that precludes its complete
demonstration on intra-oral radiographs
• in orthodontic assessment when clinically indicated (no 'screening')
• preoperative assessment of third molars, unless other adequate radiographs are available
• when mandibular fracture is suspected.
Routine 'screening' of all new patients is never justifiable and serves the dentist's pockets rather than
the patient's welfare.

Administration of radiation protection


The process of radiology in medicine/dentistry is divided into specific 'roles'.
Employer (legal person)
The employer (e.g. NHS Trust, Health Authority, principal in general dental practice) has legal
responsibility to ensure that regulations are followed. The employer must ensure that referrers have
written guidance (referral/selection criteria) on referral of patients for X-ray examination.

Referrer
The referrer is a registered medical/dental practitioner. The duty of the referrer is to supply adequate
clinical information to allow the practitioner to justify the examination. In general practice, the
referrer is the dentist.
Practitioner
The practitioner is an individual who is qualified to justify radiological examinations. In hospitals,
this is the radiologist or, depending on local arrangements, the radiographer. In general dental
practice, it is the dentist.
Operator
The operator is the person who carries out the radiological examination. In hospitals this is the
radiographer. In general dental practice, it is the dentist or a suitably qualified therapist, hygienist or
dental nurse.
Radiation Protection Supervisor
The Radiation Protection Supervisor is an individual who takes the role of checking that legal
requirements and 'good practice' are being followed. In general dental practice, this is usually a
dentist.
Radiation Protection Adviser
All facilities, including general dental practices, must appoint a Radiation Protection Adviser. This
is a medical physicist who provides expert support in ensuring that regulations are followed and
good practice is maintained.

1032.
1)Anesthetic that can be given to heart patients
2)la for cardiac arrhythmia patients
3)anesthesia for COPD patients
4)la for patient with congenital heart disease 

ANS;
As such i dont think there is any special LA for different cardiac conditions and most preferred is
PRILOCAINE being rapidly metabolised in liver.
Another LA which can be given is mupivacaine without adrenaline.
Adrenalinine is contraindicated in patient taking digoxin (which is taken by cardiac arrhythmia
patient)as it may precipitate dysarrthymias.

For COPD,no special LA is given as far as skully is concerened.


Though LA can be given without adrenaline as it may interact with theophlline(given to copd
patients for dyspneoa)
 1, 2 and 4 Prilocaine
3.Lignocaine
1033. Stratum intermedium  and stallete reticulum secretes Glusoeaminoglycan 
Stratum intermedium in a developing tooth is a layer of two or three cells between the inner enamel
epithelium and the newly forming cells of the stellate reticulum. It first appears during the early bell
stage of tooth development, at around the 14th week of intrauterine life. The stratum intermedium
has a notably high alkaline phosphatase activity.
Glycosaminoglycans (GAGs) or mucopolysaccharides are long
unbranched polysaccharides consisting of a repeating disaccharide unit. GAGs form an important
component of connective tissues. Some examples of GAG include chondroitin sulphate, dermatan
sulphate, keratan sulphate, heparin, heparan sulphate and hyaluronan.
So therefore, I assume answer is stratun intermedium secretes alkaline phosphatase.
1034. Most comman type of trauma in primary and permanent dentition...
Luxation injuries
intrusion is most common luxation injury

1035.
HERE ARE SOME IMPORTANT DISEASE CLINCHERS TO FACILITATE DIAGNOSIS:

PATHOGNOMONIC SIGNS
1. COPD - Barrel-Chested
2. Pneumonia - Greenish Rusty Sputum
3. Pernicious Anemia - Beefy Red Tongue (Schilling's Test)
4. Kawasaki Disease. - Strawberry Tongue
5. Typhoid - Stepladder Fever
6. Typhoid - Rose Spot
7. Tetany - Chvostek Sign (Muscle Twitching Face)- Trosseau's Sign (Jerky Mov'ts)
8. Pancreatitis - Cullen Sign (Bluish discoloration preumbilical area)
9. Appendicitis - McBurney's Point (rebound tenderness)- Rovsing Sign (RLQ pain w/ palpation in
LLQ)- Psoas Sign(pain on lying down putting pressure on MB pt)
10. Thrombophlebitis - Homan's Sign
11. Hepatitis - Icteric Sclera (yellowish discoloration of sclera)
12. Meningitis - Burdzinski Sign (Pain on nape)- Kernig Sign (pain on leg/ knee area)
13. Pyloric Stenosis - Olive-Shaped Mass
14. Hyperthyroidism - Exopthalmus
15. Addison's Dse. - Bronze-like skin
16. Cushing Syndrome- Buffalo Hump
17. Cholera - Rice Watery Stool
18. SLE - Butterfly Rashes
19. Leprosy - Leoning Face (contracted face)
20. Bulimia Nervosa - Chipmunk Face
21. Liver Cirrhosis - Spider Angioma
22. Asthma - Wheezing Inspiration
23. Hyperpituitarism - CAROTENEMIA (Discoloration of skin)- XANTHAMIA
24. Down Syndrome- Single Crease on Palm
25.TOF - Clubbing of FingernailsVentricular Septal DefectPulmonary StenosisOverriding of
AortaRight Ventricular Hypertrophy
26.Cataract - Blurry Vision / Hizzy Vision
27.Glucoma - Tunnel-like Vision
28. PTB - Low grade fever in a ternoon
29. Cholecystitis- Murphy's Sign (pain RUQ)
30. Myasthemia Gravis (MG) – Ptosis (inability to open upper eyelids)
31. Dengue - Petechiae
32. Parkinson's Dse. - Pill Rolling Tremors
33. MI - Levine's Sign (Clutching of the chest)
34. Measles - Koplick's Spot

1036. Crown fracture with pulp exposure.patient comes 5 days later.what's th treatment?pulp
cap,pulpotomy,superficial pulpotomy,pulpectomy

1037. MOST COMMON VACCINE WITH INACTIVATED VIRUS?


Ans: INFLUENZA
An inactivated vaccine (or killed vaccine) consists of virus particles which are grown in culture
and then killed using a method such as heat or formaldehyde.
Examples

Types include:

 viral: polio vaccine (Salk vaccine) and influenza vaccine[1]


 bacterial: typhoid vaccine, cholera vaccine, plague vaccine, and pertussis vaccine[1]
Inactivated vaccines are contrasted with Attenuated vaccines, or "live" vaccines. Examples of "live"
(i.e. attenuated) vaccines include:

 Viral: measles vaccine, mumps vaccine, rubella vaccine, chicken pox vaccine, yellow fever


vaccine,[1] and nasal-spray flu vaccine (including the seasonal flu nasal spray and the 2009
H1N1 flu nasal spray). Rabies vaccines are now available in two different attenuated forms, one
for use in humans, and one for animal usage.
 Bacterial: BCG vaccine,[1] typhoid vaccine

1038. which is preferred in cardiac arrhythmia patient lidocaine or prilocaine 


Ans:
1. Mepivcaine without adrenaline
2. prilocaine with or without felypressin
 ( nb: limit the felypressin dose in child with coronary heart disease as it cause coronary spasm.

1039.
1)patient with rampant caries , what will be our first step?
 a) dietry analysis
b) pain relief
b) restoration of lesion

2)age for nursing caries


3)how much time avulsed tooth can stay out of mouth?
4)duration of splinting of avulsed tooth?  its given one week in churchil)
5)which method of behaviour control is used in children with a minimal occlusal cavity?

Ans:
1-b.
2- 1-3 years old, i guess.
3- 30mins the best, 2 hrs with bad prognosis.
4- 7-10 days.
5- modelling, 

Its true, immature tooth with extra alveolar time of less than 45 minutes may undergo pulp
revascularisation.
However mature tooth with dry storage of more than 1 hr will have non vital pdl.In this case pulp
should be removed at chairside and tooth to be placed in 2.4 %NaF for 20 min and root canal is then
obturated and tooth reimplanted and splint for 4 weeks.

Normally we splint avulsed tooth for 2 weeks..

1040. Perio
Tetracyclines
Tetracyclines are a group of related bacteriostatic antimicrobials. They provide a 'broad spectrum' of
activity against both Gram-positive and Gram-negative microorganisms. Tetracyclines are effective
against most spirochaetes and many anaerobic and facultative bacteria. Additional properties of
tetracyclines that may be valuable in the management of periodontal diseases are
• inhibition of collagenase
• anti-inflammatory actions
• enhancement of fibroblast attachment to root surfaces
• inhibition of bone resorption.

In periodontal treatment, metronidazole has been used systemically; common dosage is 200 mg
three times a day for 3-5 days. For more severe infections thedose is increased to 400 mg twice
daily for 3-5 days.

1041. which is the most standard radiograph method used for patients need orthodontic treatment?
OPG
lateral cephalo
pa
true occlusal

1042.  How does calcium hydroxide produce calcific barrier? 


its has a basic/alkaline ph , but how a calcfic bridge is formed?

1043. Intraligamentary anaesthesia


Yes it affects permanent tooth germ..i causes tooth hypoplasia...
it is very painful as LA is being injected in very small place..it causes discomfort due to mild
extrusion of the tooth in socket as well. needle is smaller than our normally used needle 4 LA

1044. WHY PERIAPICAL GRANULOMA BECOME ASYMPTOMATIC? 


because of pulpal necrosis,nerve supply is lost

1045. Junctional epithelium derived frm?


Inner enamel epithelium which fuses with outer enamel epithelium and forms reduced enamel
epithelium which forms junctional epithelium later.

1046. Cells that can't be seen in pulp?


odontoblasts,fibroblasts,undifferentiated ectomesenchymal cells and macrophages are the cells of
pulp.so it could be anything except these cells.

1047. Principal substances controlling ca2 and metabolism?


Options calcitonin,pTH,vit D etc
Ans; vitamin D

1048. appropriate investigatio for a patient with excess salivation on eating


options:
MRI,IOPA
SAILOGRAPHY
lower standard occlusal, DPT

Sailometry is used to assess the salivary flow.

1049. Which enzyme cause destruction of periodontal ligament?


Matrix metallloproteases cleaves type 1 and iii collagen so could cause destruction of pdl

1050. which fluoride varnish is c/i in porcelain crowns?


ACIDULATED FLOURIDE

The acid in acidulated fluoride is hydrofluoric acid, which has the property of etching porcelain and the
tiny glass particles in many composites, and thus it will make them more susceptible to staining. It can
also actually remove the color from some porcelain crowns!

1051. - what does a single motor neuron to a lingual musculature supply?

Muscle spindle

Single muscle fiber 

Multiple muscle fiber

2- what kind of neurons supply mastication muscles ?


Low conduction

Fast conduction

3- ten years old amalgam raised on the tooth surface - reasons why it's raised? - creep

4- Which orthodontic appliance requires least patient compliance? - fixed

5- Which fracture will lead to meningitis? lefort 2 and 3

6- Which fracture will lead to emphysema? orbital blow out fracture (emphysema is a feature of lefort 2
and orbital fracture )

1052. Patient with xerostomia will benefit more from(tooth brush,flouride rinses,scaling and
polishing)

1053. 1- bpe exam. Is it the WHO probe e or which one ?


2- furcation involvement. Is it curved probe?
3- plaque index silness loe.
4- plaque index quigley and hein.
5- oral hygiene index Greene vermillion.

Ans;
1. CPITN probe or WHO probe
2. NABERS probe, its curved
3, 4,5 i guess normal straight probe..not sure though

1053.
1. RECALL FOR A PT WITH ACTIVE RAMPANT CARIES:
( 3M, 6M, 12M )

2. MAXIMUM TIME WHERE TOOTH CAN STILL BE REIMPLANTED AFTER AVULSION

3. DIETRYY ADVICE BEST GIVEN ( BY DENTIST TO CHILD WHAT`S THE GOOD AND
THE BAD FOOD , BY NURSE TO THE CHILD, TELL THE PARENTS NOT TO GIVE SWEETS
TO CHILD, SEND THE CHILD TO DIETICIAN).

Ans;

1. 6-12months is given in master dentistry for high caries risk. So we can choose 6 months here

2. We dont reimplant primary teeth, so its always a permanent with either root completely formed or not. 

After 30 mins pulp dies and after 60 minutes PDL. 


So safely we can say 1 hour time will have good prognosis, however depending upon the options given and
considering age of the patient, we can replant even after 60 minutes to maintain space and alveolar ridge
contour.

RCT is required for 


 closed apex replanted before or after 60 minutes
 open apex if replanted after 60 mins

Splint is removed
 after 2 weeks if replanted within 60 mins
 after 4 weeks if replanted after 60 mins

http://www.dentaltraumaguide.org/Permanent_Avulsion_Treatment.aspx

3. According to pink book # 42: many patients relate better to advice from a hygienist or nurse. So maybe the
answer is by nurse. What do u think?

RAMPANT CARIES
It is important to consider the many factors that determine the treatment of a child with a high caries
rate (509HFig. 9.48). If the child presents with an acute problem of pain or swelling, then
immediate treatment is indicated to relieve the child of the pain.
After that, it is important that the clinician considers the attitude of the child and his or her parents
together with motivation towards dental treatment, the co-operation of the child, the age, and the
extent of decay.
It may be possible to place temporary restorations while preventive strategies arecommenced. These
will include:
1. Dietary analysis and appropriate advice to the child and the parent.
2. Plaque control, oral hygiene instruction depending on age to the child or the parent,
the techniques of toothbrushing, and disclosure.
3. Fluoride
-tooth paste
-mouth rinse;
-varnish application every 6 months.
4. Fissure sealants
5. Regular recall.
Once the caries is under control, definitive restorative treatment can commence

1054. How tooth mobility can result purely due to heavy occlusal loading?
traumatic occlusion

1055. Inicial caries lesion surface characteristic?  


Chalky white surface..this is the characteristic feature off incipient /initial caries lesion

1056. Patient with desequmative gingivitis and epistaxis , what does he have ? 
Ans: mucous membrane phemphigoid
http://books.google.co.uk/books?
id=fOLaR6PCU8sC&pg=PA25&lpg=PA25&dq=desquamative+gingivitis+epistaxis&source=bl&ots=zt
9dOa-
LgI&sig=xNqxxxHK2RqJlyIoGYuvuxFfaYY&hl=en&sa=X&ei=m8pbT6X_CI2S0QXBg4m_DQ&ved
=0CCgQ6AEwAA#v=onepage&q=desquamative%20gingivitis%20epistaxis&f=false

1057. Patient with malar rash extending across nasal bridge what oral finding will she have?
Its a characteristic sign of SLE...BUTTERFLY RASH
SLE  Oral manifestations
Painless, shallow oral ulcers, most often occur on the hard and soft palate. There is also
a mild involvement of mucosal ulcers as symptom of this disease. 
Oral ulcers occur at onset in 11% of patients, while at any time are present in 30% of
patients. The lesions appear as maculae (red patches) that will later transform into
irregular erosions and ulcers which often heal with scarring. Purpuric lesion such as
ecchymoses and petechiae may occur.
In 30% of the cases, pathology of major salivary glands may occur leading to secondary
Sjogren’s syndrome and severe Xerostomia
 
1058.
1- child with ulcers on soft palate ,fever ,malaise what is the diagnosis?
2- child with fever ,blisters on trunk and mouth what will be the diagnosis ?
3- analgesia that cause ringing in ears?
Ans:
1. Herpangina
2. Chickenpox
3...aspirin
            

1059. Background radiation.


2 b/w--F speed films, rectangular collumator,70 kv--6.4 hrs
          E speed films, rectangular collumator,70 kv--8HRS
          D speed films, rectangular collumator,70 kv--32 HRS
1060. Which restoration is used in tooth with amalgam nayar core?  
Purpose/Objectives: To describe a coronal-radicular dowel and core technique for endodontically
treated posterior teeth. Discussion: Diagnosis is the first consideration in selection of the
appropriate restorative treatment. The following criteria must be evaluated: 1) the size of the
remaining pulp chamber should be of sufficient width and depth to provide for adequate bulk of
amalgam and for retention and 2) adequate dentin thickness in the area of the pulp chamber is
required for rigidity and strength. The amalgam dowel and core can be completed at the end of the
obturation appointment. All gutta percha should be removed from the pulp chamber and to a depth
of 2-4 mm into each canal. If needed, a matrix band or copper band should be placed. Amalgam is
condensed into the canals with a periodontal probe or root canal plugger and into the pulp chamber
and remaining cavity by conventional methods. If fast-setting amalgam is used, the tooth may be
prepared for a cast restoration immediately after hardening and a final impression can be made at
the same appointment. Summary: The described restorative method is a viable method for restoring
endodontically treated posterior teeth and is a method that can reduce cost and time to the patient
and practitioner. Over a four year period of time, approximately 400 restorations of this type had
been placed at the authors institution without any failures attributable to the amalgam dowel and
core reported.

1061.
1- Patient for mitral valve replacement which drug?

2- Patient had stroke which drug?

1062.
1- 12 years old child who lost his central incisor
2- sport person who lost his central incisor

Fixed - fixed conventional 


minimal preparation 
Implant
Fixed movable
Cantilever 

Ans:
1.minimal preparation fixed bridge
because it can be changed into an implant,when the child grows...
2.implant

1063. How would u treat a person with TB,HIV,Hepatitis,MRSA,...options(send them to a hospital,use


single equipment,treat using standard conditions of infection control
Ans:
for all these condition u can treat normally using the standard precautions.
In case of TB : pt with open pulmonary tb is contagious and ttt should be deferred until after disease is
controlled, however in case of er take adequate precautions to limit infectious:

1. use high volume suction


2. rubber dam
3. personal respirator (HEPA filters)

4. adequate ventilation

5. avoid ultrasonic to limit the areosole


check it in scully
the only exception IS CJD you need extra precautions :
or treat in the dental practice but u need to :
1. incinerate the instruments after us  
2, sterilization cycle for 134 for 18 min  instead of the standered 4 min 

1064.
1-Restoration to be given if marginal ridge is to be protected,contact point is to be retained
2-Restorative complexity index

3-values of pulp tester to diagnose the condition, pulpitis, sinusitis,  abscess etc .

4-Materials used for crowns in various conditions..

5-Matrix band poor technique – marginal overconstruction, underconstruction, open contact point,
overhang.

Ans;
1. It is given in pink book, RMGIC
2. FOR RESTORATIVE COMPLEXITY INDEX
HERE IS THE LINK
http://www.rcseng.ac.uk/fds/publications-clinical-guidelines/clinical_guidelines/documents/
complexityassessment.pdf
5. In my opinion,improper matrix band can lead to overcontour,undercontour as function of matrix
band is to restore anatomical contours and establish good contact.
Though marginal overconstruction can be corrected (by polish properly) and underconstruction is
due to inadequate filling which again can be corrected.
But if improperly placed band,it means might be placed too gingivally and can lead to overhang.

1065.
1- what component of amalgam was formerly used as a scavenger but is no longer widely used? -
Zinc

2- strength of amalgam depends on? - Copper


3- black stain of amalgam tattoo is caused by? - Silver
4- what component causes tooth blackening? - Silver

Zinc
Copper
Silver
Tin
1066.
1. Ideal rake angle achievable by subgingival curettes
2. what`s required to increase the efficiency and decrease the fatigue of universal curretes
3. what helps not to get tired while using curettes something related to the size of the handle

Ans:
1 NOT SURE THINK 0 
2 SHARPENES
3 GRIP  
Explanation:
Ideal rake angle for universal curettage subgingivae is -20
Rake angle achieved by universal curette in subgingival curettage is 0
If leading edge is ahead of prependicular(acute angle),rake angle would be negative and if
behind(obtuse) rake angle is 0.

1067. Which two of the following conditions present as complete vesicles

A. Pemphigus
B. Herpes simplex
C. Aphthous ulcer
D. ANUG
E. Erythema migrans
F. Erythema multiforme

I just goggled and found All these conditions form vesicles


atropic dermatitis
bullous pemphigiod
chicken pox
contact dermatitis
herpes simplex 
impetigo
Erythema multiforme

1068.
1- what's choice of crown with MOD filling and fractured palatal cusp?

Full gold crown


3/4 gold crown
Porcelain bonded crown 

2- what cement you use to glue alumina core ceramic crown?

GIC
Panavia
Zinc phosphate
RMGI
Poly carboxylate
Panavia is self-etching advanced esthetic resin cement that bonds directly to metal and silinated
surfaces with no need for a bonding agent. So,ii think it should be the first option and then RMGIC.

1069.
1- Which one remove smear layer? - Conditioner
2- which one open dentinal tubules? - Conditioner
3- which one increase surface energy ? -Primer

1070.
Ideal time for hand scrubbing (1,2,3,4,5)
Best solution used for hand scrubbing?
Ans:
2-3 minutes
Povidone Iodine or chloroxedine Gluconate (7.5  and 4)

1071. THE GREATEST PREDISPOSING FACTOR TO ANUG


IS IT HIV OR SMOKING?

1072.
1- LA for patient taking Beta blocker is it Articaine?
2- LA for hyperthyroidism
3- LA for hypothyroidism 
4- LA for pregnancy is lidocaine with epinephrine ?
5- LA for sever hypertension is Mepivacaine or prilocaine ?
6- LA for unstable arrhythmia is Mepivacaine or prilocaine ? 
7-LA for diabetes mellitus is it lidocaine with epinephrine?
8- LA for child is it Lidocaine with epinephrine ? 
9- LA for Myasthenia Graves is it Prilocaine ?
Ans:
1.yes,it is articaine(scully)
2.Usually in hyperthyroidism,epinephrine is nt given as can cause thyrotoxicosis,but acc to scully it
is therotical and we can give lignocaine with less dose of epinephrine as prilocaine with fleypressin
doesnt show to have better results.
3.as such no complications,we can give normal la.
4.In pregnancy,lidocaine and novicaine are the best anesthetics.
5.hypertension is a vast topic,and we dont preffer giving epinephrine to patients taking b-blockers
as may induce hypertension.and lidnocaine should be used with precaution.if pressure is below
200mm\hg and 115(diastolic)we can give lignocaine and epinephrine.
6.Both can be given
7.la has no effect on blood sugar level so normal la can be given though early morning
appointments will minimize risk of stress induced hypoglucemia.(which is the only issue with
diabetics),so best to give oral glucose before appointment.
8.yes
9.Anything lignocaine,nupivacaine or prilocaine can be given.
For conscious sedation in these patients,inhalation sedation is preffered.

1073. WHAT ARE THE GRAM POSITIVE BACTERIA ASSOCIATED IN PERIODONTITIS??


Majority of oral organisms are facultative anerobes.
Organisms that predominates early plaque are gram positive facultative coci and rods
ie streptococcus sanguis
    Actinomyces viscosus

Organisms that predominates late plaque are Gram negative anerobic roda and filaments
P.gingivalis
P.intermedia
fusobacterium
capnocytophaga

Regarding chronic periodontitis,mostly are anerobic(90%)and gm -ve species(75%)

1074.
Quest 1: How would you design a partial denture so the patient can taste food and the
denture be aesthetically acceptable
a. Uncover the anterior part of the hard palate
b.Uncover the posterior part of the hard palate

2.Which position will you put a patient having a myocardial attack?


a. Lying down
b.Recovery position
c.Sitting up

3. After starting the CPR,you realise that the patient has a ventricular fibrillation,how would you
proceed?
a. Stop CPR for 2 minutes
b.Give oxygen
d. 200 shots with the defribrillator

4.How do you get incisal guidance for setting incisal teeth?


Study casts
Unsupported/supported labial fullness

5.Risk of cancer in dental Radiography (IOPA)


1:20,000
1:200,000
1:20,00000
1:200, 00000
ACCORDING TO ERIC WHITES
ESSENTIALS FOR DENTAL RADIOGRPHY AND RADIOLOGY
THE ESTIMATED RISK OF FATAL CANCER IS:
1.FOR TRADITIONAL BITEWING AND PERIAPICALS IS 1:2000 000 (50KV,D speed film , 10cm
fsd)

2.MRODERN BITEWING AND PERIAPICALS 1;20 000 000  (70kv, F speed films, 20fsd)
3. panorama 1 in 1 000 000
3. head CT 1 IN 10 000
5. SKULL PA 1 in 670 000

1075.
Composition of the tooth paste:
1. Humectant is 10-30% : glycerol, sorbitol, propolyene glycole used to retain the moisture and prevent the
Harding of the paste on exposure to air
2. detergent 1-2% lower the surface tension of the paste facilitating its distribution in the mouth (sodium
lauryl sulphate, sodium N lauryl sarcosinate)
3. Abrasives : is the major constituent 30-40%: silica, hydrate dalumina calcium carbonate,sodium
metaphosphate
all the toothpastes in UK have a standard amount of abrasives set by the British standards society

4. BINDING AGENT 1-5% : Algenate , gum cellulose


they prevent the separation of the solid and the liquid ingredient during storage
5. flavouring substance as u said 1-5%
6. preservatives: 0.5-0.05 preven the bacterial growth ( alcohol, benzoate and formaldhyde
7. colouring agent

THE THERAPUTIC AND PREVENTIVE AGENTS:


1. FLOURIDE: ADULT 1350-1450

CHILDREN: WE DON`T USE 500 PPM ANYMORE


FOR CHILDREN LESS THAN 3 USE 1000 PPM BUT IN A SMEAR FORM THE ONLY EXCEPTION ARE PT
WITH LOW CARIES AND WELL FLOURIDATED AREAS U CAN USE 500 ACCORDING TO MASTER BUT
NOT ACCORDING TO THE RECENT GUIDLINES

2. desensitizing agent : formaldhyde, strontium chloride, potassium nitrate

3.Antiplaque: triclosan
4. anti calclus: sodium pyrophosphate, gantrez acid, biphosphonates
5. bicarbonate: to decrease the acidity of the plaque
6. xylitol: sweeten the tooth
paste and provide an antibacterial action byreducing the level of streptococuss mutans and enhancing the
remineralization

1076.
Ulcers not common in which type of viral infection
Coxsackie , EBV, rubella virus, hand foot and mouth...

1077. Extracranial cause of facial palsy(stroke,melkerson-rosenthal syndrom,bells palsy)


Is it bells palsy?
Bells palsy
HIV
HEPES SIMPLEX
GULLIAN BARE DISEASE
GRANULOMATOUS DISORDER
TRAUMA
LOCAL ANESTHESIA
PAROTID MALIGNANT NEOPLASM
LYME DISEASE

1078. 1 Best radiological xray for bilateral condyles#. Is it DPT?

2And for careis in 3 yrs old ?

3For successor teeth in a young child

1079.  What is nayyar core?


 FOLLOWING THE ROOT CANAL TREATMENT AND The RETORATION OF THE TOOTH
WITH AMALGUM. Remove 3-4 mm from the coronal RCT filling so it would be filled with
amalgam

1080. 1] best sedation and analgesic in following situations

a. severly disabled patient


b. epileptic patient under control, cominf from work
c. pregnant women requiring rct, anasthesia tried by previous dentist did not act
d. epileptic patient under control, has pulmonaryproblem, and gag reflex..extraction to be done.

2]patient with incior crown comes with blue marginal gingiva..what to do


3]tumour found in teenagers appears like OKC

4]which area in uk has gor best oral health

1081. how many weeks for the maternity leave should be ? is it 52 weeks?
and how many weeks of them should be paid? is it 39 weeks?
Ans:
THE MATERNITY LEAVE IS 52 WEEKS
39  WEEKS ARE PAID

IN THE FIRST 6 WEEKS: 

THE EMPLOYEE IS PAID 90% OF AVERAGE WEEKLY PAY THERE IS NO UPPER LIMIT
IN THE  REMAING 33 WEEKS: 

SHE IS PAYED THE SLANDERED WEEKLY RATE OF 128.93 OR 90% OF THE WOMEN AWE
IF ITS LOWER THAN THE STANDERD WEEKLY RATE
1082. Nitrous oxide is?

> Soluble in blood
> Insoluble in blood 

1083.
1- how many ml of lignocaine 2% 1/80000 can be injected to 20kg person?

2.2 ml
4.4 ml
6.6 ml
pink book says 

lidocaine 2% with 1:80,000 adrenaline ,max dose = 4.4mg/kg 

 2.2 ml for a healthy 10kg  1.5 yr old child

so  2.2 for 10 kg
       x   for 20 kg 
= 4.4 ml
Dose of lignocaine without adrenaline is 4.4 mg\kg
With adrenaline its 7mg\kg

2- how many ml of lignocaine 2% 1/80000 can be injected to 125 kg person ?


16 ml
24 ml
26 ml
30 ml

1084. Local anaesthesia that cause neuropathy?


Articaine

1085. renal disease


Drugs contraindicated
Paracetamol, penicillin, tetracycline

1086.
Which drug causes rashes in infectious mononucleosis ?

Penicillin v
Penicillin g
Aspirin

1087.
What's the antibiotic used to manage super infected herpetic lip lesion?

Ciprofloxacin

Metronidazole 

Cefuroxime

1088.
Anaesthesia and Sedation 5 sub questions for different clinical situations – 
Anxious pregnant woman-conscious sedation
Not-cooperating child with 4 molars to be treated- inhalan sedn
Nervous patient for extraction- local anesthesia
Normal pt for simple restorative procedure – local anesthesia
Anaesthesia indicated for patients with COPD –GA
Anaesthesia indicated for pregnant woman –conscious
Anaesthesia indicated for epileptic patient –GA

1089. Glandular fever

ampicillin 

1090.
what type of lesion occurs in the following?

A pt with deviation of th corner of his mouth when he smiles,but wrinking of his forhead - UMNL

A patient with bells palsy LMNL

A patient with deviation of the corner of his mouth when he smiled but cannot wrinkle his forhead
LMNL

1091. A man with multiple myeloma comes for an extraction and comes back 6 weeks later and his socket
has not healed what can cause this? a. Multiple myeloma of the mandible b. Drug induced osteoradionecrosis
c. Dry socket d. Drug induced osteosclerosis –

1092. All maxillary teeth moving together .what's th fracture?

Lefort 1

1093.
1.most common food infection in UK
2.main diagnostic feature of Rheumatoid arthritis.. is it ulnar deviation

3.amalgam lichenoid reaction..which type of hypersensitivity

Ans
2- yes ulnar deviation and swan - neck deformity 

3- type 4 delayed

1094.
Question 1: patient 21 years old complains of headache who has got university 

Is the answer will be tension headache or stress or anxiety

2 ques: . In an dentate patient the teeth periodontal ligament can respond to? 
a.pain, b. temperature,c.pressure 

3 ques: Type of receptors in the PDL of a dentate adult


Ans:
3-mechanoreceptors (It has nociceptors for pain
mechanoreceptors which respond to physical stress caused by touch and pressure
receptors for propioception for positioning of mandible and biting forces.)

1095. Microorganism causing urinray tract infection?


In acute uncomplicated infection, the organisms are those normally colonising the perineum:

 E. coli - 65 to 80% of all UTIs


 Proteus
 Klebsiella
 Enterobacter
 Candida
 Enterococci
 Staphylococci - especially, Staph. saprophyticus in young, fit adults, especially women
In complicated UTI’s, more unusual organisms are found - such as:

 Pseudomonas aeruginosa
 Staphylococcus epidermidis
 Klebsiella
 Enterobacter
 Proteus
 Enterococcus (1)
Occasionally, organisms secondary to systemic disease arising elsewhere - such as:

 Tuberculosis
 Salmonellosis
Rarely, viruses:

 adenovirus types 2 and 11, in children


 Herpes simplex

1096. Single visit is equivalent in outcome to multiple extra in what situation


A. Vital pulp with acute pain
B. Necrotic pulp with acute pain
C. Necrortic pulp without pain
D.necrotic pulp with draining sinus

1097. Q oral features of gardeners syndrom?


multiple impacted supernumerary and permanent teeth,
osteomas of the bones,including long bones,skull and jaw bones
multiple epidermoid and sebaceous cysts.......
 

1098. Where is th teeth ideally placed in lower jaw during setting?


Ideally in lowers teeth are place on the crest of ridge.

1099. What's hanau principe of articulation?


Hanau principle of articulation is basically the factors that affects the balanced occlusion ... 5
factors are:
1) condylar guidance
2) incisal guidance
3) plane of occlusion
4) compansatory curve
5) cuspal inclination

1100.
1- the angle between prepared and unprepared surfaces in amalgam restoration occlusal?
2- the angle between prepared and unprepared surfaces in amalgam restoration gingival ?
3- cavosurface angle cvs?
4- AMA angle?
Ans:
1. 90 degree
2. 45 degree
4. AMA is amalgam marginal angle i.e the angle between amalgam restoration and external cavity wall. It is
70 degree according to pink book.

1101.
1- Setting contraction of composite 
2- setting contraction of GIC
3 - setting contraction of RMGIC

1102. Which cells are found in Herpetic stomatitis ?

Lymphocyte
Monocytes 
Neutrophil 
Macrophage 
Plasma cell

1103.
1- antibiotic action of trimethoprim 
2- antibiotic action of ciprofloxacin 
3- antibiotic action of acyclovir 
Ans;
Trimethoprim – antimetabolite action
ciprofloxacin inhibit DNA gyrase
Aciclovir inhibit viral DNA synthesis. 

1104.
1.FOR A CHILD WITH 4 TEETH TO BE TREATED WIL U CHOOSE CONSIOUS SEDATION
OR GA?

2. TISSUE THAT HEALS WITHOUT A SCAR?

Ans: 1- It depends of treatment if it's extraction it's most likely GA


If its restorations could be done inhalation
But generally for children inhalation is the best Option

2- Oral mucosa

1105. Black stain of amalgam tattoo is caused by ?

Silver
Mercury 

1106. Antibiotic for ear use?

1.AMOXICILLIN FOR OTITIS MEDIA FIRST LINE

1107.
Ore questions about wheatear it's DNA or RNA virus
1- Hepatitis A
2- hepatitis B
3- hepatitis C
4- HIV

I think

1- RNA non - enveloped 


2- DNA double shell structure / enveloped 
3- RNA enveloped
4- RNA or retrovirus 

1108.  patient with deviation of th corner of his mouth when he smiles but wrinkling of his forhead
is normal
It’s upper motor neuron lesion once he can raise his eyebrows and wrinkles are normal

1109. Whts the best treatment for a patient with denture stomatatis with resisted angular chelitis
(options. Miconazole lozenge,fluconazole,nystatin..etc
Ans;
for denture stomatitis Fluconazol 
RESISTANT ANGULAR CHELITIS: ITS MICONAZOLE 2%  WITH HYDROCONAZOLE 1% GEL
OR OINMENT

1110. Bacteria that cause urinary tract infection?

E coli
N gonorrhoea 
Treponema palladium 
Chlamydia 

1111. Bacteria that most commonly cause infection in digestive system in UK?
Is it campylobacter or helicobacter pylori?

1112. 1) 55yrs old lady with increaded production of growth hormone


Acromegaly

2) 2yrs old boy with increased production of growth hormone


Gigantism

1113. In which condition is there separation of epithelium at th basement membrane

pemphigus
Pemphigoid
Erythma multiform

1114. A patient with skin pigmentation is most likely to have a) addisons disease,b)crohns
disease ,c)cushings syndrome
1115. intact vesicles or bullae are frequently seen in the mouth in:
A- pemphigus bulgaris
B bullous erythema multiforme
C- liquen planus 
D- herpetic stomatitis 
E- mucous membrane phemphigoid

1116. 1. Commonest site for oral cancer


C
2. Commonest site for malignant melanoma in mouth
Posterior part of palate and alveolar ridge is the common site for malignant malenoma.
OPtions: a.palate b.buccal mucosa c.ventral surface of tongue d.lips

1117. Which is least likely to be unilocular?


a,ameloblastoma, b. OKC, c, aneurysmal bone cyst d.myxoma, d,osteoporosois 
Ans: cawson says unicystic or unilocular ameloblastoma is very rare

1118. Clinical presentation of


• Asthma - expiratory wheeze
• Chronic bronchitis -daily mucous production
• Tumour in the upper lobe of the lungs -dry cough
• Codeine use
• Pethidine use
• Carbamazepine use
The options were: inspiratory wheeze, expiratory wheeze, stridor, daily mucus production, dry
airway, dry cough, constipation.

1119. what is the maximum dose of LA with adrenaline in children.

-7.7mg/kg
-2.2mg/kg
-4.4mg/kg
-6.6mg/kg

1120. a. pt. with bleeding gums- chronic gingivitis

b. pt. with foetid oris pockets al over the teeth OPG shows bone loss- generalize periodontitis
c. 21 yr old pt. who is a smoker since 2 years, bleeding gums and metallic taste 
in mouth - NUG
d. patient with pocket in relation to UR6 and UPPER incisors- localised aggressive periodontitis,
options : chronic gingivitis, generalize periodontitis . localised aggressive 
periodontitis, NUG

1121.  Most common congenital cardiac defect? 


it is given in the skully that ventricular septal defect is the most commonest .

1122. For 3rd molar extraction what is th patient most likely to have?
Is it trismus? Or dry socket?

And if th molar lies close to th ID nerve what he likely to have? 

1123. IS  NITROUS OXIDE  IN ASTHMATIC PT CONTRADICTED ?


Ans: ACCORDING TO MASTER 2
NO 
ON THE CONTRARY TS INDICATED IN condition aggravated by stress
1. ASTHMA
2. EPILEPSY
3.HYPERTENSION
4. ISCHEMIC HEART DISEASE

however its CI in COPD 


since their respiratory drive depend son partial pressure oxygen instead  oF PP CO2

1124.
1.Salivary gland that release the least mucous secretion: labial glands, palatal glands, parotids,
submandibular, submaxilar

2.Origen of internal auditory meatus and middle ear: 1st, 2nd, or 3rd branchial arch, 1st, 2nd,
or 3rd branchial pouch

3.Most likely place to develop a MRSA infection: groin, nail, nostril, armpit, etc

Ans:
 
1. parotid 
2.1st branchial pouch
3. nostrils

1125.
1.Patient is insulin dependent diabetic, complains of faint after prolonged dental
session, what is best to give, (SAQ)
Oral Glucose
IV Glucagon
Insulin
IM Glucagon

2. Patient is waiting in waiting room and collapse and faints. Patient is cold, clammy, but pulse is
good. What do you give?
IM Glucagon
Oral Glucose
GTN
Hydrocortisone
Diazepam
For the first question the patient 'complains' of faint.. which means he/she is still conscious, hence u
can give oral glucose, but u ll have to switch to i.m glucagon if the patient becomes unconscious.. 

2nd quest: the patient has fainted.. u can't give anything orally to patient who is not conscious, isn't
it.. now comes the question about diagnosis.. gtn is out of the question- its for cardiac pain/
conscious patient. diazepam- we don't give in dental setting but its for status epilepticus,
hydrocortisone- an adrenal crisis is a very rare emergency and is preceded by anxiety, even though
its possible that a patient can be anxious in the waiting room, its highly unlikely that it will cause an
adrenal crisis. hypoglycemia is the only option that fits, since the patient is unconscious, I.M
glucagon will have to be given. 
other causes of collapse can be a syncope- t/t would be raise patients legs, give oxygen (but they've
not given such an option). or a cardiac arrest, which is excluded here because the patient has a
pulse.. 

3. A 90-year old gentleman presented to clinic who is edentulous and has dentures upper and lower
full 15-years old. Denture bit uncomfortable, tooth structures little bit worn out, freeway space 2-
4mm, polished surfaces satisfactory and occlusal wear minimal. How would you proceed?
Copy dentures
Hard reline
Soft reline
Construction of new dentures

4.Patient had trauma on 14 (vital pulp), only thin buccal cusp remaining, best treatment option
MOD-Onlay 
Inlay
Direct composite MODL
Extract and options for replacement

1126.
1. EMQ based question to match the options
Apical periodontitis
Lateral apical abscess
Crack tooth syndrome
Dentine hypersensitivity
Hyperemic pulp
Reversible pulpitis
Options
Intermittent pain from molar which is having huge restoration leaking HYPEREMIC PULP
Prolonged intense pain for several days  LATERAL APICAL ABSCESS
Acute pain which decreases 10-15 minutes after the stimulus is removed REVERSIBLE
PULPITIS
Intermittent pain on biting - apical periodontitis
Intermittent pain with hot things DENTINE HYPERSENSTIVITY

2.Best ways to reduce radiation for patient who is going to have IOPA X-rays
Lead apron
Paralleling technique
Rectangular collimation
D-Film

1127. A patient presents with a history of clicking from their temporomandibular joint. This
click occurs mid way through the opening cycle and is consistent. There is some pre-
auricular pain and the lateral pterygoidf muscle on the affected side is tender to resisted
movement test. There is no trismus and the click is not present when the patient opens
from an incisor edge to edge relationship, instead of her normal Class I occlusion. The
patient would like treatment.
The most appropriate occlusal splint for this patient would be:
A. Stabilisation splint
B. Localised Occlusal Interference Splint
C. Bite Raiser
D. Soft Bite Guard
E. Anterior Re-positioner Splint

answer

A.Stabilization Splint: mainly used for the correction of the occlsuion or creating a perfect
occlusion for patient with TMD reflected from occlusal interference, till the muscles
return to their normal tension and lengh, then restorative or selective grinding work
will b done

B. Localized occusal splints: used for clenching and bruxism, focusing the
occlusion on less number of teeth, making their propioseptive response sensetive
for biting, so the brain avoid excessive biting and then decrease the Bruxism

C.Bite Raiser: used for the raisong of the bite to relive muscle stress mainly

D. Soft Bite Guard: first line of treatment in symptomatic bruxer, act to decrease
muscle spasm and decrease the habbit.

E. Anterior repositioning splint: thats my choice in that case, it is done as we say


[to catch up the disc] as the patient doesnt experience clicking when he opens
from Edge to Edge, thats means that first stage of translation is executed in this
edge to edge movement where the condyle and the disc are aligned as in normal
position. keeping this realtion for a long time allows the retrodiscal tissues [where
pain is moderate ] to heal on this level and the pterigoid spasm will b decreased.

1128. Anaesthesia indicated for epileptic patients


1- procaine
2- benzodiazepine
3- nitrous oxide
4- thiopentone
According to  scully , inhalation sedation and IV sedation may be indicated for epilepsy
As stress is a factor that might lead to fits
Nitrous oxide should be safe although can increase CNS depression
Benzodiazepine are anti epileptic although there were cases with fit when using it

1129.
orange bag-  all clinical infectious waste , so all clinical waste contaminated with saliva ,
blood goes in this.
its will be incinerated or alternative treatment

yellow bag with black strip-  offensive/ hygenic waste |(where no known of infection


risk is present) -  so gloves ,gown etc nont contaminated with blood,  medicines and
othr chemicals goes in this

3) yellow box , (SHARP BOX) -  needles, wedges,sharp instruments, medical vials,


teeth without amalgam, syringes, syringe barrels contaminated with medicines
( except cytotoxic and cytostatic)

4) box with BLUE lid-  Non cytotoxic and non cytostatic medicines + out of date
stock

5) Orange lidded sharp bin-  sharps not contaminated with medicinal products.


( england and wales)

( Scotland n N. Ireland) - 
fully discharged or those used not used for medicines - orange lid
partly discharged with medicine -  yellow lid. 

 
6) purple lid - cytotoxic n static

1130. YOU TAKE A PANORAMIC RADIOGRAPH OUT OF A PT'S RECORD BUT YOU FIND
THAT THE FILM HAS LOW DENSITY AND POOR CONTRAST. WHICH OF THE
FOLLOWING ERRORS COULD LEAD TO LOW DENSITY AND POOR CONTRAST?
 
1.TOO LONG A DEVELOPMENT TIME
2.DEVELOPER TEMPERATURE TOO HIGH
3.DEVELOPER TEMPERATURE TOO LOW
4.INADEQUATE FIXATION
5.POOR FILM STORAGE AFTER PROCESSING

1131.
a. inhibit bacterial cell wall synthesis - PENICILLINS
b. inhibit bacterial protein synthesis - MACROLIDES
c. disrupt bacterial DNA - METRONIDAZOLE 
d. anticollagenase effects – TETRACYCLINES

1132.
which is the most risk for cancer??smoking,alcohol,sunlight,tobacco,paan chewing,betel nut etc???

which is the most synergistic risk of cacer??smoking and tobacco,smoking and alcohol,alcohol and
tobacco etc

1133. what is the type of collagen seen in drug induced ginigval enlargement?
Fibrotic gingival enlargements induced by phenytoin or nifedipine were examined with special
reference to type VI collagen expression. Immunolocalization studies showed abnormal
accumulation of type VI collagen around the collagen fiber bundles in the fibrotic gingival
enlargements. Examination of total RNA extracted from fibroblasts and tissues of enlarged gingivae
demonstrated increased type VI collagen steady-state mRNA levels. These results suggest that
excessive deposition of type VI collagen in drug-induced gingival enlargement is attributed to
increased expression of the collagen genes. 

1134. Ulcer on lateral border of tongue what drug cause it?


ACE inhibitors
Ca,cium channel blockers
Potassium sparing drugs 
Ans:
ITS ACE INHIBITORS
SCULLY TABLE IN PAGE 113
6TH EDITION

1135.
Horizontal Bitewings
These radiographs are likely to be taken routinely for assessing caries.
They may also give early warning of localised bone loss and the presence of
subgingival calculus. The normal positioning of the film should automatically ensure
a non-distorted view of bone levels in relation to the CEJ’s.
Vertical Bitewings
Correctly positioned, this type of radiograph should also give a non-distorted view of
bone levels in relation to CEJs, in opposing arches. However they can only be
positioned accurately in patients with quite tall palatal vaults. Selected paralleled
periapicals may be more appropriate in cases with more difficult access and where
the teeth are restored, in order to assess apical status.
Paralleled periapicals
The “gold standard” radiograph for periodontal cases. Correctly positioned this
radiograph will give an accurate, non-distorted two dimensional picture of bone levels
in relation to both CEJs and total root length. This technique involves the use of a
paralleling device, of which there are several on the market. These devices take the
guess work out of radiography and you will achieve better and more consistent results.
Visualising root anatomy in its entirety can be very useful in assessing bone level in
relation to total root length in:
• Assessing prognosis
• Helping to assess furcation involvements
• Identifying possible endodontic complications
http://www.bsperio.org.uk/publications/downloads/Young_Practitioners_Guide.pdf 

1136.
A. Acromegaly
B. Cranio-facial dysostosis
C. Fracture of base of skull
D. Haemoglobinopathy
E. Histocytosis X
F. Hypoparathyroidism
G. Metastastatic disease
H. Multiple Myeloma
I. Osteogenesis imperfecta
J. Osteopetrosis
K. Paget’s disease
 
Most likely diagnosis for each of the following abnormalities of the skull:
 
1) The skull X-ray of a 6 year old Greek Cypriot child reveals widening of the bones with a hair-on-
end appearance. - Haemoglobinopathy
 
2)The skull X-ray of a 55 year old male who presents with headaches reveals prognathism and an
enlarged sella turcica. - Acromegaly
 
3) the skull X-ray of a 32 year old female who complains of headaches and occasional muscle
cramps reveals flecks of calcification around the base of the brain. - Hypoparathyroidism
 
4) The skull X-ray of a 72 year old male who presents with headaches, weight loss and backache
reveals numerous small lytic lesions within the vault resembling a pepper pot. - Multiple Myeloma
1137.
Patient presents with rapidly progressive root caries on many teeth. Which of the following
laboratory results would be a possible indicator of this?

A. Stimulated salivary secretion rate of 1.5ml/min


B. S. mutans concentration of 105 organism/ml
C. A plaque sample containing 5% S. mutans
D. A lactobacilli concentration of 105 organism/ml
E. Salivary buffering PH 5.5

http://books.google.co.uk/books?
id=hy3UWiCXglkC&pg=PT133&lpg=PT133&dq=laboratory+investigation+for+rapid+caries&sou
rce=bl&ots=DN6gUZJOZN&sig=GollEcsBs6mXZk9VHZv7yD9ziQs&hl=en&sa=X&ei=JUN_T5
H_JPOp0AWC2p37Bg&ved=0CCQQ6AEwAA#v=onepage&q=laboratory%20investigation
%20for%20rapid%20caries&f=false
1138. How to differentiate between central giant cell granuloma? And ameloblastoma
radiographically?
Though expansion of bone is seen in both.
but in centeral giant cell granuloma  a round or ovoid radiolucency can be seen with well
defined,non corticated margins.
In ameloblastoma multilocular radiolucency is mostly seen.

1139. What kind of neurons supply the masticatory muscle? fast conducting,slow conducting??
They r supplied my v3, so myalinated and fast conducting

1139. Several approaches have been suggested to increase the


fixation of professionally applied topical fluoride, which of the
following statements IS INCORRECT regarding increasing the
fixation
A. Increase concentration of fluoride in solutions
B. Raise the PH of the fluoride solution
C. Increase the exposure time to topical fluoride
D. Pre-treat the enamel with 0.5% phosphoric acid
E. Use NH4F rather than NaF at a lower PH

1140. 89. Which of these muscles may affect the borders of mandibular
complete denture
A. Mentalis
B. Lateral pterygoid
C. Orbicularis oris
D. Levator angulioris
18
E. Temporal
1141. Biopsy is least useful in the diagnosis of
A. Geographic tongue
B. Aphthous ulcer
C. Cysts
D. Granuloma
E. Myeloma

1142. 8 years old child who has sustained a fracture of maxillary


permanent central incisor in which 2mm of the pulp is exposed;
presents for treatment three hours after injury. Which of the
following should be considered
A. Remove the surface 1-2 mm of pulp tissue and place calcium
hydroxide
B. Place calcium hydroxide directly on the exposed pulp
C. Pulpotomy using formocresol
D. Pulpectomy and immediate root filling
E. Pulpectomy and apexification

1143. Fight and flight controlled by ?


Sympathetic nerve supply

1144.
1- Which stage there is histo differentiation 
2- which stage there's morpho differentiation 
Bud stage
Cap
Bell 

Histodifferentiation begins in cap stage - enamel organ, dental papilla, dental follicle
Histodifferentiation continues in bell stage - EEE, SR, SI, IEE
Morphodifferentiation begins in bell stage - Preameloblasts(IEE), Odontoblasts(DP)

1145.
1Hypodontia seen in
A cledocranial dysplasia
B downs syndrom
C papillon le fever syndrom
D rickets

Hypodontia is often familial, and can also be associated with genetic disorders such as ectodermal
dysplasia or Down syndrome. Hypodontia can also been seen in people with cleft lip and palate.
Among the possible causes are mentioned genetic, hormonal, environmental and infectious.
Etiology due to hormonal defects: idiopathic hypoparathyroidism and pseudohypoparathyroidism.[8]
[9]
 Exists the possibility that this defect depends on a moniliasis (candidiasis, candida
endocrinopathy syndrome).[10][11][12]
Environmental causes involving exposure to PCBs (ex.dioxin),[13][14][15] radiation,[16][17][18]
[19]
 anticancer chemotherapeutic agents,[20] allergy [21] and toxic epidermal necrolysis after drug.[22]
Infectious causes of hypodontia: rubella,[23] candida.[24]

2 Hyperdontia can be seen in


cledocranial dysplasia

Hyperdontia is seen in a number of disorders, including Gardner's syndrome and cleidocranial


dysostosis where multiple supernumerary teeth are seen that are usually impacted.

1146. In root canal therapy it is generally accepted that the ideal root filling
> > A. Should extend to the level of the apex to minimize irritation
> > B. Should extend slightly through the apex to ensure a complete seal
> > C. Should extend to the dento cemental junction for healing
> > D. The extension of the filling is not critical
1147. Which blood vessel has 1. MAJOR AMOUNT OF BLOOD
                                 2.MOST ELASTIC VESSEL
                                 3.LEAST SMOOTH MUSCLE CONTAINING VESSEL
                                 4.VESSEL WHICH SERVES AS THERMOREGULATOR
 
 1.vein
2.arteris
3.capillaries
4.terminal arterioles

1147. Patient comes into clinic complaining of sensitivity in mandibular premolar .


Class V amalgam restoration was placed in the tooth four months ago and didn't have discomfort
for 3 months and since then it has become increasingly sensitive , this problem might be due to

A fracture restoration 
B irreversible pulp damage 
C tooth brush abrasion
D marginal leakage

1148. intramembranous ossification


parital 
vomer
maxilla
frontal
nasal bone
lacrimal

mixed-
temporal
occipital
mandible
sphenoid

endochondrial
hyoid
inferior nasal conchae
ethmoid bone

1149.
Please help with those qs about Fluoride concentration 

> 1- child less than 4 years
> 2- child 10 years
> 3- parents
> 4 - duraphat tooth paste 

> Options 
> 1400 ppm
> 2800 ppm
> 1000 ppm
> 1500 ppm
> 500 ppm

Ans:
A)1000
B)1500
C)2800
D)1400

1150. Oral manifestation of cystic fibrosis include all the following except
A tend to be mouth breathers
B salivary gland enlarge
C increase in dental caries
D increase in calculus deposits
E Intrinsic teeth staining due to tetracycline 

Salivary glad enlargement,page 340 Cawsons 

1151. Table 3 Some Actions of the Sympathetic and Parasympathetic Nervous Systems

Sympathetic Parasympathetic
Dilates pupils Constricts pupils
Decreases salvation Increases salvation
Increases respiratory rate Decreases respiratory rate
Increases heart rate Decreases heart rate
Constricts blood vessels Dilates blood vessels
Inhibits digestion Stimulates digestion
Relaxes bladder muscles Contracts bladder muscles
Inhibits defecation Stimulates defecation

You might also like